APhA NAPLEX Review Questions

Réussis tes devoirs et examens dès maintenant avec Quizwiz!

Which of the following drugs is used to treat Cushing's disease? A. clotrimazole B. propylthiouracil C. mitotane D. vasopressin E. prednisone

C. Mitotane is used to treat Cushing's dx by suppressing ACTH from anterior pituitary and by reducing synthesis of cortisol. Vasopressin is used to treat diabetes insipidus. Clotrimazole is an topical antifungal.

Which of the following is true for moderate to severe CD? A. azathioprine is the drug of choice for initial therapy B. budesonide is appropriate maintenance therapy C. oral corticosteroids are the drugs of choice for initial therapy D. mtx has no role in CD E. topical aminosalicylates are appropriate maintenance therapy

C. Moderate to severe disease initially requires oral corticosteroid therapy. Corticosteroids have no role in maintenance therapy. Topical aminosalicylates may be used as adjuncts in colonic CD. MTX is used as maintenance therapy for mod to severe CD

Which of the following correctly characterizes Tc-99m HDP? A. localizes in areas of high bone metabolic activity B. limited to use in adult patients with metastatic lesions C. frequent adverse reactions D. lipophilic cationic complex

A. Tc-99m HDP can be used in the pediatric population, and adverse reactions are uncommon. It is a diphosphonate complex but not a lipophilic complex

Androderm is administered A. once daily B. three times per week C. once weekly D. every 2 weeks E. monthly

A. Testosterone transdermal systems (Androderm, Testoderm) are applied once daily for 24hrs. Longer-acting androgens are available, such as nandrolone decanoate (Deca-Durabolin) for once weekly administration

Which of the following is best described as the repair or correction of a dysfn gene by selectively introducing rDNA into cells or tissues (ultimately leading to the expression of a functional gene product)? A. monoclonal antibody therapy B. gene therapy C. antiviral therapy C. cell therapy E. rDNA therapy

B. Gene therapy

Which of the following agents is used in cancer regimens but is NOT considered an antineoplastic agent? A. mtx B. leucovorin C. doxorubicin D. cyclophosphamide

B. Leucovorin is a reduced folate agent that is used in combination with 5FU to potentiate the therapeutic effects of 5-FU and as a rescue treatment for high-dose mtx

Which of the following fluoroquinolones covers both s. pneumo and p. aeruginosa A. moxifloxacin B. levofloxacin C. ciprofloxacin D. ofloxacin

B. Levofloxacin displays broad-spectrum activity against both pneumococci and p. aeruginosa. In contrast, cipro does not display reliable activity against pneumococci, and moxifloxacin does not display reliable activity against P. aeruginosa

Excessive doses of levothyroxine may cause A. weight gain B. osteoporosis C. cold intolerance D. bradycardia E. sedation

B. Levothyroxine decreases bone mineral density and when given in supratherapeutic doses may cause osteoporosis. For this reason, the lowest possible replacement dose should be administered. Doses should be given to maintain euthyroid (normal TSH ranges)

70yo bm PMH: DM, HTN, OSA, OA VI: 248lb, 5'11'' SH: smokes 1.5ppd, 4-5 cups coffee, 2 drinks/w EtOH Education regarding lifestyle modification issues in this pt should include all of the following except A. limit alcohol intake to no more than 2 drinks/d B. reduce daily intake of dietary mg, ca, na C. increase aerobic physical activity, decrease weight, and limit dietary saturated fat and cholesterol D. consume diet consistent with the DASH diet E. stop smoking

B. Lifestyle mod issues to be considered in htn pts include weigh loss, limit alcohol, increased aerobic activity, reduce sodium intake, maintenance of adequate dietary potassium, calcium, and mg intake, and smoking cessation

Common fluid and electrolyte abnormalities associated w/ loop diuretics include all of the following except A. hypokalemia B. hyperkalemia C. hypomagnesemia D. dehydration E. hypocalcemia

B. Loop diuretics enhance renal excretion of water, potassium, mag, and calcium

Which of the following best describes GI side effects of antacids containing mg and ca salts? A. mg causes constipation; ca causes diarrhea B. mg causes diarrhea, ca causes constipation C. both cause diarrhea D. both cause constipation E. neither has GI ade

B. Mag salts are often used as osmotic laxatives and may cause diarrhea. Calcium salts cause constipation

MeSH terms are A. not appropriate to include in a search strategy of the secondary lit B. controlled vocabulary developed by the US Nat'l Library of Medicine C. the same as keywords D. useful for identifying drug information in tertiary references

B. MeSH terms are controlled vocabulary developed by the US Nat'l Library of Medicine

Close monitoring of adrenal hormone secretion may be required when administering which of the following? A. methyltestosterone B. mitotane C. desmopressin D. iodides E. propylthiouracil

B. Mitotane is cytotoxic to adrenal cells and thus reduces cortisol synthesis and release. ACTH increased cortisol release. Close monitoring of cortisol levels is important when mitotane is used.

Which of the following is most likely to cause cellulitis A. candida albicans B. streptococcus species C. shigella boydii D. e. coli E. klebsiella pneumo

B. Most cellulitis infx are associated w/ strept

Which of the following medications inhibits signal 3 by binding to the CD25 subunit of the IL2 receptor? A. antithymocyte globulin (rabbit) B. antithymocyte globulin (equine) C. basiliximab D. everolimus E. belatacept

C. Basiliximab selects for destruction of activated lymphocytes by binding to the CD25 subunit of the high-affinity IL-2 receptor

Which of the following is best defined as the application of computer sciences and information technology to the management and analysis of biological information? A. biometrics B. biotherapy C. bioinformatics D. biostatistics E. biotechnology

C. Bioinformatics

Which of the following are examples of secondary lit? A. Medline and AHFS drug information B. Embase and Facts and Comparisons C. Medline and Embase D. AHFS DI and Facts and Comparisons

C. MEDLINE and Embase are examples of secondary lit. Other types of secondary lit include Internationals Pharmaceutical Abstracts (IPA) and Iowa DI Services (IDIS)

What is the drug of choice for enhancing gastric emptying in a pt receiving EN support? A. bismuth subsalicylate B. azithromycin C. metoclopramide D. loperamide E. acyclovir

C. Metoclopramide enhances gastric emptying and is commonly used in pts w/ gi intolerance. This is true in both dm and nondm

Which of the following diagnoses carries an increased risk for developing esophageal adenocarcinoma? I. typical reflux syndrome II. reflux cough syndrome III. reflux laryngitis IV. non-troublesome sx of GERD V. Barrett's esophagus A. I, II, and III B. I and III C. II and IV D. V only E. all of the above

D. Pts w/ Barrett's esophagus have an increased risk of developing esophageal adenocarcinoma

Which of the following is not an acceptable technique for managing data from those patients who withdrew from a study prior to completing the study protocol? A. analyzing the data using an intent-to-treat method B. analyzing the data using a per protocol C. analyzing the data using both intent-to-treat and per protocol D. analysing the data from those subjects completing the protocol following purging of withdrawal subject info from the study database

D. Pts who withdraw from a study prior to completing the study protocol can be managed by analyzing data using an intent-to-treat method and both intent-to-treat and per protocol methods and by categorizing pts according to reason for withdrawal using a per protocol method. Purging data from the study database upon withdrawal from the study and including only subjects completing the protocol is inappropriate and is the exception

Which of the following medications is a polyclonal antibody? A. belatacept B. basiliximab C. mycophenolate sodium D. rabbit antithymocyte globulin E. sirolimus

D. Rabbit antithymocyte globulin is a polyclonal antibody used in treatment of acute rejection

Which of the following drugs may increase the risk of osteoporosis? A. hctz B. warfarin C. lisinopril D. oral contraceptives E. depo-provera

E. Depo-provera may have a negative effect on bone mineral density. Hctz may actually have a beneficial effect on bone as it decreases calcium ion loss in the urine

Which of the following will NOT increase BP via a1 adrenergic activation? A. phenylephrine B. dopamine C. epinephrine D. norepinephrine E. dobutamine

E. Dobutamine is pure beta activation, Epi is potent a and B, Norepi is potent a and some B

Which of the following agents produces an addition 15% LDL reduction when combined with a statin? A. colesevelam B. nicotinic acid C. 15g soluble fiber D. fenofibrate E. ezetimibe

E. Ezetimibe when used in combination with a statin, produces an additional 15% reduction in LDL

A 62 yo pt with a h/o htn and gout presents to being pharmacotherapy for htn. Which agent is the most appropriate choice as initial therapy based on JNC8? A. chlorothiazide B. thiazide C. tenormin D. chlorthalidone E. losartan

E. For pts w/ HTN and gout, do not use diuretic therapy which increases the risk of gouty attacks. The only medication listed that is not a diuretic is atenolol which is a BB, and losartan which is an ARB. BB are not first line, so go with losartan

Which of the following is used to treat adrenal crisis? A. cosyntropin B. aminoglutethimide C. fluoxymesterone D. vasopressin E. hydrocortisone

E. Hydrocortisone is the drug of choice for adrenal crisis b/c it posses BOTH mineralocorticoid and glucocorticoid properties. Although cosyntropin increases cortisol release, pts w/ adrenal crisis may not have enough adrenal reserve to meet their increased demand. Aminoglutethimide is used in Cushing's and decreases cortisol levels by inhibiting conversion of cholesterol. Vasopressin is for diabetes insipidus.

Which of the following is NOT available in both IV and po? A. metoprolol B. amio C. verapamil D. digoxin E. ibutilide

E. Ibutilide is IV only

A pt presents to your ambulatory clinic w/ a BP 210/125, PMH: DM2, CHF, CKD What would be recommended treatment for the pt w/ htn emergency? A. Clonidine orally, 0.1-.2mg, repeat in 1-2hr PRN (max 0.6) B. Labetalol po, 100-400mg, repeat in 2-3hr PRN C. Nifedipine SL, 10mg, repeat in 0.5-1hr PRN (max 60) D. Labetalol IV, 20-80mg bolus, followed 0.5-2mg/min infusion E. Enalaprilat IV, 1.25-5mg q6hr

E. In a pt w/ CHF and HTN emergency, recommended treatments include nitro, nitroprusside, and enalaprilat. Clonidine and labetalol are incorrect choices b/c need IV therapy. Nifedipine SL is not indicated for immediate reduction of BP. Labetalol IV is not an appropriate choice in this pt w/ CHF b/c could decrease CO.

Identify the di that involves the cyp system A. ezetimibe and niacin B. colestipol and simvastatin C. gemfibrozil and cholestyramine D.. fenofibrate and ezetimibe E. lovastatin and itraconazole

E. Lovastatin is metabolized by CYP3A4 and itraconazole will inhibit this enzyme system. Inhibition causes lovastatin concentrations to rise, thus predisposing the pt to muscle or liver toxicity

Which of the following drug interactions may result in increased pharmacologic effect of estrogen? A. macrolide antibodies B. itraconazole C. ketoconazole D. a and c E. a,b, and c

E. Macrolides, and azoles may result in increased pharmacologic effect of estrogen by inhibiting CYP 3A4

Which of the following drugs works by decreasing cortisol synthesis? A. Cortrosyn B. ACTH C. oxandrolone D. prednisone E. metyrapone

E. Metyrapone (Metopirone) works to decrease cortisol synthesis by inhibition of 11-hydroxylase activity

ST is a 32yo woman who wants to begin using a prescription contraceptive product. She is a new mother and would like to know if any products are safe for using during breast feeding. ST states that she is not interested in using a device intravaginally and experiences irritation and inflammation with condom use. Which of the following products would be appropriate for ST? A. ortho tri-cyclen B. micronor C. depo-provera D. seasonale E. B and C

E. Micronor is a progestin-only oral contraceptive and is considered compatible with breastfeeding. Depo-provera is an injectable progestin-only contraceptive option that is considered safe and appropriate for women who desire to breastfeed because it does not affect milk production or adversely affect infant development. Ortho Tri-cyclen is a combined oral contraceptive that may decrease the quantity of breast milk available and may adversely affect the infant.

For the treatment of chronic afib, warfarin therapy with target INR of 2-3 would be appropriate for each of the following pts except A. pts with HF and DM B. pts over 75yo w/ htn C. a 50yo male with no risk factors for thromboembolism D. a 77yo female with DM and HTN E. a 63yo male who has had a previous stroke

C. Pts under 75 w/ no risk factors for stroke do not require antithrombotic therapy

Which of the following research design types is NOT considered experimental research? A. cohort study B. RCT C. case control study D. retrospective epidemiologic study

D. Experimental research design include cohort studies, rcts, and case-controlled studies. Retrospective epidemiologic studies are observational and are the exception

Which of the following radiopharmaceuticals is a monovalent cation and a potassium analog? A. F-18 FDG B. Tc-99m tetrofosmin C. Tc-99m sestamibi D. Thallous chloride T1-201

D. F-18 FDG is a glucose analog; Tc-99m tetrofosmin and Tc-99m sestamibi are monovalent cations but not potassium analogs; thallous chloride T1-201 is a monovalent cation and a potassium analog

Which of the following PET radiopharmaceuticals is commercially available from a generator? A. F-18 FDG B. I-23 ioflupane C. Florbetapir F-18 D. Rb-82 chloride

D. F-18 and I-123 are not available from a generator

Which of the following PET radiopharmaceuticals is used for assessing malignant and benign skeletal disease? A. F-18 FDG B. Florbetapir F-18 C. Rb-82 chloride D. F-18 sodium fluoride

D. F-18 sodium fluoride; F-18 FDG is used in the assessment of other oncology pathologies as well as cardiac dx; florbetapir F-18 is used in estimating density of b-amyloid plaques in pts being evaluated for cognitive impairment; Rb-82 chloride is used in cardiac studies

Which of the following medications is classified as a calcineurin inhibitor? A. sirolimus B. everolimus C. belatacept D. azathioprine E. cyclosporine

E. Cyclosporine is a CI

All of the following increase the risk of acute rejection EXCEPT A. pediatric recipient B. HLA mismatch C. living donor D. noncompliance E. h/o previous transplantation

C. Living donor

Which of the following agents can cause red-colored staining of body fluids A. rifampin B. isoniazid C. pyrazinamide D. ethambutol

A. An important pt counseling point for pts being administered rifampin is to be aware of discoloration of body fluids such as tears, sweat, and saliva. Pts who wear contact lenses, for example, should be notified that the lenses may become permanently discolored

On the basis of the pt's weight and height (180cm, 200lb), you calculate the pt's body surface area to be 2.1m2. Paclitaxel is supplied as 6mg/mL in 5mL, 16.7mL, and 50mL vials. Your pharmacy has all quantities available. What is the best way to correctly dose the pt? Paclitaxel dose: 175mg/m2 over 3hr q3week A. 1x 50mL vial and 1x 16.7mL vial B. 1x 50mL vial and 1x 5mL vial C. 2x 16.7mL vials D. 2x 16.7mL vials and one 5mL vial

A. 175mg/m2 x 2.1m2 = 367.5mg dose 367.5mg/ 6mg/mL = 61.25mL So need 50+16.7mL vial

What is the max duration of therapy for ketorolac? A. 5d B. 7d C. 14d D. 30d E. there are no restrictions on length of use

A. 5d d/t risk of GI bleeding

An SNP always results in a change in which of the following? A. the nucleotide sequence in the genome B. the nucleotide sequence of a codon C. the encoded amino acid of the codon D. the encoded amino acid of the codon, with no change in fn of the encoded protein E. the encoded amino acid of the codon, with a clinically relevant change in the function of the encoded protein

A. A SNP may occur outside of an open reading frame (coding region), it may induce a mutation where no change is encoded amino acid occurs, and it may or may not cause a functional change in an encoded protein

Which of the following statements is NOT true for variables? A. a continuous random variable has interruptions or gaps in its values B. height is a quantitative variable C. blood pressure can be a random variable D. weight is a continuous variable

A. A continuous random variable does not have interruptions or gaps in its values, or it can assume any value within an interval between any 2 of its values

Positive: 30 (present), 40 (absent), 70 (total) Negative: 20 (present), 10 (absent), 30 (total) What is the predictive value negative for the test? A. 10/30 B. 10/50 C. 30/50 D. 30/70

A. A predictive value negative of a test is the probability of an individual not having the disease when the test result is negative (negative absent)/ (negative total)

In a pt with mildly symptomatic PSVT, verapamil should be used for which of the following rhythms? A. narrow QRS complex, regular interval B. wide QRS complex, regular interval C. wide QRS complex, irregular interval D. all wide QRS complexes E. narrow QRS complex and hypotension

A. A wide QRS complex signifies conduction via an accessory pathway other than the AV node. B/c ccb prolong conduction in the av node and not in the accessory pathways, administration of these agents will block the av node and force impulses to be conducted via the accessory pathways, which have shorter refractory periods. Consequently, the ventricular response will significantly increase. In the presence of hypotension, dccv should be used, verapamil will worsen hypotension.

Which of the following hormones is secreted by the pituitary gland? A. adrenocorticotropic hormone B. testosterone C. cortisol D. thyroxine E. corticotropin-releasing hormone

A. ACTH is secreted by the anterior pituitary gland and acts on the adrenal glands to increase release of cortisol. Corticotropin-releasing hormone is released by the hypothalamus and acts on the pituitary to stimulate ACTH release

What is the most common ade caused by the acarbose? A. flatulence B. hypoglycemia C. renal failure D. hyperglycemia E. weight gain

A. ADE of acarbose: flatulence, abd pain, and diarrhea. Decrease ADE by titrating dose gradually and taking drug w/ first bite of each meal

Which of the following ADE of lisinopril can be avoided by switching to candesartan? A. cough B. renal insufficiency C. hyperkalemia D. hypotension E. injury or death to developing fetus

A. ARBs can cause renal insufficiency, hyperkalemia, hypotension, fetal death same as ACEIs. Not associated with bradykinins though

Which of the following products is indicated for prevention of blood clots post-PTCA? A. Abciximab B. Basiliximab C. Infliximab D. Trastuzumab E. Becaplermin

A. Abciximab. Basiliximab is indicated for management of acute organ transplant rejection. Infliximab is indicated for the treatment of Crohn's dx and RA. Trastuzumab is indicated for the management of metastatic breast cancer. Becaplermin is indicated for treatment of diabetic foot ulcers.

What is the recommended minimum leukocyte count prior to a patient receiving strontium-89 chloride? A. 2,400 B. 6,200 C. 8,800 D. 10,000

A. Any level over 2400 is above the minimum level of leukocytes recommended before a pt receives Sr-89 chloride

Which of the following is an important pt counseling pt for a pt receiving a fluoroquinolone antibiotic? A. avoid antacids or calcium-containing substances w/in 2hr of administration B. take only on an empty stomach C. take only with a high fat meal or an acidic beverage D. avoid ppi w/ use

A. Absorption and drug concentrations can be greatly reduced by the concomitant administration of cations that can bind fluoroquinolone abx. This is an important pt counseling point to explain to pts to reduce the chances of treatment failure

According to the Noyes-Whitney equation, the drug concentration immediately at the solid surface within the diffusion layer (stagnant layer) is recognized as the A. drug solubility B. dissolution rate C. thickness layer D. diffusion coefficient

A. According to the Noyes-Whitney equation, the drug concentration in the stagnant layer is equal to the drug solubility

On binding of antigen displayed by the antigen-presenting cell to the T-cell receptor complex, what additional step is required for T-helper cell activation? A. binding of the costimulatory pathway (ex: CD58/CD2) B. activation of the promoter gene NFAT C. transcription of the IL-2 gene D. no additional step required E. translation of IL-2

A. Activation is dependent on antigen-HLA binding to the Tcell receptor complex and the subsequent binding of a second signal or "co-stimulatory pathway"

Which of the following agents is most appropriate for a pt with stage 5ckd on hd, secondary parathyroidism (PTH 800) and hypercalcemia (corrected calcium 10.5) who requires treatment to reduce PTH? A. cinacalcet B. paricalcitol C. calcitriol D. ergocalciferol

A. Active vitamin D (calcitriol) and vitamin D analogs (paricalcitol and doxercalciferol) may worsen hypercalcemia in this pt through increased GI absorption and are not recommended until the calcium levels are lowered. Cinacalcet works to decrease secretion of PTH and does not cause hypercalcemia (risk hypocalcemia). This agent is approved only in pts w/ CKD on HD so would be appropriate in this case. Vit D should be considered when this pt's calcium lvls are within normal range. Ergocalciferol is a vit d precursor that requires activation by the kidney and would not be recommended as the sole vit d agent for a pt with stage 5ckd w/o the necessary activity of the enzyme in the kidney responsible for final conversion to the active form. Ergocalciferol would also carry the risk of hypercalcemia if converted to the active form.

71yo wf, 5'4'', 150lbs PMH: HF (recent gain 20lbs over past month), HTN, OA LVEF: 25, BP 130/85, HR 80, Digoxin: 0.8ng/mL Meds: lanoxin 0.125 qd, lasix 80 qd, zestril 20 qd, toprol 50 qd, naproxen 500 bid cc Which of the following medications could improve this pt's sx and survival? A. spironolactone B. amlodipine C. verapamil D. dobutamine weekly infusion E. warfarin

A. Addition of aldosterone antagonists has now been shown to improve survival and decrease sx in pts w/ HFrEF. Amlodipine would have no effect, and verapamil would likely worsen HF d/t negative inotropic effects. Weekly dobutamine infusions do not improve survival

How soon after the onset of severe sepsis or septic shock should antibiotic therapy be administered? A. 1hr B. 3hr C. 4hr D. 6hr E. 12hr

A. Admin as soon as possible. Waiting past 6hr increases mortality risk by 9% every hour

Which of the following meds is considered to a DMARD? A. plaquenil B. allopurinol C. belimumab D. nalfon

A. Allopurinol is a XO inhibitor used in the treatment of gout. Nalfon is an NSAID. Belimumab is a mAb used in treatment of SLE

Which of the following is indicated for IBS-C? A. tegaserod 6mg bid B. alosetron 1mg bid C. loperamide 2-16mg/d D. paroxetine 10-40mg/d E. diphenoxylate + atropine 2tabs qid

A. Alosetron is approved for restricted use in IBS-D. Loperamide and diphenoxylate + atropine are antidiarrheal medications that will exacerbate constipation. Paroxetine is an SSRI with potent anticholinergic effects, which could worsen constipation. Another drug should be chosen if an SSRI is needed for depression

Alosetron is indicated for which group of IBS pts? A. women w/ IBS-D B. men w/ IBS-D C. women w/ IBS-C D. mean w/ IBS-D E. children w/ IBS-D

A. Alosetron is approved for women w/ IBS-D. it is not found to be effective in men and is not approved for use in children

39yo f presents w/ mild abd pain and diarrhea x 12w. No significant PMH and occasionally misses days from her full-time job. Her sx are worse after meals. Her sx are controlled for several months until she loses her job. Her abd pain then returns. She has 5 episodes of diarrhea per day and complains of fatigue and insomnia. Which of the following are the most appropriate for this pt? I. initiate psychological counseling II. prescribe amitriptyline 10-50mg qhs III. add loperamide 2mg after each loose stool (max 16/d) IV. add alosetron 1mg qd V. start tegaserod 6mg bid A. I, II, and III B. I and III C. II and IV D. V only E. all of the above

A. Alosetron is reserved for pts w/ severe, IBS-D who have failed other therapies. Several types of psychotherapy have been shown to be more effective than usual care in IBS. TCAs improve abdominal pain in IBS and may also help with insomnia. Loperamide may be used on a PRN basis Tegaserod is for IBS-C

Which of the following abx may have a higher incidence of predisposing a pt for a C. diff infection? A. clindamycin B. fidaxomicin C. metronidazole D. oral vancomycin

A. Although all antibiotics can cause C.diff, clincadymcin's spectrum of activity covers many anaerobes except for C.diff. For this reason, it is commonly implicated as a predisposition for C. diff infx

Recombinant DNA technology has led to the development of vaccines for which of the following diseases? A. Hepatitis B B. Hepatitis A C. Haemophilus influenzae type B infection D. Malaria E. AIDs

A. Although promising, this technology has not yet yielded vaccines for hepatitis A, malaria, AIDS, or infx d/t H.flu type B

Which of the following B-lactams antibiotics displays the best activity against enterococci? A. ampicillin B. cefepime C. meropenem D. aztreonam

A. Ampicillin is a drug of choice for enterococcus that are susceptible. No cephalosporins display reliable activity against enterococci

64yo wm, 5'11'', 185lbs PMH: MI, HTN, HF, HLD LVEF: 30, HR 88, BP 145/90 Meds: lanoxin 0.125 qd, lasix 40 qd, kcl 20 qd, zocor 40 qhs, ASA, plavix 75 qam Which of the following meds should be added to the regimen? A. lisinopril and carvedilol B. valsartan and prazosin C. torsemide and amlodipine D. verapamil and amio E. clonidine and hctz

A. An ACEI and BB are indicated in this pt with HF to improve survival and slow dx progression

According to JNC8, which of the following agents are suitable as initial therapy for the treatment of uncomplicated htn? A. hctz, chlorthalidone, amlodipine B. chlorthalidone, atenolol, amlodipine C. amlodipine, hctz, atenolol D. atenolol, amlodipine, chlorthalidone

A. Appropriate choices for initial agents in the treatment of uncomplicated HTN include diuretics, ACEIs, ARBs, and CCBs. Hydralazine is a direct vasodilator and atenolol is a BB, which are not first line options.

Which of the following questions is NOT an aid in evaluating a clinical trial? A. were all confounding variables avoided in the study? B. did the study group have similar baseline characteristics? C. how large was the treatment effect? D. are the benefits worth the harm and costs?

A. Asking if the groups have similar baseline characteristics, how large was the treatment effect, and if the benefit of the treatment are worth the harm and costs are aids in evaluating a clinical trial. Determining if all confounding variables were avoided in a study is not an aid in evaluating a trial

Which of the following is CI in pts with a history of lisinopril-induced angioedema? A. captopril B. torsemide C. spironolactone D. milrinone E. carvedilol

A. Avoid other ACEIs, monitor if starting ARBs

Pts presenting with acute bronchitis without risk factors should be treated empirically with A. supportive care B. clarithromcyin C. cefuroxime D. ciprofloxacin E. erythromycin

A. B/c half of bronchitis infx are caused by viral etiology, antibacterial therapy for low-risk pts should not be attempted unless severe presentation

Which of the following is true regarding phenytoin? A. the max rate of IV admin is 50mg/min B. it IV access cannot be established, phenytoin can be given IM C. b/c phenytoin contains propylene glycol, it is soluble as any IV fluid D. it is an inhibitor of the cyp system E. a major limitation to the use of the product in pediatric pts is the lack of a commercially available liquid formulation

A. B/c phenytoin contains propylene glycol and it is itself cardiotoxic, the IV formulation should not be infused faster than 50mg/min. Phenytoin is extremely alkaline (pH -13) Not only is IM admin associated w/ tissue damage, but also it is erratically absorbed. Phenytoin can be admixed only with normal saline, is an inducer, and is also available as a suspension and a chewable tablet

Which of the following medications can cause bradycardia? A. carvedilol B. furosemide C. ramipril D. milrinone E. dobutamine

A. BB is associated with decreased HR and bradycardia. Milrinone/dobutamine can increase HR, lasix and ramipril do not affect HR

Synthroid requires which counseling? A. take 4hrs before or 4hr after Questran B. take w/ food C. watch for sx of infx D. take PRN to keep her desired lvl of energy E. d/c if she experiences nausea

A. Bile acid sequestrants (Questran) reduce levothyroxine absorption and must be separated from administration by at least 4 hours. Levothyroxine should be administered before a meal on an empty stomach to maximize absorption.

What is bioavailability? A. Bioavailability is the measurement of the rate and extent of active drug that reaches the systemic circulation B. It is the relationship between the physical and chemical properties of a drug and its systemic absorption C. It is the movement of the drug into body tissues over time D. It is the dissolution of the drug in the GI tract

A. Bioavailability is the measurement of the rate and extent of systemic circulation of an active drug

Which of the following agents is considered first line therapy for postmenopausal osteoporosis? A. risedronate B. calcitonin C. prempro D. denosumab E. teriparatide

A. Bisphosphonates are first line therapy b/c they reduce risk of fracture

From the following choices, select the most common side effect associated with estrogen therapy A. breast tenderness B. depression C. diarrhea D. brittle fingernails E. hair loss

A. Breast tenderness and nausea are the most common ade associated with estrogen therapy. Headache and depression are associated with progestin therapy

Which of the following is CI in anorexia and bulimia? A. bupropion B. sertraline C. citalopram D. clonazepam E. fluoxetine

A. Bupropion is CI d/t risk of seizures

A young man (73kg, 28yo, clcr 124ml/min) recieves a single 200mg po antibiotics. The following pk paramaters of the antibiotic are reported in the literature: F=90%, Vd=0.31L/kg, t1/2=2.1h, fup=0.77, 67% antibiotic's absorbed dose is excreted unchanged in the urine. Determine the renal cl of the antibiotic. What is the probable mechanism for renal cl of this drug? A. 84ml/min, glomerular filtration and tubular reabsorption B. 98ml/min, GF and TR C. 112ml/min, GF D. 167ml/min, GF and TR E. 236ml/min, GF and tubular secretion

A. CL = Vd x ln2/(t1/2) CL= 0.31L/kg x 73kg x ln2/2.1 = 7.47L/h CLrenal = fe x CL = 0.67 x 7.47 = 83.3mL/min Eratio = CLrenal/ fup x GFR Eratio = 83.3/ 0.77 x 124ml/min = 0.87 Eratio < 1 indicates that glomerular filtration and net reabsorption are the probable renal clearance mechanisms

J.D is a 47yo wm who has been prescribed codeine for lower back pain. The pharmacist dispensing the medication remembers reading a study in which pts who took codeine with grapefruit juice experienced an enhanced analgesic effect. The study found that grapefruit juice enhanced oral bioavailability (F) of codeine. Interestingly, there was no effect on codeine hepatic clearance or volume of distribution. Thus, the pharmacist counseled the pt not to take his codeine with grapefruit juice. Based on the pk data for codeine, what is the most likely explanation for the enhanced oral bioavailability of codeine? CL: 1350ml/min, fe: 0.1, Vss: 3.3L/kg, plasma pr binding: 35% A. grapefruit juice increases the absorption (fa) of codeine B. grapefruit juice decreases the fraction escaping first-pass metabolism (F*) C. grapefruit juice increases renal secretion of codeine D. grapefruit juice increases the fraction escaping first-pass metabolism (F*)

A. CLhepatic=1350 x 0.9 = 1215mL/min ER= (1215mL/min)/ (1350mL/min) = approx 1 F= fa x fg x F* No effect on CL hepatic. No effect on F*

If excessive amounts of calcium are added to a standard PN formulation, it will likely precipitate with A. phosphate B. gluconate C. magnesium D. chloride E. sodium

A. Calcium phosphate is a relatively insoluble compound, so manufacturer guidelines for the concentrations of these two elements must be followed closely to prevent precipitation. The order of mixing these components in the PN formulation is also important

Which of the following medications has the greatest efficacy in raising HDL levels? A. lovastatin B. pravastatin C. gemfibrozil D. nicotinic acid E. colesevelam

D. Nicotinic acid (Niaspan) has efficacy in raising HDL cholesterol by 15-35%

For a transfer of product into the controlled area, A. bottles, bags, and syringes must be removed from brown cardboard boxes before being brought into the buffer area B. vials stored in laminated cardboard may not be brought into the controlled area C. stainless steel carts may be used to transfer items into the controlled area directly from the storage area D. large-volume parenteral bags of IV solution must be removed from their protective overwrap before being brought into the controlled area E. the refrigerator should be placed next to the laminar flow hood for easy access

A. Cardboard must be kept out of the buffer area. Vials stored in laminated cardboard may be stored in the buffer area. No items should be brought into the buffer area without being sanitized. Large-volume parenteral bags should be removed from their overwrap just before being used. The refrigerator should not be in the buffer room because it is a source of contamination

Which of the following clinical study design characteristics is consistent with a case report? A. observational B. prospective C. randomized D. controlled

A. Case reports are always observational. Prospective designs, randomization, and controls are all study design characteristics commonly used with randomized clinical trials and are not associated with case reports

A pt with a bone fracture and gr negative pneumonia excretes 15g (normal 6-8) of urea nitrogen during a 24hr urine collection. On the basis of these data, the pt is A. hypercatabolic B. hypermetabolic C. hypocatabolic D. hypometabolic E. euvolemic

A. Catabolism is related to loss of body protein. b/c urea nitrogen output is increased, the pt would be considered hypercatabolic in this case

Which of the following is NOT a secondary cause of dyslipidemia? A. high LDL cholesterol B. hypothyroidism C. diabetes D. renal dx E. bb

A. Causes of dyslipidemia must be ruled out. The common secondary causes are renal failure; hypothyroidism; obstructive liver dx; diabetes; and drugs such as BB, thiazide diuretics, ocps, oral estrogens, glucocorticoids, and cyclosporine

Which of the following describes a known characteristic of most antineoplastic chemotherapy agents? A. they have a narrow TI B. they do not interfere w/ DNA synthesis and replication C. acute ade occur primarily in slowly dividing normal cells D. they have only phase-specific actions

A. Chemotherapy agents have a very narrow TI index. This is one of the main reasons these drugs have so many toxic effects. They can be phase-specific or phase-nonspecific drugs and can cause many ade to normal cells that undergo rapid proliferation

25yo wm has been increasingly disruptive at home. He has been claiming that he is president of the US and has been staying awake all night planning bills for Congress. He has been argumentative and threatening with his friends and family. Though he appears extremely tired physically, he is constantly active. As the pharmacist on the treatment team, you alert the pt's family to which of the following as a common ade of lithium? A. polyuria B. weight loss C. elevated hepatic enzymes D. akathisia E. euphoria

A. Common ade that occur with the initiation of lithium include polyuria, polydipsia, tremor, and GI upset. Common ade that may occur late in therapy include weight gain and mental dulling. Elevated hepatic enzymes and alopecia are side effects that may occur with divalproex sodium .Akathisia is an EPS effect induced by antipsychotics

Cough is an ADE associated with with of the following medications? A. ramipril B. valsartan C. carvedilol D. torsemide E. eplerenone

A. Cough is common ADE of ACEIs d/t bradykinin accumulation

The intent-to-treat analysis includes data from which subjects? A. all pts regardless of whether they completed B. only pts who complete a specified protocol C. only pts who complete the protocol with favorable outcomes D. none of the pts who did not complete the entire trial

A. Data from all pts regardless of whether they completed the protocol

45yo female presents w/ aki. Has had difficulty over past 5d keeping down anything she eats or drinks. Decrease in urine over past 24hr. PMH: DM2, RA. Meds: asa 325, ibu 600mg qd, metformin, glyburide, apap Labs: Scr 2mg/dL, BUN 56 (normal 1 mo ago) The most likely etiology of this pt's aki is A. dehydration from poor oral intake B. age-related decreases in kidney function C. kidney failure caused by dm D. obstruction of urine outflow

A. Dehydration is the most likely cause of aki in this pt b/c she has had a decrease in oral intake over the past 5d. Dehydration would be classified as a prerenal cause of aki. A serum creatinine of 2mg/dL would not be considered normal in a person age 45yo, eliminating age as a rationale for kidney dx. DM would be more likely to cause a chronic decrease in kidney fn as opposed to an acute change. She has had some urine output in the past 24hr which rules out obstruction

Which of the following antiarrhythmic agents requires dosage adjustment in pts with impaired renal fn? A. digoxin B. amio C. lidocaine D. verapamil E. propafenone

A. Digoxin

Which of the following medications would be preferred for control of ventricular response in pts with afib and HF? A. digoxin B. verapamil C. diltiazem D. amlodipine E. dofetilide

A. Digoxin would be first choice b/c it will help slow the ventricular rate and it does not have negative inotropic effects. Both verapamil and dilt are negative inotropes and should not be used in HFrEF. Dofetilide is used for conversion to and maintenance of sinus rhythm, not rate control

Which of the following is not true for tablet formulations? A. A disintegrating agent promotes granule flow B. Lubricants prevent adherence of granules to the punch faces of the tableting machine C. Glidants promote flow of the granules D. Binding agents are used for adhesion of powder into granules

A. Disintegrating agents are added to the tablets to promote breakup of the tablets when placed in the aqueous environment. Lubricants are required to prevent adherence of the granules to the punch faces and dies. Glidants are added to tablet formulations to improve the flow properties of the granulations. Binding agents are added to bind powders together in the granulation process

Which of the following will increase dronedarone plasma concentrations and should be avoided in pts taking this medication? A. grapefruit juice B. ranitidine C. atenolol D. nitro E. warfarin

A. Dronedarone is metabolized by CYP3A4 and grapefruit juice is a potent CYP3A4 inhibitor. results in significant increases in dronedarone plasma concentrations and risk ade

Which of the following is the general vasopressor of choice in septic shock? A. dopamine B. epinephrine C. phenylephrine D. norepinephrine E. vasopressin

D. Norepi

Which of the following well-tolerated antifungal agents is commonly used to treat disseminated candidiasis caused by Candida krusei? A. caspofungin B. ampho B C. posaconazole D. fluconazole

A. Echinocandins are drugs of choice for disseminated candidiasis caused by non-albicans or fluconazole-resistant Candida spx. Echinocandins are often initiated as empiric therapy for disseminated candidiasis until the fungal spx is known by culture

Which of the following biotechnology agents is indicated for treating anemia caused by chronic renal dx? A. Epoetin alfa B. Becaplermin C. Filgrastim D. Alemtuzumab E. Sargramostim

A. Epoetin alfa. Becaplermin is indicated for the management of diabetic foot ulcers. Filgrastim is indicated for neutropenia. Alemtuzumab is indicated for the treatment of CLL. Sargramostim is indicated for myeloid reconstitution after bone marrow transplant; after bone marrow transplant failure, as an adjunct to chemo in AML, and in peripheral blood progenitor cell transplant

Which of the following medications is associated with torsades de pointes? A. erythromycin B. ampicillin C. atenolol D. verapamil E. propafenone

A. Erythromycin can prolong the QTc interval and increase the risk of torsades

Which of the following medications interacts with sirolimus? A. erythromycin B. metoprolol C. furosemide D. nifedipine E. gentamicin

A. Erythromycin inhibits 3a4 which metabolizes sirolimus

When the addition of an active ingredient lowers the melting point of cocoa butter used as a suppository base, what percentage of white wax can be used to replace an equal amount of the base to restore the melting point? A. 5 B. 10 C. 15 D. 25

A. Five percent cocoa butter replaced by an equal amount of white wax will overcome the lowered melting point. The higher percentages would result in a higher melting point than desired

Which of the following statements is true? A. Flocculation is desirable for pharmaceutical suspensions B. The diffusion rate of molecules of a smaller particle size is less than that of molecules of a larger particle size C. The particle size of molecular dispersions is larger than a coarse dispersion D. Psuedoplastic flow is shear-thickening type, and dilant is shear-thinnning type

A. Flocculation is the formation of light, fluffy conglomerates held together by weak van der Waals forces and is a reversible process. Pseudoplastic flow is a shear-thinning process (as dV/dr goes up, n goes down=less viscous), whereas dilatant is a shear-thickening process (as dV/dr goes up, n goes up=more viscous).

Which of the following combinations of drugs interact? A. fluconazole and tacrolimus B. fluconazole and valganciclovir C. dapsone and lasix D. valcyte and prednisone E. valganciclovir and EC ASA

A. Fluconazole is an inhibitor of cyp3a4, which is the enzyme system responsible for tacrolimus metabolism

Candida albicans infx of mild to moderate severity may be treated with A. fluconazole B. ampho B C. voriconazole D. caspofungin E. ketoconazole

A. Fluconazole is the preferred treatment for mild to moderate infx caused by Candida albicans. Ampho B, vori, and caspofungin are active against Candida but should be reserved for more severe infx or less susceptible candida species. Ketoconazole is not a preferred treatment for candida infections

The population average values for the clearance and volume of distribution of nifedipine have been reported as 0.41L/h/kg and 1.2L/kg, respectively. What would be the max dosing interval you can use for a multiple-dose regimen with an immediate-release oral dosage form of nifedipine if peak-to-trough fluctuation should not exceed 100% A. 2hr B. 4hr C. 6hr D. 8hr E. 12hr

A. For IR formulations, upper limits for peak concentrations (Css,max) and lower limits for trough concentrations (Css,min) can be estimated by assuming immediate drug absorption. If fluctuation is equal to 100%, Css,min is exactly one-half of Css,max. This is the case when the dosing interval tau is equal to the elimination half-life t1/2 of the drug. A population average half-life for nifedipine can be calculated as k=CL/V = (0.41L/h/kg)/(1.2L/kg)= 0.34/h t1/2= ln2/ k= ln2/ (0.34/h)= 2.03h

If the pka of a weak acid is 2.5, at which pH will this drug become more ionized? A. pH 5 B. pH 2.5 C. pH 1.5 D. pH 1

A. For a weak acid, a higher pH increases ionization

If the pka of a weak base drug is 8.3, at which of the following pH values does this drug become more ionized? A. pH 4 B. pH 9 C. pH 10 D. pH 13

A. For a weak base, a lower pH value increase ionization of the molecule

When one compounds an o/w emulsion containing a flavoring agent, the flavor should be in which emulsion phase? A. external B. internal C. dispersed D. discontinuous

A. For the flavoring agent to be tasted, it must be in the external or continuous phase. In an o/w emulsion, the oil is the internal, dispersed, or discontinuous phase

Which of the following medications should NOT be given because of the pt's sulfonamide allergy? A. furosemide B. fluconazole C. azathioprine D. valganciclovir E. tacrolimus

A. Furosemide is structurally similar to sulfonamides and would be expected to elicit a similar allergic response

A pt has htn, dm, ckd (scr 6.8) and has developed seizures. Which of the following anticonvulsants would require dosage adjustment in this pt? A. gabapentin and topiramate B. lamotrigine and felbamate C. phenobarb and gabapentin D. phenytoin and valproic acid E. phenobarb and levetiracetam

A. Gabapentin and topiramate would require dosage adjustments because they are renally eliminated

Diastat is given by which of the following routes? A. rectally B. IM C. IV D. intranasally E. SQ

A. Gel given rectally

Which of the following agents is NOT indicated in the setting of STEMI when pharmacologic reperfusion is the planned strategy? A. eptifibatide B. lmwh C. asa D. tPA E. metoprolol

A. Glycoprotein IIb/IIIa inhibition can be given in the setting of STEMI but is not preferred when a fibrinolytic agent is administered. Trials to date in combination w/ full and half dose fibrinolytic agents have shown a more complete reperfusion at the price of higher bleeding rates

Which of the following is an ade of pioglitazone? A. HF B. MI C. angioedema D. hypoglycemic unawareness E. afib

A. HF exacerbation (beware edema, bladder cancer, resumption of ovulation, hepatotoxicity, etc)

Methotrexate (Rheumatrex) is NOT available as which of the following dosage forms? A. IV inj B. oral tablet/ capsule C. intrathecal inj D. ointment

D. Not available for topical use

64yo wm, 5'11'', 185lbs PMH: MI, HTN, HF, HLD LVEF: 30, HR 88, BP 145/90 Meds: lanoxin 0.125 qd, lasix 40 qd, kcl 20 qd, zocor 40 qhs, ASA, plavix 75 qam Serum K is 2mEq/L (normal: 4-5) could A. increase the risk of lanoxin (digoxin) toxicity B. be treated by increasing the dose of lasix C. be considered a ADE of ASA D. be caused by an interaction btw zocor (simvastatin) and lanoxin E. increase the bleeding risk of plavis

A. Hypokalemia increase the risk of digoxin toxicity

An example of a radiolabeled peptide that is used in localized primary and metastatic neuroendocrine tumors is A. In-111 pentetreotide B. I-123 iobenguane C. In-111 capromab pendetide D. I-123 ioflupane

A. I-123 benguane is used in the detection of pheochromocytomas and neuroblastomas, but is not a peptide, In-111 capromab is a monoclonal antibody used in pts with prostate cancer. I-123 ioflupane is used in the eval of pts with parkinsonian syndromes

Which of the following radiopharmaceuticals is a controlled substance? A. I-123 ioflupane B. Florbetapir F-18 C. Thallous chloride T1-201 D. Tc-99m exametazime

A. I-123 ioflupane is the only one is a controlled drug

Which of the following is correct? A. for work done in a horizontal laminar flow workbench, arrange items in the hood so that your hand is never between the HEPA filter an the object B. For work done in a horizontal laminar flow workbench, vials that are not being used should be stacked up along the side of the hood to increase workspace in the hood C. before each shift, 70% isopropyl alcohol is used to sterilize the laminar flow workbench D. an object placed in the horizontal laminar flow workbench disturbs the airflow downstream of the object equal to 2 times the diameter of the object E. syringes and IV bags are placed in the hood in their protective overwrap

A. IN an HLFW, never put your hand behind an object, and in a VLFW, never put your hand above an object. In an HLFW, a vial distubs the laminar airflow equal to three times the diameter of the object. If the vial is next to the side wall, the airflow is disturbed equal to 6 times the diameter of the object. Syringes and IV bags should be taken from their overwrap at the edge of the hood.

Which of the following is an example of an rDNA-generated cytokine used for the management of acute relapsing-remitting MS. A. Interferon beta-1b (Betaseron) B. Aldesleukin (IL-2) (Proleukin) C. Eptifibatide (Integrilin) D. Bivalirudin (Angiomax) E. Abciximab (ReoPro)

A. INF b-1b is used in MS. Aldesleukin (IL-2) (Proleukin is a recombinant cytokine indicated for the treatment of metastatic renal cell carcinoma and melanoma. Eptifibatide (Integrilin) is a recombinant enzyme indicated for treatment of acute coronary syndromes. Bivalirudin is an enzyme indicated for use in coronary angioplasty and unstable angina. Abciximab is a monoclonal antibody indicated for prevention of blood clots post-PTCA and unstable angina to PTCA.

70yo bm PMH: DM, HTN, OSA, OA VI: 248lb, 5'11'' SH: smokes 1.5ppd, 4-5 cups coffee, 2drinks EtOH The appropriate initial antihypertensive agent in this pt could be A. benazepril B. terazosin C. minoxidil D. metoprolol E. clonidine

A. In pts w/ HTN and comorbid CHF and DM, initial agent should be ACEI

If the pt is not able to tolerate lisinopril b/c of ADE such as cough, an appropriate alt would be A. telmisartan B. labetalol C. guanabenz D. reserpine E. prazosin

A. In pts who cannot tolerate ACEI therapy secondary to ADE of cough, ARBs are good alt agents

Which of the following radiopharmaceuticals is indicated in cisternography studlies? A. In-111 DTPA B. Tc-99m sestamibi C. Tc-99m bicisate D. I-123 ioflupane

A. In-111 DTPA is used for cisternography studies, Tc-99m sestamibi does not have indication for the CNS. I-123 ioflupane is used in the assessment of pts suspected of having parkinsonian syndromes. Tc-99m bicisate is used in stroke

Which of the following statements describes the use of In-111 oxine? A. autologous WBCs are used B. it is ideal for leukopenic pts with a fever C. radiolabeling occurs in vivo D. it provides palliative therapeutic benefit

A. In-111 oxine is not useful for leukopenic pts, and radiolabeling occurs in vitro. It is a diagnostic agent rather than a therapeutic agent

Which of the following RCT designs evaluate intrasubject effects between two or more study treatments? A. crossover B. parallel C. sequential D. latin square

A. Intrasubject effects are analyzed in crossover study designs. Parallel, sequential, and Latin square designs analyze intersubject effects.

Which of the following antitubercular agents is the drug of choice for latent TB infection? A. isoniazid B. cycloserine C. ethambutol D. capreomycin

A. Isoniazid daily therapy is the preferred regimen for latent TB infx. Pts should be supplemented w/ pyridoxine and should avoid alcoholic beverages while taking isoniazid

Which of the following dx states or clinical conditions would usually require the administration of parenteral nutrition? A. severe acute pancreatitis B. mvc resulting in femur fracture and head injury C. 20% body surface area burn from a house fire D. laparoscopic cholecystectomy E. acute exacerbation of hepatic encephalopathy

A. It is difficult to feed pts w/ severe pancreatitis enterally unless there is access to the small bowel (ex: jejunostomy). The other clinical conditions, such as trauma and burns, would occur in pts in whom the gi tract could and should be used for nutrition support. A pt receiving laparoscopic cholecystectomy would not need nutrition support. Most pts w/ hepatic encephalopathy can be fed enterally if they require nutritional support.

Which of the following is true regarding Sm-153 lexidronam? A. localizes in areas of osteoblastic activity B. localizes in medullary thyroid cancer C. is a calcium analog D. is given orally

A. It localizes in areas of osteoblastic metastatic bone disease, and it is administered IV

Decreased ketoconazole absorption may occur if it is administered concomitantly with which of the following? A. antacids B. food C. warfarin D. cyclosporine E. cyp450 3a4 inhibitors

A. Ketoconazole is used in Cushing's dx by interacting w/ CYP enzymes. Antacids can decrease ketoconazole absorption b/c ketoconazole requires the presence of stomach acid to be absorbed. Any drug that decreases gastric acidity will decrease the extent of ketconazole absorption. Food increases ketoconazole absorption b/c it stimulates release of gastric acid.

Which of the following agents can be used to reverse respiratory effects caused by opioid overdose? A. naloxone B. pentazocine C. buprenorphine D. naltrexone E. tramadol

A. Naloxone is a u antagonist useful in opioid overdose. naltrexone is also a u antagonist, but it is reversed for use in alcoholism and opioid addiction

A 63yo female w/ stage 5 CKD is recieving peritoneal dialysis. Her most recent laboratory analysis reveals the following: BUN 58, Scr 5.2, phosphorus 7.4, calcium 9, albumin 2.5, iPTH 542pg/mL In addition to dietary restriction of phosphorous, which of the following agents is best for initial management of this pt's hyperphosphatemia? A. lanthum carbonate B. calcium acetate C. aluminum hydroxide D. calcium carbonate

A. Lanthanum carbonate or sevelamer carbonate is preferred over a calcium containing binder for initial management b/c the pt has a corrected calcium of 10.2mg/dL (corrected ca = measured serum calcium + 0.8(4-measured albumin) and a calcium x phosphorous product of 75. This elevated product increases the risk of metastatic calcifications. She requires a phosphorus binding agent w/o calcium to minimized calcium absorbed in the GI tract. Aluminum is not preferred b/c of the risk of accumulation of ade

Which of the following can be used as an oral formulation to treat infections caused by MRSA? A. linezolid B. daptomycin C. vancomycin D. ceftaroline

A. Linezolid is the only gram-positive antibiotic agent (covers MRSA) listed with an oral formulation. This can be helpful when a pt is in the outpt setting or transitioning to outpt care

A pt with afib may require a decreased warfarin dosage when which of the following drugs is initiated? A. liothyronine B. propylthiouracil C. methimazole D. ACTH E. diphenhydramine

A. Liothyronine is a T3, a potent thyroid hormone. In states of hypothyroidism, metabolism is decreased. However, if thyroid hormone is supplemented, blood clotting factors will be metabolized more quickly, leading to decreased warfarin requirements.

MW is a 25yo pregnant female who is admitted to the MICU following several days of severe n/v. She is hypotensive, tachycardic, and confused, and urine output is very low. Her serum sodium is 128. WHich of the following should be given to treat her fluid and sodium abnormality? A. IV NS or LR soln B. IV D5W C. po water D. IV lasix E. desmopressin

A. MW is hyponatremic and her clinical s/sx indicated severe dehydration from GI losses of water and sodium. She requires rapid fluid resuscitation with a fluid that has an approximately physiologic amount of sodium (either NS 154mEq/L or LR 130mEq/L). This amount of sodium will increase her serum sodium into the normal range over time, and the osmotic effect will hold water in the extracellular compartment (the vasculature and interstitium) to help restore organ perfusion

Which of the following has been shown to induce or worsen depression? A. ocp B. amoxicillin C. thiamine D. methylphenidate E. phenelzine

A. Many medications can cause or worsen depression such as antihypertensives (esp a blockers), antiparkinson agents; hormonal agents; corticosteroids; cycloserine, and the anticancer agents vinblastine and vincristine

Which of these agents reduces the likelihood of congenital malformations in epileptic women receiving valproate? A. folic acid B. vitamin B12 C. gingko biloba D. iron E. selenium

A. Many of the anticonvulsants can cause folic acid deficiency. An association exists between folic acid deficiency and spina bifida; hence, all women with epilepsy who are of childbearing age should receive 1g supplemental folic acid

What is the maximum BUD that can be given to powder-filled capsules according to USP Chapter 795 guidelines? A. 6mo B. 3mo C. 30d D. 14d

A. Maximum 6mo BUD for a nonaqeous preparation if all conditions are met

Pt information is provided by which of the following drug information resources? A. Medline Plus B. Martindale: The complete drug reference C. red book D. orange book

A. MedlinePlus is a di reference containing di tailored to the pt

Which of the following drug information references provides specific info about medications' ADEs? A. Meyler's side effects of drugs B. King guide to parenteral agents C. red book D. orange book

A. Meyler's Side Effects of Drugs is a di resource that provides specific info about a medication's side effects

Which of the following classes of agents is best known for causing infusion-related reactions, such as fever and chills A. monoclonal antibodies B. tyrosine kinase inhibitors C. vinca alkaloids D. platinum alkylating agents

A. Monoclonal antibodies are commonly associated w/ infusion-related rexns. Pts should receive premedication, such as apap and benadryl to prevent this

Pt is on paclitaxel, carboplatin for metastatic NSCLC Labs: WBC 6500, ANC 3500, PLT 67, H/H 13/39 On the basis of the pt's laboratory values, which of the following adverse reactions appears to have occurred as a likely result of the chemo? A. thrombocytopenia B. anemia C. neutropenia D. leukocytopenia

A. Myelosuppression is a common ade to most chemo agents. Both paclitaxel and carboplatin can cause anemia, thrombocytopenia, and leukopenia. In this case, thrombocytopenia is the dominant hematologic toxicity

Which of the following is NOT an appropriate treatment for hypoglycemic episode with a blood glucose reading of 64mg/dL A. 1/2 cup diet soda B. 6-7 hard candies containing sugar C. 3 glucose tablets D. 4oz regular soda E. 4oz orange juice

A. NO DIET SODA!

Heart failure may be exacerbated by which of the following medications? A. naproxen B. glipizide C. metformin D. Crestor E. fenofibrate

A. NSAIDs can worsening HF by increasing renal sodium and water retention, and by attenuating the efficacy and enhancing the toxicity of ACEIs and diuretics.

Which of the following are true of NSAIDs? I. they inhibit production of prostaglandins II. they are directly toxic to gastroduodenal epithelium III. they require dose adjustments in renal insufficiency IV. they cause only gastric ulcers V. they allow healing of PUD during continued therapy A. I, II, and III only B. I and III only C. II and IV only D. V only E. all are correct

A. NSAIDs inhibit production of prostaglandins, are directly toxic to gastroduodenal epithelium, and require dose adjustments in renal insufficiency. D/C to allow healing of ulcer

Which of the following penicillins does not have to be dose adjusted for renal insufficiency, yet can cause interstitial nephritis? A. nafcillin B. amoxicillin C. ticarcillin-clavulanic acid D. pen G

A. Nafcillin is a member of the penicillinase resistant penicillin class that does not have to be dose adjusted for renal insufficiency. However, this class of medications can cause interstitial nephritis. The first medication in this class, methicillin, was removed for the market because of a high incidence of interstitial nephritis.

The primary advantage of cisatracurium over pancuronium and vecuronium is that A. elimination is not organ dependent B. it has a shorter duration of action C. it has a longer duration of action D. it is more effective E. it does not require monitoring

A. Nimbex is metabolized by nonspecific plasma esterases, whereas pan and vec have varying degrees of hepatic and renal elimination. Thus, SCCM guidelines recommend nimbex for use in pts w/ renal and hepatic dysfn

Which of the following medications may cause seizures in an adult pt w/ renal failure? A. meperidine B. phenobarb C. carbamazepine D. lamotrigine E. theophylline

A. Normeperidine, a metabolite of meperidine, can accumulate in pts w/ renal failure who receive normal doses and cause seizures. The other agents listed are not eliminated renally in adults

Which of the following anticonvulsants is available in a liquid, chewable tablet, and IV form? A. phenytoin only B. valproic acid and carbamazepine C. carbamazepine and phenytoin D. valproic acid only E. primidone and valproic acid

A. Only phenytoin is available as a liquid (125/5mL) as a chewable tablet, and is in an IV dosage form. Carbamazepine is on available IV and valproic acid is not available po chewable

All of the following are side effects of potassium replacement therapy except A. constipation (po) B. GI upset (po) C. cardiac arrhythmias (po) D. vein irritation (IV) E. poor taste (po liquid)

A. Oral k replacement therapy does not normally cause constipation. Cardiac arrhythmias are a concern if IV k is given faster than 10meq/h

The preferred route of opioid administration is A. oral B. iv C. sq D. rectal E. im

A. Oral medications should be used whenever possible b/c of convenience, flexibility, and steady serum levels

An elderly male pt comes to your pharmacy and is worried that he may have prostate cancer. He just had some lab studies done, and his healthcare provider told him that some level was abnormal, indicating potential prostate cancer. Which lab test might he be talking about? A. PSA B. cortisol C. ESR D. PRR

A. PSA is a lab test that is commonly done in men over age 40. It should be performed annually in men over age 50 to check for prostate cancer.

Which of the following is best described as the scientific discipline of using genomewide approaches to understand the inherited basis of the differences between individuals in their response to drugs? A. pharmacogenomics B. functional genomics C. comparative genomics D. pharmacodynamics E. molecular genetics

A. Pharmacogenomics

Which of the following is an adverse effect of oral amio? A. photosensitivity B. pulmonary embolus C. phlebitis D. hyperkalemia E. acute renal failure

A. Photosensitivity is a common ade of oral amio. Pts should be counseled to limit sun exposure and use sunscreen. Phlebitis would be expected to occur only during IV amio infusion, particular through the peripheral IV line

Which of the following AADs' MOA is primary the result of sodium ion transport blockade? A. propafenone B. ibutilide C. sotalol D. verapamil E. diltiazem

A. Propafenone blocks sodium entry into the cardiac cell, slowing depolarization. Ibutilide and sotalol act primarily by blocking potassium transport, whereas verapamil and diltiazem inhibit calcium channels.

RT is a 40yo male admitted to the MICU following a severe asthma exacerbation. RT's serum phosphorous is 0.9mEq/L, and his body weight is 70kg. Which of the following acute phosphorous supplementation regimens is most appropriate? A. 45mmol sodium phosphate over 6hr B. 45mmol sodium phosphate over 10min C. 15mmol po phosphorous over next 24hr D. 15mmol of IV sodium phosphate over 2hr E. No acute phosphorous therapy is required

A. RT is severely hypophosphatemic and requires high-dose IV therapy (0.64mmol/kg x 70kg = 44.8mmol). The dose should be infused with 7.5 mmol/h (total time 6hr) to avoid precipitation with calcium

Randomization is a key process w/in clinical trial to ensure which of the following? A. validity of inferential statistics B. avoidance of placebo effect C. avoidance of carryover effect D. normal distribution of study outcome

A. Randomization is a key process w/in clinical to ensure the validity of inferential statistics used in analyzing the study results. It does not avoid the placebo effect or the carryover effect or ensure a normal distribution of the study outcome

54yo male presents to ED with crushing substernal chest pain and radiation to left arm. PMH: htn, copd, gout. H/o smoking x 30y and occasional EtOH. BP 170/85, hr 72, rr 18, and temp 97. Before admission, took EC ASA 81, combivent, tiazac, and zyloprim Allergies: sulfa EKG: ST depression, Twave changes Troponins positive LVEF <35 Dx: NSTEMI, HF Which of hte following therapies should be avoided in this pt? A. reteplase B. clopidogrel C. enalapril D. atorvastatin E. UFH

A. Reteplase is a thrombolytic agent, which does NOT have a role in NSTEMI treatment. Only for STEMI when PCI not possible w/in 120 min. When PCI not available, use plavix or brilinta in NSTEMI.

A dose-limiting ade of sotalol is A. bradycardia B. polyneuropathy C. metallic taste D. agranulocytosis E. lupus-like syndrome

A. Sotalol possess significant BB activity along with K channel blocking and therefore may lead to ade of traditional BB

Cholesterol biosynthesis can be decreased by which of the following A. statins B. oat bran C. bile acid sequestrants/ resins D. zetia E. aspirin

A. Statins competitively inhibit HMG-CoA reductase, which is the enzyme responsible for converting HMG-CoA to mevalonate. Inhibition of mevalonate reduces cholesterol synthesis

Which of the following is NOT a characteristic of stratification? A. conducted after randomization B. ensures equal distribution of subject characteristics C. requires larger number of subjects D. can be used in parallel or crossover studies

A. Stratification ensures equal distribution of selected subject characteristics between groups. It requires a large number of pts and can be used in both parallel and crossover study designs. Stratification is performed before randomization.

Which of the following agents is a selective serotonin agonist? A. sumatriptan B. ketorolac C. dihydroergotamine D. metoclopramide E. caffeine

A. Sumatriptan. Triptans act as agonists for serotonin 5-HT1B and 5-HT1D receptors at blood vessels and nerve endings in the brain.

Which federal legislation enacted in November 2013 amended the Food, Drug, and Cosmetic Act to make a distinction between traditional pharmacy compounding regulated by a state board of pharmacy and a compounding outsourcing facility regulated by the FDA? A. Drug Quality and Security Act B. Food and Drug Administration Modernization Act C. Pharmacy Compounding Compliance Act D. American Medicinal Drug Use Clarification Act

A. The Drug Quality and Security Act was enacted in Nov 2013. The Pharmacy Compounding Compliance Act does not exist, and other federal acts were enacted before Nov 2013 B= Nov 1997 D= 1994

Which of the following statements is false? A. The Henderson-Hasselbalch equation describes the effect of physical parameters on the stability of pharmaceutical suspensions B. The passive diffusion rate of hydrophobic drugs across biological membranes is higher than that of hydrophilic compounds C. When the dispersed phase in an emulsion formulation is heavier than the dispersion medium, creaming can still occur D. Targeted drug delivery systems deliver the drug to the target or receptor site in a manner that provides maximum therapeutic activity

A. The Henderson-Hasselbalch equation describes the relationship between ionized and nonionized species of a weak electrolyte

The pH of a buffer system can be calculated with A. the Henderson-Hasselbalch equation B. the Noyes-Whitney equation C. the Michealis-Mention equation D. Yong's equation

A. The Henderson-Hasselbalch equation form a weak acid and its salt is represented as pH = pka + log ([salt]/[acid]), where pka is the negative log of the dissolution constant of a weak acid, as [salt]/[acid] is the ratio of the molar concentration of salt and acid used to prepare a buffer

The WHO analgesic hierarchy emphasizes A. concurrently using nonopioids, opioids, and adjuvant medications B. avoiding opioid use C. reserving opioid use only for severe pain D. using single agents rather than a combination of medications E. using nonopioids only for treatment of mild pain

A. The WHO analgesic hierarchy involves choosing among 3 stepped levels of treatment. Mild pain may respond to nonopioid drugs alone. Combining a low dose opioid with a nonopioid can relieve moderate pain. More severe pain requires the addition of a higher dose opioid preparation to the nonopioid. At any step, analgesic adjuvants may be useful.

Which of the following represents the best way to decrease potential toxicity with mtx while achieving optimal therapeutic benefit? A. add 1-3mg folic acid /d to pt's regimen B. decrease dose mtx to 25mg once monthly C. add monthly inj of leucovorin to the pt's regimen D. add leflunomide to the pt's regimen

A. The addition of folic acid to the MTX regimen has been demonstrated to reduce the risk of liver toxicity. Lowering the dose of MTX is likely to decrease risk but is also likely decrease its effectiveness. Leucovorin, an injective formulation of folate, is normally used to reverse MTX toxicity

Which of the following medications increases digoxin serum concentrations? A. biaxin (clarithromycin) B. hydralazine C. glyburide D. lipitor E. warfarin

A. The macrolide antibiotic is associated with a 50-100% increase in serum dig concentrations

Which of the following is correct concerning USP media transfers? A. an operator must successfully complete one media fill before compounding any CSPs B. an operator who passes a written exam may compound sterile preparations until the chief pharmacist finds time to watch his or her aseptic technique C. an operator who has successfully completed a media fill must re-qualify semiannually if he or she is preparing low-risk CSPs D. when an operator successfully completes one media fill for high-risk compounding, he or she needs to re-validate quarterly by completing one media fill E. fluid thioglycollate media are used for media transfers

A. The operator must successfully complete one media fill before compounding a sterile preparation. Once validated for low- or medium-risk compounding, the operator must re-validate annually. For high-risk compounding, the operator must re-validate semiannually. Passing only a written exam does not allow the operator to compound a CSP. Typticase soy broth is the medium most often used in media fills

A pt w/ a h/o dm2, anaphylaxis to asa, and htn is being discharged on a new prescription for Deltasone 40mg qd for 7d. He should be instructed to A. check feet daily for wounds B. take ibu for musculoskeletal pain C. take on an empty stomach D. take at bedtime E. wear identification for steroid therapy

A. The pt has a h/o fo diabetes and will be given prednisone which would be expected to increase blood glucose. When diabetes is poorly controlled, infections are more likely to occur. Steroids also cause immunosuppression. For this reason, he should monitor more closely for wounds that may become infected. He will not be taking prednisone long enough to develop adrenal insufficiency, so there is no need for him to wear identification for steroid therapy

Pt is on paclitaxel, carboplatin for metastatic NSCLC Which of the following regimens would be appropriate to prevent CINV? A. dexamethasone, granisetron, and aprepitant B. granisetron and prochlorperazine C. metoclopramide, dexamethasone, and aprepitant D. palonosetron and granisetron

A. The pt's regimen contains carboplatin and paclitaxel. Together these agents have a high likelihood of causing acute and delayed CINV. The pt should receive a corticosteroid, a 5HT3 antagonist, and a neurokinin-1 inhibitor. Therapy should not contain more than one class of agent

What is the ratio of lactose to sucrose when used as the base to compound tablet triturates? A. 4:1 B. 3:2 C. 1:4 D. 2:3

A. The ratio of lactose to sucrose is 4:1. Lactose is used as a filler, and sucrose is the binder that holds the tablets together

Positive: 30 (present), 40 (absent), 70 (total) Negative: 20 (present), 10 (absent), 30 (total) What is the sensitivity for the test? A. 30/50 B. 10/50 C. 30/70 D. 10/30

A. The sensitivity of a test is the probability of the test result being negative when an individual does not have the dx (total negative)/ (total absent)

Which of the following tests do NOT require endoscopy to test for H.pylori infection? I. serum antibody test II. fecal antigen test III. urea breath test IV. histology test V. rapid urease test A. I, II, and III only B. I and III only C. II and IV only D. V only E. all are correct

A. The serum antibody test, fecal antigen test, and urea breath test do not require endoscopy

When mixing rapid or short acting insulin with intermediate or long acting insulin which of the following insulins should be drawn up first? A. regular B. nph C. detemir D. degludec E. glargine

A. regular/ clear insulin is always drawn up first to ensure that all persons mixing insulins will use the same procedure and that no ir or long-acting insulin will be placed in the regular insulin vial

Which of the following is correct concerning the USP sterility test? A. the validation test must be done on each CSP to determine if the article to be tested adversely affects the reliability of the test B. the growth promotion test does not require that the test organisms listed in the USP be used C. after the inoculation, the media must be incubated for 14d or fewer at the appropriate temperature D. no growth on the sterility test proves that the aseptically produced product is sterile E. Tyrpticase soy broth is incubated at 30-35%C, and fluid thioglycollate is incubated at 20-25%C

A. The validation must be completed one time for each CSP. The growth promotion organisms listed in the USP are used for the validation test and for the growth promotion test. Sterility tests proves proper aseptic technique and does not prove a certain product is sterile. Incubate for at least 14d

59yo aa male recently diagnosed w/ PUD. Tissue biopsy + for H. pylori. NKDA Which of the following is the ideal therapeutic regimen for H. pylori-related PUD in this case? A. PPI, clarithromycin, amoxicillin B. PPI, bismuth, metronidazole, tetracycline C. omeprazole, amoxicillin D. omeprazole, bismuth, clarithromycin, furazolidone E. omeprazole, sucralfate, clarithromycin, furazolidone

A. Triple drug therapy is recommended for H. pylori. Quadruple therapy w/ a bismuth-based regimen is less convenient but may be used first line in pts who are penicillin allergic. 2drug regimens are less effective and are not recommended. Furazolidone is unavailable in the US

Concerning treatment of osteoarthritis, which of the following statements is correct? A. tylenol is generally considered the initial drug of choice B. tylenol is considered safe effective, and it has minimal adverse effects, especially in doses greater than 4g/d C. NSAIDs may be helpful, and they have minimal side effects to consider D. hyaluronic acid derivatives have been approved by the FDA for the treatment of pain associated with OA, and are robustly effective

A. Tylenol is generally considered to be safe and effective, and it is considered the drug of choice. Hyaluronidase derivatives are marginally effective.

The use of antimotility agents in a pt w/ suspected C. diff infx is A. discouraged b/c of the potential to cause toxic megacolon B. encouraged b/c of increased cure rates C. discouraged b/c of increased reinfections D. encouraged b/c of decreased reinfections E. discouraged b/c of lack of efficacy

A. Use of antimotility agents in C. diff infx increases the risk of toxic megacolon

Pts taking eplerenone for HF should avoid taking A. NSAIDs B. ACEIs C. BBs D. Demadex E. calcium supplements

A. Use of eplerenone is associated with renal potassium retention. Concomitant use of NSAIDs significantly increases the risk of hyperkalemia

Which of the following agents would be most likely to produce myalgias when combined with a statin? A. fibrate B. asa C. ACEI D. levothyroxine E. colesevelam

A. Use of fibrates in combination with statins or niacin increases the risk of myalgias. Though hypothyroidism can also contribute to myalgias, treatment with levo would not have any effect.

When one is conducting a lit search in secondary lit database, which of the following are generally useful strategies for narrowing a search? A. using "limits" B. using the Boolean operator or C. using only keywords D. combining only 2 MeSH terms at a time

A. Using "limits" is a useful strategy for narrowing a search of secondary lit

CF is a 65yo male diagnosed with endocarditis. Blood cultures reveal a highly sensitive strain of Streptococcus. Which of the following is most appropriate if CF has an anaphylactoid penicillin allergy? A. vancomycin B. gentamicin C. ceftriaxone and gentamicin D. meropenem E. rifampin and gentamicin

A. Vancomycin is appropriate for penicillin allergic pts with endocarditis caused by Strept species. Other regimens for strep include penicillin or ceftriaxone (w/ or w/o gent), which has a potential for cross-linking reactivity in pts w/ penicillin allergies

Which of the following describes appropriate use of vasopressin in septic shock? A. It is an option if first-line vasopressor is failing B. its recommended as a first-line vasopressor C. it should be titrated similarly to a catecholamine vasopressor D. it should be used only if hydrocortisone is used E. it can be used to increase co more than BP

A. Vasopressin is recommended for use as a second vasopressor if a first-line agent is failing. It is not titrated; its use is not related to hydrocortisone use, and it improves only BP, not cardiac output

All of the following medications can cause bradycardia except A. terazosin B. verapamil C. diltiazem D. Ziac E. clonidine

A. Verapamil and dilt are nonDHP CCBs, Ziac (bisoprolol+hctz) is a BB, and clonidine is a centrally acting a2 agonist. They all have negative inotropic effects on the myocardium. Terazosin is a peripherally acting a1 blocker which does not cause bradycardia

Which of the following medications produces a significant pk interaction when administered with azathioprine A. allopurinol B. fluconazole C. sirolimus D. probenecid E. tacrolimus

A. XO is responsible for the elimination of the active metabolites of azathioprine. Concomitant use of allopurinol with azathioprine results in significantly increased azathioprine induced toxicity. Reduce dose aza by 65-75%

Which of the following agents can be used to treat breast and prostate? A. Zoladex B. Tamoxifen C. Celebrex D. Percocet 5/325

A. Zoladex (goserelin) is an LHRH agonist that can be used to treat both breast and prostate cancer. LHRH agonists are approved by the FDA for premenopausal women b/c they inhibit estrogen production from the ovaries

48yo wm PMH: DM2, epilepsy, htn, fungal toenail infx Meds: tolazamide, glimepiride, lisinopril, phenytoin ER, itraconazole, insulin R Which of the following lab tests is needed to monitor the drug regimen? A. hemoglobin A1c B. glimepiride serum concentration C. insulin receptor substrate-1 D. insulin concentration E. serum Cpeptide conc lvl

A. a1c; serum c-peptide may be diagnostic for the determination of functioning b-cells; however, it is not used to assess safety or efficacy of the medication regimen

Which of the following drugs take with alcohol is most likely to cause a disulfiram-like reaction? A. chlorpropamide B. acarbose C. nph insulin D. glucagon E. pioglitazone

A. chlorpropamide (a 1st gen sulfonylurea)

Which of the following antidiabetic agents does NOT require LFTs for monitoring? A. glargine B. miglitol C. rosiglitazone D. acarbose E. metformin

A. lantus does not require LFT monitoring

50yo man. PMH: htn, recent-onset dm2, and hld. FH nonsignificant. SH nonsignificant. NKDA. Occasionally takes APAP for h/a and no other OTC meds or herbal products. Current meds: hctz 25qd, lipitor 10qd. BP 144/90, pulse 70, 185lb, 5'9''. a1c: 7.3%. TC 250, HDL 40, TG 145 What medications changes, if any, are recommended for this pt? A. no changes should be made B. change atorvastatin to 40mg qd C. add niacin to atorvastatin 10mg qd D. d/c hctz E. add welchol to the treatment regimen

A. the new guidelines recommend a moderate dose statin in a dm pt btw the ages of 40 and 75 with no other contraindications unless the ASCVD risk is greater than 7.5%. Because the ASCVD risk is not known, a moderate dose statin is ok

Which of the following is a common ade of tradjenta when used as monotherapy? A. URTI B. weight gain C. hypoglycemia D. angioedema E. myopathy

A. tradjenta (linagliptin) DPP4; risk URTI. angioedema is rare but can occur

Which of the following medications requires bile for emulsification and absorption? A. azathioprine B. cyclosporine C. tacrolimus D. prednisone E. all of the above

B. Cyclosporine is highly lipophilic and requires bile for emulsification and absorption

Sulfur Benzoyl peroxide aa 10% PEG base qs ad 90g Base: PEG 3350 1 part, PEG 400 1.5 parts If benzoyl peroxide contains 24% water, what quantity should be weight for the preparation? A. 9g B. 12.2g C. 6.6g D. 11.34g

B. A 10% concentration requires 9g to be weighed (90g total weight x 0.1=9g). If benzoyl peroxide contained 26% water, it is 74% (100-26) benzoyl peroxide. If 74%/100%=9g/x, then x=12.2g

50yo man. PMH: htn, recent-onset dm2, and hld. FH nonsignificant. SH nonsignificant. NKDA. Occasionally takes APAP for h/a and no other OTC meds or herbal products. Current meds: hctz 25qd, lipitor 10qd. BP 144/90, pulse 70, 185lb, 5'9''. a1c: 7.3%. TC 250, HDL 40, TG 145 At what time point should you assess the effectiveness of therapy? A. 3weeks B. 4weeks C. 6mo D. only if ade are noted E. annually

B. A repeat FLP should be drawn after 4-12w to assess adherence to therapy. Atorvastatin 40 would be expected to produce a 50% reduction in LDL

Which of the following combinations represents optimal pharmacotherapy of pts w/ stage C HFrEF? A. furosemide, clonidine, hctz, and propranolol B. furosemide, lisinopril, and carvedilol C. carvedilol, verapamil, amlodipine, and nesiritide D. cardizem, hctz, digoxin, and sotalol E. dobutamine, amiodarone, furosemide, and nitro

B. ACEI, BB, and diuretics should be used routinely in HF

Which of the following statements regarding methadone PK is true? A. the half-life corresponds to analgesic efficacy B. it is highly plasma pr bound and widely distributed in tissue C. the clearance of methadone is rapid, resulting in frequent dosing D. methadone has low bioavailability from the GI tract and therefore is not useful when given orally E. methadone is metabolized by hepatic glucuronidation

B. About 90% of methadone is bound to plasma protein and is widely distributed in tissues. Methadone has a long terminal half-life, resulting in slow clearance. This half-life does not correspond to analgesic dosing. Its metabolized via cyp

Which of the following statements is false? A. The partition coefficient is the ratio of drug solubility in n-octanol to that in water B. Absorption of a weak electrolyte drug does not depend on the extent to which the drug exists in its unionized form at the absorption site C. The drug dissolution rate can be determined using the Noyes-Whitney equation D. Amorphous forms of drugs have faster dissolution rates than do crystalline forms

B. According to pH partition theory, absorption of a weak electrolyte drug depends on the extent to which the drug exists in its un-ionized form at the absorption site. However, pH partition theory often does not hold true because most weakly acidic drugs are well absorbed from the small intestine, possibly because of the large epithelial surface areas of the agents

The regimen of choice for post-traumatic seizure ppx is A. phenytoin indefinitely B. phenytoin x 7d C. carbamazepine x 7d D. bzd prn if seizures occur E. keppra x 7d

B. According to the Brain Trauma Foundation guidelines, phenytoin for 7x postinjury is the regimen of choice for post-traumatic seizure ppx in pts requiring such therapy. There are not enough high quality data to recommend keppra

In the gi tract, calcitriol A. promotes absorption of calcium and inhibits absorption of phosphorous B. promotes absorption of both calcium and phosphorous C. promotes decreased binding of calcium and phosphorous D. promotes increased elimination of calcium and phosphorous

B. Active vitamin D (calcitriol) promotes absorption of both calcium and phosphorus in the GI tract. This is one reason that therapy with calcitriol or vit D analog may need to be withheld if the calcium x phosphorus product is elevated.

Alteplase is an rDNA protein of which of the following type? A. Hormone B. Enzyme C. Clotting factor D. Chemokine E. Cytokine

B. Alteplase is an rDNA protein of the enzyme type

Which of following is true for UC? I. aminosalicylates are the drugs of choice for maintenance therapy II. oral corticosteroids are the drugs of choice for maintenance therapy III. azathioprine often allows reduction in corticosteroid dose IV. topical aminosalicylates are the drugs of choice for severe or fulminant disease V. ciprofloxacin is alternative first-line therapy for mild to moderate dx A. I, II, and III B. I and III C. II and IV D. V only E. all of the above

B. Aminosalicylates are the drugs of choice for maintenance therapy of UC, not corticosteroids. Azathioprine often allows a reduction in dose of corticosteroids in the management of active UC. Severe or fulminant UC requires oral, not topical therapy. Ciprofloxacin may be used as an alt first line therapy in the treatment of mild to moderate CD, not UC.

66yo wm presents with chest pain that is sharp, aching, and nonradiating x past few weeks. Occur mainly during daily walk and relieved at rest. EKG normal. Cath 2y ago. Troponins negative PMH: HTN, PUD, asthma, CAD FH: significant for CVA, DM2, and MI SH: smokes 1ppd x40years, EtOH socially Meds: Proventil, Flovent, Prilosec, asa, hctz Vitals: 148/92, HR 82, RR 18, 72 inches, 200lbs Which treatment algorithm should be considered given the current sx and presentation? A. UA B. stable angina C. variant angina D. silent ischemia E. NSTEMI

B. Angina is considered stable is sx have been occurring for several weeks w/o worsening and lasts <30min, and relieved at rest or w/ SL NTG.

Which of the following is an ADE of digoxin? A. hepatotoxicity B. anorexia C. stroke D. PE E. AKI

B. Anorexia is a common sx of digoxin toxicity.

Which of the following is NOT a descriptive statistic commonly used in characterizing data from observational studies? A. arithmetic mean B. normality C. mode D. range

B. Arithmetic mean, mode, and range are descriptive statistics. Normality is used in inferential statistics and is the exception

Which B-lactam antibiotics would be most appropriate to treat gram negative pathogens in a pt with a serious, anaphylactic penicillin allergy? A. oxacillin B. aztreonam C. cephalexin D. pen V

B. Aztreonam displays a purely gram-negative spectrum that tends to have the lowest allergic cross-reactivity in pts w/ anaphylactic penicillin allergies

Which of the following statement regarding the use of SSRIs in anxiety disorders is most accurate? A. SSRIs are not usually effective in the treatment of anxiety B. SSRI doses for anxiety should be started lower than initial doses for depression C. SSRIs should be used only after a failed trial w/ benzos D. SSRIs may be used on a prn basis when anxiety sx emerge E. SSRIs work very rapidly to resolve sx of anxiety

B. B/c SSRIs can be activating and may initially cause sx of anxiety, it is important to start low and go slow with these agents when used for anxiety disorders

A young female enters your pharmacy and informs you that she plans on becoming pregnant and would like you to review her medication profile to see if any of her medications would be potentially harmful. On reviewing her profile, you notice that she is taking Arava for RA. Which is the most appropriate response? A. arava is a cat C drug and could potentially harm the fetus. She should discuss the risks and benefits of becoming pregnant with her healthcare provider first B. avara is a cat X drug and she should undergo the drug elimination procedure w/ cholestyramine before trying to become pregnant C. avara is a cat X drug with no active metabolites and a short half-life; therefore, she should discontinue the drug and wait 1-2 weeks before trying to become pregnant D. avara is a cat B drug and the risk of toxicity to the fetus is extremely low

B. B/c arava is a teratogenic agent with an active metabolite w/ a long half life, a drug elimination procedure should be performed before becoming pregnant

A 66yo male with a pmh of CHF and htn is receiving lisinopril 10mg po qd, digoxin 0.25mg po qd, carvedilol 25mg bid, and spironolactone 25mg po qd at home. He now presents to the ED with a 1w h/o intermittent palpitations and dizziness. The ekg reveals afib with vr of 130bpm. The decision is made to attempt to restore nsr. Which of the following represents the best therapeutic approach to cardioverting the pt? A. perform tee, if no thrombus present, cardiovert, there is no need for anticoag B. perform tee, if no thrombus present, cardiovert, anticoag for at least 4w postcardioversion C. anticoagulate for 4w prior to cardioversion, d/c anticaog postcardioversion D. anticoagulate for 2w prior, continue for 4 weeks after E. DCCV immediate

B. B/c the pt appears to have been in afib x 1w, there is a signifiant risk of thromboembolism during the conversion. Proper treatment would require at least 3-4w of anticoag prior to cardioversion followed by at least 4w afterwards Alternatively, a TEE can be used to r/o atrial thrombus, allowing immediate cardioversion. B/c the atria will require time to recover to ns, anticoag will require at least 4weeks postcardioversion

A 66yo male with a pmh of CHF and htn is receiving lisinopril 10mg po qd, digoxin 0.25mg po qd, carvedilol 25mg bid, and spironolactone 25mg po qd at home. He now presents to the ED with a 1w h/o intermittent palpitations and dizziness. The ekg reveals afib with vr of 130bpm. The decision is made to attempt to restore nsr. After the initial successful cardioversion, the pt continues to have recurrent symptomatic afib. Chronic therapy to maintain nsr is to be initiated. Which of the following AADs would be the best choice to maintain nsr? A. flecainide B. amio C. sotalol D. ibutilide E. dronedarone

B. B/c the pt has HF, the results of the CAST indicate that class Ic agents should be avoided b/c of increased risk of death. Sotalol may worsen HF. Ibutilide is indicated for conversion only, not for maintenance of sinus rhythm. Dronedarone is ci in pts w/ hf b/c of an increase risk of mortality

Which of the following best outlines the central dogma of molecular biology? A. mRNA--> DNA --> protein B. DNA --> mRNA --> protein C. protein --> RNA --> DNA D. DNA --> protein --> mRNA E. protein --> DNA --> mRNA

B. DNA is translated into mRNA, which is translated ultimately to protein

Of the following, which medication regimen is the best choice as discharging therapy for a 65yo pt weighing 70kg who is post ACS w/ DES placement and preserved LVEF? A. asa 325, plavix 75, dilt 240mg, simvastatin 40 B. asa 81mg, lipitor 80, ticagrelor 90bid, lopressor 50 bid C. prasugrel 5, enalapril 10, toprol xl 100, simvastatin 40 D. asa 81mg, lopressor 100bid, prasugrel 10, SL NTG E. asa 325, morphine 2-4 IV PRN, o2, SL NTG

B. BB, DAPT, and statin should be given to all pts post MI. DAPT x 12mo after stent placement (both DES and BMI, can continue x 15mo if DES) associated with ACS

Which of the following medications is a monoclonal antibody? A. belatacept B. basiliximab C. horse antithymocyte globulin D. astagraf XL E. zortress

B. Basiliximab is a monoclonal antibody used in prevention of acute rejection

Which of the following would be a ci to using Enbrel. A. renal insufficiency B. active infection C. pt over the age of 65 D. pt w/ class I or II CHF

B. Because of its effects on tumor necrosis factor, Enbrel may decrease a pt's ability to fight infx. Enbrel is CI in pts with an active infx. Its use should be temporarily d/c'd until the acute process has resolved

Which of the following is best described as "the manipulation (as through genetic engineering) of living organisms or their components to produce useful usually commercial products (as pest resistant crops, new bacterial strains, or novel pharmaceuticals)"? A. biology B. biotechnology C. biotherapy D. bioinformatics E. nanotechnology

B. Biotechnology

Which of the following combinations of drugs represents therapeutic duplication: A. tacrolimus and azathioprine B. azathioprine and mycophenolate mofetil C. dapsone and valganciclovir D. glimepiride and tacrolimus E. prednisone and mycophenolate mofetil

B. Both azathioprine (Imuran) and mycophenolate mofetil (Cellcept) are classified as antiproliferative agents. Both agents inhibit purine biosynthesis and would not act synergistically

One of the most severe drug interactions is that between digoxin and quinidine. Administration of quinidine to pts taking digoxin results in 2-3 fold increase in digoxin Css and AUC after oral and intravenous admin of digoxin. Digoxin and quinidine are substrates for the multidrug resistance transporter, P-glycoprotein. According to the following pk data for digoxin, what is the most likely mechanism to explain this drug-drug interaction? CL: 125ml/min, Vss:1.2L/kg(IBW), fe>0.99, fup:0.25 A. quinidine reduces the digoxin fraction escaping 1st pass metabolism B. quinidine inhibits renal secretion of digoxin by blocking p-gp C. quinidine decreases digoxin fraction reabsorbed in the kidney tubule D. quinidine reduces the fraction of digoxin absorbed

B. CLrenal= CL(tau) x fe CLrenal = 125ml/min x 1 CLrenal = 125ml/min Eratio = CLrenal / fup x GFR Eratio = 125ml/min / 0.25 x 125ml/min Eratio = 4 Eratio > 1 filtration and net secretion

Which of the following is LEAST likely to contribute to a rise in blood glucose? A. D5W in NSS B. captopril C. epinephrine D. methylprednisolone E. anaphylaxis

B. Captopril increases insulin sensitivity and would not be expected to contribute to increased blood glucose. Epinephrine increases blood glucose by increasing glycogen breakdown, methylprednisolone promotes glycogenolysis, and anaphylaxis would be expected to increase stress response, thereby leading to increased epi release and blood glucose.

Which of the following ACEIs has the shortest duration of action? A. ramipril B. captopril C. lisinopril D. monopril E. fosinopril

B. Captopril must be given TID in pts with HF

Which of the following BB also blocks a1 receptors and is effective in treating HFrEF? A. metoprolol B. carvedilol C. bisoprolol D. propranolol E. atenolol

B. Carvedilol

Which of the following B-lactam antibiotics displays activity against MRSA? A. zosyn B. ceftaroline C. imipenem-cilastatin D. aztreonam

B. Ceftaroline is currently the only B-lactam antibiotic that displays activity against MRSA

45yo wf, h/o seizure dx has experienced heartburn after meals intermittently for the past 2 weeks. Worsens when reclining for bedtime. Meds: phenytoin 300 qhs. Sx not troublesome and wants to self-treat w/ OTC meds Which of the following should NOT be recommended? A. aluminum hydroxide B. cimetidine C. famotidine D. ranitidine E. magnesium hydroxide

B. Cimetidine is a potent inhibitor of cyp3a4 and will increase serum conc of phenytoin in this pt

Each of the following is recommended for monitoring pts requiring chronic amio therapy except A. ekg B. coagulation tests C. thyroid function tests D. LFTs E. cxr

B. Coagulation tests are not routinely recommended but may be required if a pt develops severe hepatotoxicity secondary to amio or simply requires concomitant warfarin therapy for afib

Cohort studies are the same as RCTs except for which of the following study design characteristics? A. evaluation of causal relationships between study variables B. randomization of study participants C. exposure to a test treatment or condition D. option for use of historical control groups

B. Cohort studies do not randomize study participants. Cohort studies do share commonality with rcts in that they evaluate causal relationships btw study variables, involve exposure to a test treatment or condition, and may occasionally use historical control groups

Which of the following drugs is used only for severe infx caused by MDR pathogens due to a significant risk of nephrotoxicity A. rifaximin B. colistin C. clindamycin D. fidaxomicin

B. Colistin is a polymyxin antibiotic that can be used for MDR gram negative pathogens such as P. aeruginosa and Acinetobacter spp. Its main limiting effect is significant nephrotoxicity, and it should be reserved for severe or MDR infx

The hallmark of empiric therapy is A. coverage of the most common pathogen associated with the infection B. coverage of the common pathogens associated with the infx C. coverage of all possible pathogens associated with the infx D. coverage of polymicrobial pathogens associated with the infx E. coverage of all viral organisms associated with the infx

B. Coverage of common pathogens associated with the infx increases the probability of curing the infx w/o increasing anti infective exposure to other organisms, which increase the possibility of resistance

Which of the following is NOT a critical step in practicing EBM? A. search the literature for relevant clinical articles B. regularly converse with colleagues on their approach to a clinical problem C. evaluate and critically appraise the clinical studies for validity and usefulness D. formulate a clear clinical question relative to a particular pt problem

B. Critical steps in practicing EBM include all of the following except B

Your pt has received a 5FU and irinotecan for the treatment of colorectal cancer. Before he leaves the clinic, you ensure that he has a prescription to prevent or treat which of the following ade of irinotecan? A. nausea w/ aloxi B. diarrhea w/ loperamide C. h/a w/ asa D. delayed allergic rxn w/ epi

B. Diarrhea is a dose-limiting toxicity of irinotecan. late-onset diarrhea can be life threatening. All pts should receive a prescription for loperamide to treat delayed-onset diarrhea. Pts should be instructed to take 2mg po q2h while awake and 4mg po q4hr during night until diarrhea has stopped for at least 12hr. Acute-onset diarrhea can be treated w/ atropine

Regarding the biologic DMARDs, which of the following statements is correct? A. kineret is unique in that it is not immunosuppressive B. pts should have a TB test complete before initiation C. FluMist is acceptable to use for influenza prevention D. pts receiving therapy are at increased risk for hepatitis

B. Do a PPD before initiation of biologic drugs b/c of their immune-suppressing properties, which could cause the reactivation of a dx such as TB

32yo wm presents with blood diarrhea (fewer than 4 stools per day) for 2 days. PE, CBC, and ESR are normal. Colonoscopy reveals distal colitis. The pt continues to have bloody diarrhea w/o systemic disturbances. Which one of the following is the best choice? A. add prednisone 40mg po qd and d/c enema B. add methylprednisolone 16mg IV q8h and continue enema C. add methylprednisolone 16mg IV q8h until remission achieved D. add mesalamine 2-4g po qd and d/c enema E. add azathioprine 1-2.5mg/kg/d and d/c enema

B. Do not d/c topical aminosalicylate. Oral aminosalicylates should be added if no response is achieved w/ topical aminosalicylates. Oral and IV steroids are reserved for mod to severe UC or for pts with systemic disturbances. Azathioprine may be added if UC is refractory to aminosalicylates and to allow corticosteroid dose reduction

What is the recommended dosage regimen for dofetilide in a pt with a calculated creatinine of 30mL per minute A. dofetilide therapy is not recommended B. 125mcg po BID C. 125mg po BID D. 500mcg po BID E. 500mg po BID

B. Dofetilide is renally eliminated and therefore must be adjusted according to creatinine clearance to decrease the significant risk of torsades

Which of the following is a drug of choice for Rickettsia infx such as rocky mountain spotted fever? A. clindamycin B. doxycycline C. azithromycin D. rifaximin

B. Doxycycline is a drug of choice for tick-borne illnesses such as Lyme dx and Rocky mountain spotted fever

What is the moa for grapefruit juice to inhibit the metabolism of some drugs that can result in drug toxicity? A. decreased renal excretion of drug B. inhibition of GI cyp3a4 C. decreased systemic clearance of drug D. inhibition of hepatic cyp3a4 E. expanded apparent volume of distribution

B. Drugs such as amlodipine, carbamazepine, and cyclosporine are profoundly metabolized in the gi tract before absorption. Grapefruit juice from frozen concentrate has been shown to inhibit gi cyp3a4 and thus allows more of the drug to be absorbed, thereby causing drug toxicity for drugs with a narrow ti

Electronic drug information resources, such as smartphones and tablets, provide which of the following advantages in terms of pt care? A. they increase potential errors associated with di B. they decrease potential errors associated with di C. they replace the role of the clinical pharmacist D. they decrease the interaction between the pt and the health care provider

B. Electronic di resources, such as smartphones, have been shown to decrease potential errors associated with di, thereby improving pt care

The best antibiotic selection for empiric treatment of peritonitis in a peritoneal dialysis pt is A. ancef + vanco B. ancef + ceftazidime C. vanco alone D. gent alone

B. Empiric therapy should include antibiotics with gram positive and gram negative coverage

Which of the following is correct? A. gram-negative bacteria must be alive to cause a pyrogenic response B. the lipopolysaccharide portion of the cell wall of gram-negative bacteria causes the pyrogenic response C. endotoxin can be removed by a 0.2 micron filter D. steam sterilization will depyrogenate an object just as well as the hot air oven E. an article that is depyrogenated is not necessarily sterile

B. Endotoxin will pass through a 0.2 micron filter. Steam sterilization will not depyrogenate an article (use dry). Bacteria do not have to be alive to be pyrogenic.

Which of the following refers to a plasmid designed to allow for the expression of an inserted gene within a host cell for the production of the specified protein? A. Cloning vector B. Expression vector C. Transcription factor D. Translation initiation factor E. Transposable genetic element

B. Expression vector. A cloning vector is used to carry a fragment of DNA into a cell for cloning. A transcription factor is a protein that regulates transcription in eukaryotic cells. A translation initiation factor is involved in the initiation of translation, A transposon is a portion of the DNA that can move from one part of the genome to another.

Which of the following is a myocardial radiopharmaceutical that provides information on both perfusion and left ventricular function? A. Rb-82 chloride B. Tc-99m tetrofosmin C. F-18 FDG D. Thallous chloride T1-201

B. F-18 FDG is an indicator of metabolism, Rb-82 chloride and thallous chloride provide information on myocardial perfusion. Tc-99m tetrofosmin provides information on both myocardial perfusion and left ventricular fn

Which of the following antifungal agents is the drug of choice for infections caused by candida albicans? A. terbinafine B. fluconazole C. flucytosine D. griseofulvin

B. Fluconazole is a drug of choice for C. albicans, yet it does not cover some non-Candida albicans species including C. glabrata or C. krusei. Although fluconazole can display dose-dependent susceptibility for some C. glabrata, it is inherently resistant to C. krusei

Which vitamin should be supplemented above standard amounts during nutrition support of a pregnant patient? A. cyanocobalamin B. folic acid C. biotin D. chromium E. pantothenic acid

B. Folic acid should be given at a dose of at least 600mcg/d during pregnancy. Many practitioners administer 1mg/d above what the pt is eating or receiving via nutrition support. This practice has been shown to prevent neural tube defects in the newborn

Fomivirsen (Vitravene) is an example of which of the following biological products? A. a liposomal formulation B. an antisense oligonucleotide C. an siRNA molecule D. an rDNA-produced protein E. a monoclonal antibody

B. Fomivirsen is the first product based on this technology to come to the market.

All of the following are steps in the critical appraisal of the literature relative to a published study except: A. evaluating the validity of the study results B. searching for other studies published by the authors C. determining applicability of the study D. evaluating the basic study design

B. General steps in the critical appraisal of the lit relative to a published study include evaluating the validity of the study results, determining applicability of the study, and evaluating the basic study design. Searching for other studies published by the authors is not generally a step in appraising literature and is the exception.

Which of the following oral antidiabetic agents is a micronized formulation? A. micronase B. glynase C. glucotrol xl D. amaryl E. orinase

B. Glynase is a micronized formulation of glyburide that is significantly absorbed

Which of the following statements is true regarding the use of opioids for migraines A. opioid use is NOT associated w/ rebound h/a B. butorphanol nasal spray is efficacious in migraine abortive therapy C. opioids in combination w/ butalbital and caffeine do NOT produce physical dependence D. opioids scheduled around the clock are useful in migraine ppx E. opioids are not commonly prescribed for migraine treatment

B. Good evidence exists for the efficacy of butorphanol nasal spray in migraine abortive therapy. Although opioids are commonly sed for abortive therapy, they may be associated with rebound h/a and physical dependence.

What type of cellulose is used to compound longer-acting capsules? A. methylcellulose B. HPMC C. hydroxyethylcellulose D. sodium carboxymethylcellulose

B. HPMC is the cellulose used to compound longer-acting capsules

66yo wm presents with chest pain that is sharp, aching, and nonradiating x past few weeks. Occur mainly during daily walk and relieved at rest. EKG normal. Cath 2y ago. Troponins negative PMH: HTN, PUD, asthma, CAD FH: significant for CVA, DM2, and MI SH: smokes 1ppd x40years, EtOH socially Meds: Proventil, Flovent, Prilosec, asa, hctz Vitals: 148/92, HR 82, RR 18, 72 inches, 200lbs What additional medication should be considered? A. ticagrelor B. atorvastatin C. clopidogrel D. prasugrel E. reteplase

B. He needs a statin. Ticagrelor and prasugrel are NOT indicated in stable angina. Plavix is only ok if not able to tolerate asa

Which of the following are the most common sx of typical esophageal GERD I. heartburn II. belching III. regurgitation IV. hypersalivation V. hoarseness A. I, II, and III B. I and III C. II and IV D. V only E. all of above

B. Heartburn and regurgitation are the most common sx of the typical reflux syndrome

The anticoagulant effect of ufh requires binding to which plasma cofactor? A. thrombospondin B. antithrombin C. plasminogen D. factor xiia E. factors ii, vii, ix, and x

B. Heparin's anticoag effect requires binding to AT (previously AT III) and that binding convers antithrombin from a slow, progressive thrombin inhibitor to a very rapid inhibitor of thrombin and factor xa.

What patient preparation is recommended for skeletal imaging if the patient's condition permits? A. bowel cleansing B. hydration C. thyroid blockade D. sincalide administration

B. Hydration is recommended.

KY is a 36yo female admitted for the 5th time in the past 3yrs for MDD. She presents with lack of appetite, avolition, anhedonia, and suicidal ideation with a plan. Vitals include BP 127/76, P82, RR 18, T 98.6. Labs TSH 4, FBG 135, -UDS, and BAL 0. Her current meds are fluoxetine 40mg qd, metformin, and hydroxyzine. KY states compliance and denies ade. Past medications include citalopram and venlafaxine. What is the most likely indication for the hydroxyzine in this pt? A. pruritus B. anxiety C. htn D. appetite enhancement E. dm

B. Hydroxyzine reduces anxiety and is often used in pts w/ substance abuse issues. Hydroxyzine is indicated for pruritus, but is likely being used for anxiety in this pt. Beware of the 2 different salt formulations. Hydroxyzine may be confused for hydralazine with is used in essential htn.

One of the most commonly reported ade w/ epoetin alfa and darbepoetin alfa is A. nausea B. htn C. constipation D. anaphylaxis

B. Hypertension

Which of the following statements is NOT true about data management? A. data analysis planning needs to start before data collection B. an investigator has to use large mainframe computers for data storage C. open-ended questions are more challenging because precoding for them is difficult D. spreadsheet programs can be used for data analysis for some projects

B. In modern times, most data collection, data entry, data storage, and data analysis can be conducted using microcomputers

Which type of immunity involves stimulation of cells and soluble mediators that non-specifically recognize alloantigens with no altered response on repeat exposure? A. autoimmunity B. innate immunity C. adaptive immunity D. acute rejection E. hyperacute rejection

B. Innate immunity is the fundamental type of immunity in which antigens are recognized in a nonspecific manner. This type of immunity is not augmented on repeat exposure

Which of the following scenarios can alter the pk of insulin products? A. rotating of injection sites around the abdomen B. rubbing the injection site C. needle size and length D. administration time of day E. administration just prior to meal

B. Insulin pk can be affected by temperature of site of admin, so activities such as rubbing

Which of the following describes Tc-99m sestamibi? A. it is an anionic complex B. it localizes in myocytes via passive diffusion C. it is transported into cells via glucose transporters D. it is a potassium analog

B. It is a monovalent cation but not a potassium analog. Glucose transporters are not involved in the localization of this agent

Which of the following correctly characterizes I-123 MIBG? A. has similar structure to somatostatin B. follows cellular pathway of norepi C. is therapeutic agent for treatment of thyroid dx D. interacts with drugs that affect glucose uptake

B. It is structurally similar to norepi and guanethidine and interacts with drugs that reduce norepi uptake. It's a diagnostic agent

Which of the following best describes the use of furosemide in HF? A. furosemide reduces mortality and slows HF progression B. hypokalemia is a common ADE C. response can be evaluated by monitoring hba1c D. oral absorption is increased in pts with ADHF E. furosemide bioavailability is not affected by food

B. Lasix can cause renal k loss and thus associated with hypokalemia. Its oral absorption is slowed in pts with aDHF and bioavailability is reduced by food

Which of the following statements about lyophilic colloidal dispersions is true? A. They tend to be more sensitive to the addition of electrolytes than do lyophobic systems B. They tend to be more viscous than lyophobic systems C. They can be precipitated by prolonged dialysis D. They separate rapidly

B. Most lyophilic colloids are organic molecules (including gelatin and acacia); they spontaneously form colloidal solutions and tend to be viscous. Dispersion of lyophilic colloids is stable in the presence of electrolytes

In the list that follows, the only organism that can be cultured easily is A. treponema pallidum B. mycobacterium tuberculosis C. rickettsia rickettsii D. ehrlichia phagocytophila E. francisella tularensis

B. Mycobacterium tuberculosis, although very slow growing, can be grown on culture media. The remaining organisms cannot be grown w/o the use of cell culture techniques, forcing the clinician to rely on serum testing and direct staining for identification of the organisms

Which of the following medications causes diarrhea and myelosuppression? A. sirolimus B. mycophenolate mofetil C. prednisone D. cyclosporine E. basiliximab

B. Mycophenolate mofetil can cause dose-limiting diarrhea and myelosuppression. Sirolimus (CD80/86-CD28) can cause myelosuppression but are not commonly associated w/ diarrhea

Which of the following is the leading cause of PUD in Hpylori negative patients? A. mineralocorticoids B. NSAIDs C. DMARDs D. antibiotics E. corticosteroids

B. NSAIDs

45yo female presents w/ aki. Has had difficulty over past 5d keeping down anything she eats or drinks. Decrease in urine over past 24hr. PMH: DM2, RA. Meds: asa 325, ibu 600mg qd, metformin, glyburide, apap Labs: Scr 2mg/dL, BUN 56 (normal 1 mo ago) Which of the following meds is most likely to have contributed to her aki? A. asa B. ibuprofen C. metformin D. glyburide

B. NSAIDs are associated with hemodynamic changes (prevent compensatory vasodilation of the afferent arteriole that occurs in conditions of prerenal aki). Metformin is not a cause of aki but would need to be d/c'd in this case d/t risk of lactic acidosis (scr >1.4 females, >1.5 males)

Which of the following supplements should be recommended daily in a pt with stage 5 ckd requiring chronic hd? A. multivitamin B. nephrocaps C. vitamin A D. nephrocaps + vit A

B. Nephrocaps include water soluble vitamins (vit B + vit C + folic acid) recommended for pt with kidney failure. Supplementation w/ fat-soluble vitamins is not recommended in pts w/ kidney failure d/t toxicities associated w/ accumulation

Nesiritide would be indicated in A. pts with asymptomatic LV dysfn B. pts with ADHF not responsive to IV diuretics C. pts with stage B HF D. pts with dm2 E. pts intolerant to digoxin

B. Nesiritide is only indicated for pts w/ severe or decompensated HF. Given IV

As dictated by the nomenclature for monoclonal antibodies, which of the following is a chimeric monoclonal antibody therapeutically used for inflammatory disease. A. Abciximab B. Infliximab C. Palivizumab D. Rituximab E. Trastuzumab

B. Nomenclature of monoclonal antibodies is highly structured. The first component of the name is product specific. The second component indicates its therapeutic use (ci for cardio, li for inflammation, tu for tumor). The third component indicates the type of monoclonal antibody (mo for murine, xi for chimeric, zu for humanized) and forth component mab represents monoclonal antibody.

Which of the following statements is true regarding observational research design characteristics? A. participants are blinded B. they can be retrospective or prospective C. randomization of participants is preferred D. they establish a causal relationship between variables of interest

B. Observational studies can be retrospective or prospective. They lack randomization, are unblinded, and are unable to establish a causal relationship between variables of interest

Once relevant primary literature is identified through secondary database sources, which of the following should occurs? A. the relevant primary lit identified is usually accurate and reliable; therefore, no further eval is necessary B. the pharmacist is responsible for evaluating primary lit to assess accuracy and reliability of info C. recommendations should be made on the basis of the literature identified w/o first evaluating the information D. b/c all published lit has already been evaluated for accuracy and reliability before publication, the pharmacist does not need to conduct an eval of the literature

B. Once relevant primary lit is identified through secondary database sources, the pharmacist is responsible for evaluating primary lit to assess accuracy and reliability of info

45yo marathon runner c/o cp during morning run. FH significant for CV dx. PMH: angina, hld, htn. Meds: asa, crestor, nifedipine, clonidine. His ecg is consistent with acute ischemia. hr 52, bp 170/100 Which of the following interventions is the LEAST appropriate for this pt at this time? A. lovenox 1mg/kg bid sq B. metoprolol 50mg bid po C. SL NTG PRN and IV drip titrated to pain and BP D. continuation of asa E. morphine if NTG does not control the pain

B. One of the CI to Beta blockade is an HR < 55bpm

Which of the following is the treatment of choice for an initial, mild C. diff infection? A. metronidazole IV B. metronidazole po C. vanco IV D. vanco po E. metronidazole IV and vanco po

B. Oral metronidazole is the treatment of choice for an initial, mild-to-mod episode of c.diff. An initial severe episode should be treated with vancomycin po, and an initial episode that is severe and complicated should be treated with vanco po and flagyl IV. IV vanco is inappropriate for C.diff infx

The clearance of which of the following opioids may be increased in pts with multiple copies of cyp 2d6 gene? A. methadone B. oxycodone C. fentanyl D. morphine E. hydromorphone

B. Oxycodone is metabolized through CYP 2D6 to active metabolites. Fast metabolizers = those with multiple copies of 2d6 gene = would clear oxy and its metabolites quickly

Adding PEG groups to large molecules will enhance their A. absorption through the intestine B. solubility C. oxidation D. degradation

B. PEG groups increase the solubility of the molecule

71yo wf, 5'4'', 150lbs PMH: HF (recent gain 20lbs over past month), HTN, OA LVEF: 25, BP 130/85, HR 80, Digoxin: 0.8ng/mL Meds: lanoxin 0.125 qd, lasix 80 qd, zestril 20 qd, toprol 50 qd, naproxen 500 bid cc The recent worsening of HF is most likely d/t A. zestril (lisinopril) B. naproxen C. subtherapeutic digoxin lvls D. furosemide E. drug interaction btw zestril and furosemide

B. The addition of the NSAID naproxen approximately 3-4w before admission likely is the cause of the ADHS episode. NSAIDs can increase sodium and water retention and negate the effects of diuretics and ACEIs

Which of the following is correct regarding stress ulcer ppx? A. H2RAs or sucralfate are equally effective and considered drugs of choice B. PPIs and H2RAs are generally considered the drugs of choice C. Antacids have the most direct effect on gastric pH and are considered drugs of choice D. Sucralfate is more effective than H2RAs and PPIs E. all agents are equally effective

B. PPIs and H2RAs are considered by most clinicians to be drugs of choice. PPIs have pros and cons; they may be more effective but may have other disadvantages based on outpt use. Sucralfate is a secondary option for a number of reasons described in the text, and antacids are not used for this indication.

Which of the following is the best choice for the initial treatment of troublesome sx of typical reflux syndrome? A. nizatidine 75mg qd B. pantoprazole 40mg qd C. metoclopramide 10mg qid D. 3mo trial of lifestyle modifications E. pantoprazole 40mg qd with metoclopramide 10mg qid

B. Pantoprazole. When a pt finds their sx troublesome, pharm should be initiated. Nizatidine is dosed incorrectly (150mg bid not 75mg qd). Metoclopramide is not routinely recommended for the treatment of typical reflux syndrome

A pt on which of the following medications should be made aware of the importance of good hygiene? A. felbamate B. phenytoin C. zonisamide D. phenobarb E. levetiracetam

B. Phenytoin may cause gingival hyperplasia (overgrowth of gums) and pts should be instructed to brush and floss daily and to have regular visits to dentist

A 63yo female w/ stage 5 CKD is receiving peritoneal dialysis. Her most recent laboratory analysis reveals the following: BUN 58, Scr 5.2, phosphorus 7.4, calcium 9, albumin 2.5, iPTH 542pg/mL This pt should be instructed to take her phosphate binder A. with meals to enhance systemic absorption of phosphorous B. with meals to minimize systemic absorption of phosphorous C. between meals to avoid food-drug interactions D. between meals to minimize GI side effects

B. Phosphate binders should be taken w/ meals to minimize systemic absorption of phosphorus from the GI tract

Which of the following antihypertensive agents is most likely to cause lupus syndrome, postural hypotension, and peripheral neuropathy? A. atenolol B. hydralazine C. guanfacine D. mibefradil E. nitroprusside

B. Possible ADE of direct vasodilators include postural hypotension and peripheral neuropathy. lupus is unique to hydralazine and does not occur w/ minoxidil

Which of the following anticonvulsants is metabolized to phenobarb? A. ethosuximide B. primidone C. zonisamide D. levetiracetam E. carbamazepine

B. Primidone is an active anticonvulsant, but is also metabolized to phenobarb

45yo male PMH: migraines w/ aura, htn (controlled) Meds: sumatriptan 100, lisinopril 10 Sumatriptan fills #9: 1/1, 2/1, 3/1, 3/9, 3/14 Which medication is appropriate to give for migraine ppx? A. butorphanol B. propranolol C. dihydroergotamine D. apap E. hydrocodone

B. Propranolol has been shown to be effective for migraine ppx. This agent can reduce the numer and intensity of attacks in about 70% of pts. Butorphanol, apap, dhe, and hydrocodone are not approved for migraine ppx.

A 53yo male has a pmh of MI and htn. He presents to his physician complaining of short (about 10s in duration), intermittent palpitations during the past 2d; tests r/o an acute MI and an EKG shows a LVEF of 25%. The pt is sent home with a Holter monitor to identify any arrhythmias. The Holter monitor reveals episodes of pvs and nsvt. What is is the most appropriate intervention for this pt? A. no therapy is indicated B. place an ICD C. start propafenone D. start verapamil E. use dccv

B. Pts w/ HFrEF and previous MI are at increased risk of sudden cardiac death, usually from ventricular fibrillation. Implantation of an ICD in these pts reduces the risk of mortality. Propafenone has negative inotropic effects and may worsen heart failure. The risk of ventricular proarrhythmia is significantly increased if propafenone is used in this pt. Verapamil also would worsen HF and not reduce the risk of mortality. DCCV is not indicated b/c pt is stable.

Which of the following is consistent with the diagnosis of osteoarthritis? A. it is normally associated with elevations in CRP and ESR B. a common initial finding of pain typically worsens with weight bearing activity and subsides with rest C. it commonly occurs in the wrists or the elbows D. crepitus is uncommon

B. RA usually exhibits elevations in ESR and CRP, unlike OA. RA subsides with motion (gelling phenomenon) and OA subsides with rest. OA typically affects weight bearing joints

JB is an 18yo wf diagnosed w/ gonorrhea What is an appropriate treatment for her uncomplicated gonorrhea treatment? A. ceftriaxone 125mg IM once B. ceftriaxone 250mg IM once C. ceftriaxone 1g IM once D. ceftriaxone 1g IM or IV q24hr until improvement, then cefixime 400mg orally twice daily to complete a total of at least 7d of treatment E. treatment is not necessary in pts w/ uncomplicated gonorrhea

B. Response A is no longer recommended because of decreasing cephalosporin susceptibilities and concerns about ceftriaxone treatment failure. Response C is the treatment for gonorrhea conjunctivitis. Response D is the treatment for disseminated gonococcal infx

RH is a 45yo male recently diagnosed w/ hep C. Genotyping reveals RH is infected w/ hep C, genotype 2. His health care provider decides to initiate treatment and asks you to select an appropriate treatment regimen. Which of the following could you recommend? A. peg-ifn alfa x 24w B. sofosbuvir and ribavirin x 12w C. peg-ifn alf and ribavirin x 48w D. boceprevir, peg-ifn alfa, and ribavirin x 12w followed by peg-ifn alfa and ribavirin alone x 12-36w E. telaprevir, peg-ifn alfa, and ribavirin x 12w followed by peg-ifn alfa and ribavirin alone x 12-36w

B. Response B is recommended treatment for hep C genotype 2. Responses A,D, and E are incorrect b/c current guidelines do not recommend using boceprevir or telaprevir based regimens or ifn monotherapy to treat any hep c genotype. Response C is incorrect b/c interferon w/ ribavirin is an alt therapy for hep c, genotypes 5 and 6, but not 2.

Which of the following BB has ISA (intrinsic sympathomimetic activity) activity? A. tenormin B. sectral C. inderal D. lopressor E. coreg

B. Sectral (acebutolol) and Visken (pindolol) have ISA

Cinacalcet is a calcimimetic that works by which of the following moa? A. decreases sensitivity of calcium sensing receptors on the parathyroid gland to calcium, which prevents secretion of PTH B. it increases the sensitivity of the calcium sensing receptors on the parathyroid to calcium, which prevents secretion of PTH C. it stimulates the breakdown of PTH and prevents the effects of PTH on bone turnover D. it increases calcium concentrations which suppresses secretion of PTH from the parathyroid gland

B. Sensipar works by binding w/ the calcium-sensing receptor on the parathyroid gland and increases the sensitivity to calcium, thereby suppressing PTH release

A common cause of diarrhea in pts receiving EN is from the A. osmotic load of the EN formulation B. sorbitol in drug vehicles C. addition of fiber to the EN formulation D. solute load from the protein component of the EN formulation E. improper placement of a nasogastric feeding tube

B. Several liquid preparations for drugs contain sorbitol as a pharmaceutical vehicle. These liquid preparations are commonly used in pts with tubes because the drugs can be given easily this way, especially if the pt cannot swallow. Most EN formulations are close to being isotonic (ie: osmotic load or solute load is not a major contributing factor). Fiber will prevent or improve diarrhea in most cases

Which of the following immunosuppressants should NOT be administered at the same time secondary to an interaction related to timing of doses? A. tacrolimus and azathioprine B. sirolimus and cyclosporine C. cyclosporine and azathioprine D. sirolimus and tacrolimus E. sirolimus and mycophenolate mofetil

B. Simultaneous administration of sirolimus and cyclosporine increases Cmax and the area under the curve of sirolimus by 120-500% and 140-230% respectively. Admin 4hrs apart increases Cmax and the area under the curve of sirolimus by 30-40% and 35-80%, respectively

35yo is a 5'8'', 52kg. PMH: crohn's dx, intermittent diarrhea Meds: prednisone, mesalamine 1g tid, loperamide albumin 2.5, prealbumin 13 triceps skinfold=3mm (10-14) calf skinfold = 4mm (10-15) The triceps skin fold measurement for this pt is an anthropometric measurement for assessment of A. somatic protein stores B. fat stores C. visceral protein stores D. immune competence E. body cell mass

B. Skinfold measurements measure body fat stores which assess the lipid component of the body. Visceral protein stores are serum proteins. Body cell mass and somatic protein stores assess skeletal muscle and visceral organs. Immune competence assessment requires a skin test with a common antigen

Chronic administration of winstrol may produce all of the following complications EXCEPT A. prostate enlargement B. increased testicular size C. gynecomastia in men D. accelerated atherosclerosis E. decreased menses in women

B. Stanozolol (Winstrol) is an androgen that would be expected to promote growth and development of male sex organs. However, chronic administration leads to feedback inhibition of testosterone secretion, which leads to testicular atrophy.

At least how much time should be allowed to lapse before a change in dose of epoetin alfa or darbepoetin alfa is made on the basis of a change in hemoglobin and hematocrit? A. 1w B. 2-4w C. 6-8w D. 2mo

B. Stimulation of erythropoiesis by epo and darbe occurs immediately; however, at least 2-4w will pass before substantial changes in hemoglobin and hematocrit are observed as a result of any change in dose of ESA therapy

Stomatitis is the clinical term for which of the following chemotherapy-related adverse effects? A. N/V or both B. inflammation of the mucosal lining of the mouth C. obstruction of the lower esophageal sphincter D. inflammation of the mucosal lining of the colon and rectum

B. Stomatitis is used to describe an irritation or ulceration of the mucosal lining. This side effect is common with fluorouracil and mtx. Having the pt hold ice chips in their mouth during treatment can prevent it. The cold is thought to cause vasoconstriction of the lining and prevent damage

Which group or groups of individuals is unaware of the subject treatment in a double blind study? A. subjects only B. subjects and investigators C. investigators and data analysis personnel D. data analysis personnel and subjects

B. Subjects and investigators are unaware of the subject treatment in a double-blind study. Single blind = pts, triple blind = all 3

During a routine clinic appointment, you conduct a medication review with your pt, a 35yo male w/ RA. He states he and his wife have been trying to conceive for 1yr with no results. After reviewing his medication list, which drug would you most suspect as contributing to this pt's infertility issues? A. mtx B. hydroxychloroquine C. leflunomide D. sulfasalazine

B. Sulfasalazine cause oligospermia

When 2 anti-infective therapies together produce a greater effect than the effects of each used alone, this phenomenon is termed A. commensalism B. synergy C. antagonism D. additive E. interacting

B. Synergy

WHich of the following medications requires therapeutic drug monitoring via trough concentrations? A. belatacept B. tacrolimus C. daclizumab D. basiliximab E. azathioprine

B. Tacrolimus required TDM via trough conc

Which of the following agents requires premedication regimen of dexamethasone, benadryl, and zantac to prevent an anaphylactic rxn? A. taxotere B. taxol C. paraplatin D. cisplatin

B. Taxol is the brand name of paclitaxel. This agent has been shown to cause hypersensitivity rxns in pts. It is unclear if these rxns are due to the drug itself or the drug's vehicle (Cremophor). All pts receiving paclitaxel should receive a premedication regimen of dexamethasone, diphenhydramine, and zantac

An example of a diagnostic skeletal agent is A. Tc-99m bicisate B. Tc-99m MDP C. Sr-82 chloride D. Florbetapir F-18

B. Tc-99m bicisate and florbetapir F-18 are for CNS imaging. Sr 82 chloride is a palliative therapeutic agent

The bacterial endotoxin test is used to determine A. the amount of pyrogens B. the level of pyrogens from Gram-negative bacteria C. the fever-producing potential of bacterial endotoxins from Gram-negative bacteria D. the level of bacterial endotoxin from Gram-postive bacteria E. the amount of live bacteria present in the drug solution

B. The BET determines the level of bacterial endotoxin from gram-negative bacteria only. The BET cannot determine fever-producing potential of endotoxins. The gram-negative bacteria do not have to be alive for the endotoxin to produce its effect.

When one is conducting a search for a drug information question, a systemic approach is recommended. Which of the following, in general, is the most appropriate order for searching different types of literature? A. search primary lit first, followed by a search of secondary references to identify tert literature B. search tertiary lit first, followed by secondary then primary C. only primary lit needs to be consulted for all types of di questions D. only tertiary lit needs to be consulted for all types of di questions

B. The amount of primary lit is vast. Search tert first, followed by secondary references to identify primary lit

Sulfur Benzoyl peroxide aa 10% PEG base qs ad 90g Base: PEG 3350 1 part, PEG 400 1.5 parts What quantity of PEG 3350 should be weighed for the preparation? A. 32.4g B. 27.5g C. 0.5g D. 28.2g

B. The base consists of 1 part PEG 3350 and 1.5 parts PEG 400 for a total of 2.5 parts. With 68.6g of base needed, 68.8/2.5 pats = 27.5g per part. Thus 27.5g is needed for 1 part of PEG 3350

Which of the following represent potentially dangerous interactions w/ Zyloprim? A. amoxicillin B. amoxicillin and imuran C. amoxicillin, imuran, and asa D. asa

B. The coadministration of amoxicillin and allopurinol increases the risk of rash up to 20%. Imuran is metabolized by XO, whose activity is inhibited by allopurinol, thus increasing risk of Imuran toxicity

What is the best recommendation for antihypertensive medication in a pt who has afib, CAD w/ angina, and hyperthyroidism? A. minoxidil B. betaxolol C. telmisartan D. nicardipine E. amiloride

B. The diagnosis of afib, cad w/ angina, and hyperthyroidism are all considered comorbid conditions w/ htn in which the use of bb may have favorable effects

50yo man. PMH: htn, recent-onset dm2, and hld. FH nonsignificant. SH nonsignificant. NKDA. Occasionally takes APAP for h/a and no other OTC meds or herbal products. Current meds: hctz 25qd, lipitor 10qd. BP 144/90, pulse 70, 185lb, 5'9''. a1c: 7.3%. TC 250, HDL 40, TG 145 The pt returns for reassessment at the appropriate time. His FLP shows that his LDL cholesterol is now 115mg/dL. What is your assessment? A. stop the statin because the pt has achieved the optimal LDL lvls B. the optimal dose reduction for this pt has been achieved C. the optimal dose reduction for this pt has not been achieved D. fenofibrate needs to be added to the regimen E. add cholestyramine

B. The dose of statin achieved a 36% (115/181) reduction in LDL. Expected reduction is 30-49%. Had that dose reduction not been achieved, a recommendation would have been to investigate possible nonadherence to therapy w/ pt

A 50kg pt with no h/o epilepsy presents in status epilepticus. The pt is given an adequate dose of lorazepam and is about to be given an LD of IV phenytoin. Assuming a Vd of 0.6L/kg, what dose of phenytoin should be given to achieve a serum phenytoin conc of ~16-18? A. 18mg/kg B. 500mg C. 30mg/kg D. 50mg E. 5g

B. The equation for calculation of a LD = Cp (serum conc) desired x Vd LD = 17 x (0.6L/kg x 50kg) = 510mg

Which of the following is true regarding a pt w/ refractory SE who is placed in a medically induced coma with a barb? A. if the pt is MV, the barb will induce respiratory arrest B. the goal of a coma that is medically induced w/ a barb is to induce burst suppression (isoelectric) on EEF C. if hypotension develops, the pt should be given nitroprusside D. the barbs are not associated w/ di E. a major problem w/ this type of therapy is renal failure

B. The goal is to produce a "flat" EEG. If the pt is mv, the effect of a medication on respiration is not a factor in its administration. Although pentobarb may cause hypotension if given too rapidly, nitroprusside is a vasodilator used to treat hypertension. The barbs are known inducers. A coma that is medically induced w/ a barb does not cause renal failure

Lidocaine will be given as a constant rate infusion for the treatment of ventricular arrhythmia. A plasma conc of 3mcg/mL was decided on as the therapeutic target conc. The conc of the infusion soln is 20mg/mL lidocaine. The avg volume of distribution of lidocaine is 90L; the elimination half life is 1.1hr. What infusion rate (in mL/min) has to be set on the infusion pump to achieve the desired target conc? A. 5mL/h B. 8.5mL/h C. 14mL/h D. 23.5mL/h E. 194mL/h

B. The infusion rate Ro or maintenance dose MD needed to achieve and maintain a ss conc of 3mcg/mL is given by MD=Ro=Css x CL = Css x V x ln2/t(1/2) MD=3mcg x 90L x 0.693/(1.1hr) MD=170mg/hr 170mg/(20mg/mL)=8.5mL

For a drug product in clinical drug development, an oral dosing regimen needs to be established for a phase III study that maintains an avg ss conc of 50ng/mL. In single-dose studies, an oral dose of 80mg resulted in an AUC of 962ng*h/mL. What dosing regimen should be used? A. 35mg q12h B. 50mg q12h C. 72mg q12h D. 95mg q12h E. 125mg q12h

B. The maintenance dose MD required to achieve an average ss conc of 50ng/mL for an oral dosing regimen given by MD=Css,av x CL/F The oral clearance CL/F can be determined from the relationship btw dose and AUC CL/F=D/AUC, therefore MD= Css,av x D/AUC=50ng/mLx80mg/962ng*h/mL MD= 50 x 80/ 962 = 4.16mg/h 4.16mg/h x 12hr = 50mg q12

A 30yo schizophrenic pt was receiving quetiapine 3mg BID for 12weeks with very little response. Which recommendation is the best for this pt? A. increase quetiapine dose to 600mg bid B. switch the pt to clozapine C. add risperidone 3mg bid to quetiapine D. add fluoxetine and ECT E. continue current therapy

B. The only antipsychotic proven effective for refractory schizo is clozapine. b/c the pt has had little to no response to quetiapine, raising her quetiapine dose is inappropriate. little evidence supports the use of multiple antipsychotics in the treatment of schizo. Augmenting quetiapine therapy w/ prozac and ECT is inappropriate b/c the pt has had little response to quetiapine. Guidelines for the treatment of schizo recommend two adequate trials of antipsychotic monotherapy before switching the pt to clozapine unless clozapine use is ci or inappropriate for the pt

WHich of the following is correct regarding the treatment of ADHF? A. nesiritide is the agent of choice in pts w/ ADHF and hypotension B. milrinone is preferred over dobutamine in pts receiving concomitant BB therapy C. absorption of oral loop diuretics is increased D. dobutamine and milrinone improve survival E. verapamil reduces volume overload and improves CO

B. The positive inotropic effect of milrinone is NOT mediated through the B receptor; therefore, its effects are not diminished by concomitant BB therap. Nesiritide is associated with an increased risk of hypotension. No inotropic agents are shown to improve survival. Verapamil is a negative inotrope and would worsen volume status.

A 33yo waitress is experiencing sx of anxiety, including night terrors and mood lability. She reports no past psychiatric history but admits that these sx started about 2w after armed and masked gunmen robbed her restaurant. She is most likely experiencing which of the following? A. LSD B. PTSD C. GAD D. OCD E. SAD

B. The pt experienced a violent and frightening episode 2w before her sx appeared. The sx of reexperiencing of the event, night terrors, and mood lability are common in pts with ptsd

A 45yo pt has a pmh of dm, htn, depression, and tobacco use. He had a h/o tia 3y ago, and his LDL is 140. The decision is made to start statin therapy. Which of the following doses is most appropriate? A. lipitor 10 B. lipitor 40 C. zocor 40 D. pravachol 80 E. lescol XL 80

B. The pt has a h/o TIA, therefore, he has existing ASCVD. He is less than 75yo, so a high intensity is preferred

Pt is on paclitaxel, carboplatin for metastatic NSCLC The goal of the treatment regimen is to A. cure his dx B. palliate his dx-related sx and increase QoL C. provide adjuvant therapy following definitive surgery D. increase his survival time by several years

B. The pt has metastatic dx, when most solid tumors are diagnosed as stage IV, this finding is representative of the fact that the dx is incurable. This is true of NSCLC. The treatment goals for these pts includes relieving any dx-related sx, minimizing toxicity from treatments, and increasing the pt's QoL through treatment or supportive care measures.

BT reports the nursing station with his head pulled sharply to the side and rear. he complains of severe pain in his neck and back area. The most appropriate diagnosis and treatment would be which of the following? A. akathisia: propranolol 20mg IM until resolution B. dystonic rxn: diphenhydramine 50mg IM q30min until resolution C. tardive dyskinesia: physical therapy D. dystonic reaction: lorazepam 2mg IM q30min until resolution E. none of the above

B. The pt is experiencing a dystonic rxn which can be treated w/ either benztropine 1-2mg IM or benadryl 25-50mg IM q 30min until resolution. The dystonic rxn is thought to occur b/c of an imbalance in dopamine and acetylcholine in the nigrostriatal region of the brain

Positive: 30 (present), 40 (absent), 70 (total) Negative: 20 (present), 10 (absent), 30 (total) What is the specificity for the test? A. 30/50 B. 10/50 C. 30/70 D. 10/30

B. The specificity of a test is the probability of the test result being negative when an individual does not have the disease. (negative absent)/ (total absent)

A 48yo black male arrives to the ED seeking his "nerve pill" He has a 27y h/o schizophrenia w/ multiple hospitalizations. In recent years, he has been effectively maintained on fluphenazine 10mg po bid. He now states that he feels "really bad today" His arms and jaws are stiff, his temperature is 101F, his BP is 176/110, and his WBC is 19000. LFTs and CPK are ordered stat. While you await the test results, you should do which of the following? A d/c oral fluphenazine, and begin quetiapine B. d/c oral fluphenazine C. continue oral fluphenazine, and add bromocriptine D. d/c oral fluphenazine, and give fluphenazine decanoate 25mg IM stat E. do nothing; labs need to be evaluated first

B. The sx that the pt is displaying appear to be the result of neuroleptic malignant syndrome. All neuroleptics have the propensity to cause this rare but deadly ade. The first steps in treating NMS are to d/c all offending agents, offer supportive therapy and prescribe a dopamine agonist and commonly a skeletal muscle relaxant

Which of the following product dosing regimens is correct? A. climara transdermal: apply to skin once daily B. vagifem: 1 tablet vaginally once daily x 2weeks, then 1 tablet vaginally twice weekly C. estring: insert ring intravaginally once daily at bedtime D. premarin tablets: 0.625-2.5mg tid E. premarin vaginal cream: 20-40g vaginally once daily

B. The vagifem dosage is 1 tablet vaginally once daily for 2weeks, then 1 tablet vaginally twice weekly. Climara (estradiol) transdermal is once weekly patch. Estring vaginal ring, 17b estradiol is replaced q3months. Premarin is once daily. Vaginal cream is 3weeks on, 1 week off.

Which of the following statements is NOT true? A. the coefficient of variation ranges from 0 to 1 B. the correlation coefficient ranges from 0 to 1 C. the sign of the correlation coefficient is the same as the slope of the regression equation D. the absolute value of the correlation coefficient equals the square root of the coefficient of determination

B. The value of the correlation coefficient ranges from -1 to 1.

48yo wm PMH: DM2, epilepsy, htn, fungal toenail infx Meds: tolazamide, glimepiride, lisinopril, phenytoin ER, itraconazole, insulin R What is the highest priority drug related problem? A. insulin therapy is not indicated for pts w/ DM2 B. therapy duplication of sulfonylurea therapy C. potential decrease of BG b/c interaction btw tolazamide and phenytoin D. potential increase of BG b/c interaction btw glimepiride and itraconazole E. use of an ACEI for htn in dm2

B. Tolazamide is first gen sulf; glimepiride is second gen. Potential interaction btw tolazamide would increase BG and glimepiride and itraconazole would decrease

A 33yo woman is being started on an anticonvulsant. She is already slightly overweight and is very concerned about the effects of the various medications on her weight. Which of the following is true regarding anticonvulsants and their effect on weight? A. valproic acid and phenytoin both decrease weight B. valproic acid increases weight and topiramate decreases weight c. topiramate and phenytoin both increase weight D. topiramate, valproic acid, and phenytoin cause no change in weight E. phenytoin is the only anticonvulsant known to increase weight

B. Topiramate can cause significant weight loss, and valproate can cause significant weight gain. Phenytoin does not significantly affect weight

Pts should be told to drink plenty of fluid when taking which of the following? A. carbamazepine B. topiramate C. levetiracetam D. gabapentin E. phenytoin

B. Topiramate d/t risk kidney stones. Same is true for zonisamide

The process whereby the ribosome in the cytoplasm reads mRNA codons and matches them with the appropriate tRNAs (which, in turn, carry amino acids responsible for protein synthesis) is referred to as A. transcription B. translation C. transformation D. transfection E. transduction

B. Translation Transcription= process by which RNA polymerase copies a strand of DNA into complementary RNA. Transformation= alteration of the heritable properties of eukaryotic cell Transfection= introduction of foreign DNA into a eukaryotic cell Transduction= transfer of DNA from one bacterium to another through a bacteriophage

Which of the following is correct about Treximet? I. treximet can be used concomitantly with OTC products such as aleve II. treximet dose is limited to 3caps/d III. treximet may cause dizziness A. I only B. III only C. I and III only D. II and III only E. all of the above

B. Treximet (sumatriptan/naproxen) is indicated for the acute treatment of migraine attacks with or without aura in adults. This medication may cause dizziness. The dose is limited to 2tabs/24hr.

59yo aa male recently diagnosed w/ PUD. Tissue biopsy + for H. pylori. NKDA If the pt was allergic to penicillin, which treatment recommendation would you choose? A. PPI, clarithromycin, amoxicillin B. PPI, bismuth, metronidazole, tetracycline C. omeprazole, metronidazole D. clarithromycin, metronidazole, tetracycline E. clarithromycin, metronidazole, furazolidone

B. Triple drug therapy is recommended for H. pylori; however, because the pt is allergic to amoxicillin (pcns) A is incorrect. Switching amoxicillin to flagyl is also appropriate. Quadruple therapy w/ a bismuth-based regimen is less convenient but may be used first line in pts who are penicillin allergic. 2drug regimens are less effective and are not recommended. Furazolidone is unavailable in the US. Antisecretory therapy is an integral part of Hy.pylori regimens to promote ulcer healing

Which of the following statements about triptans is correct? A. few CI exist to use of triptans B. triptans are CI in ischemic CVD C. triptans are preferred for migraine treatment during pregnancy D. pts taking ergot alkaloids can use triptans concomitantly E. triptans are strictly CI in pts w/ HTN

B. Triptans are CI in pregnancy and ischemic cardiovascular disease. they cannot be used w/in 24hr of another triptan or ergot alkaloid

AJ is a 35yo premenopausal women who is concerned about her fh of osteoporosis. She does not eat dairy products because she is lactose intolerant. Her recent bone mineral density screening revealed a T score of 1. Select the appropriate therapy recommendation from the choices below. A. daily estrogen replacement therapy B. daily calcium and vit D supplementation C. daily combined estrogen and progestin replacement therapy D. daily teriparatide injections E. daily calcitonin nasal spray

B. Tscore of 1 is not indicative of osteoporosis or osteopenia. Preventive care is appropriate. rx therapy not needed at this time

If a USP component is unavailable for a compounded preparation, which chemical grade would be the next best choice to use? A ACS B. FCC C. AR D. Tech

B. USP/NF or FCC grade components are preferred for compounding. Always obtain the certificate of analysis (COA) for every component used, and use components manufactured in FDA-registered facilities. Other component grades may be use if the COA is reviewed to determine that they are appropriate for use

70yo bm PMH: DM, HTN, OSA, OA VI: 248lb, 5'11'' SH: smokes 1.5ppd, 4-5 cups coffee, 2drinks EtOH Possible complications that the pt is at risk of developing secondary to uncontrolled htn include which of the following? A. hyperaldosteronism, UTI B. MI, blindness C. blindness, hyperaldosteronism D. UTI, blindness E. COPD, PNA

B. Uncontrolled HTN can cause multiple organ system problems including cardio (CHF, MI, PAD); opthalmologic (retinopathy, blindness); cerebro (TIA/ CVA), renovascular (nephropathy, CKD, dialysis) issues

Which of the following is true? A. febrile seizures must be accompanied by a CNS infx B. complex febrile seizures last >15min C. most children who have a febrile seizure go on to develop epilepsy D. the drug of choice for a simple febrile seizure is carbamazepine E. simple febrile seizures should never be treated

B. Unlike simple febrile seizures which last a brief period, complex febrile seizures are prolonged (>15min) or recur w/in 24hrs of the initial seizure. Febrile seizures must occur in the absence of CNS infx in a child w/ fever. Most febrile seizures are benign, and carbamazepine is ineffective in febrile seizures

RJ is a 55yo woman who presents to your pharmacy with a prescription for premarin 0.625mg daily. She has an intact uterus and has been recently diagnosed with menopause. Which of the following statements describes the appropriate action to be take by the pharmacist? A. refuse to fill the prescription and recommend a phytoestrogen supplement B. call the physician and confirm that the patient has an intact uterus and recommend a product containing estrogen plus progestin C. fill the prescription, and counsel the pt regarding administration instructions D. call the physician and confirm that the pt has an intact uterus, and suggest a transdermal estrogen product E. call the physician, recommend cancelling the Premarin prescription, and recommend starting a provera prescription

B. Unopposed estrogen is not recommended in women with an intact uterus because of an increased risk of endometrial hyperplasia and endometrial cancer. Women with an intact uterus should receive a product containing estrogen plus progestin

Which of the following represents a method to decrease the GI toxicity associated with NSAIDs? A. changing the pt from a COX2 to nonspecific NSAID B. adding a proton pump inhibitor such as prevacid C. adding glucosamine to the pt's NSAID D. instructing the pt to take the NSAID at night, when acid secretion is limited

B. Use a PPI. Timing the dose of NSAIDs has never been shown to change their toxicity profile. COX2 inhibitors are less likely to cause GI damage than nonspecific NSAIDs

Possible reasons for the pt's BP being uncontrolled include which of the following? A. Use of NSAIDs which cause decreased effectiveness of ACEI therapy, and drug interaction between glipizide and lisinopril B. Use of NSAIDs which cause decreased effectiveness of ACEI therapy and possible problems with adherence to antihypertensive regimen C. Lack of BP response to ACEI therapy, which should not be use in combo w/ diuretics in an African American pt, and possible problems w/ adherence to antihypertensive therapy D. Lack of BP response to ACEI therapy, which should not be used in combination with diuretics in an African American pt, and DI btw glipizide and lisinopril E. pts w/ OA tend to have HTN that is difficult to control

B. Use of NSAIDs which cause decreased effectiveness of ACEI therapy and possible problems with adherence to antihypertensive regimen

Empiric therapy for meningitis for pts up to 1mo of age includes A. vanco and ampicillin B. aminoglycoside and ampicillin C. ceftriaxone and vancomycin D. vanco and aminoglycosides E. ampicillin and ceftriaxone

B. the regimen covers the most likely organisms for meningitis in this age group: Stept agalactiae, E.coli, Listeria monocytogenes (ampicillin), and Klebsiella species. Ampicillin and cefotaxime would be another appropriate choice for empiric therapy in pts up to 1mo of age

A 24yo woman has complex partial seizures that are currently controlled with valproic acid, gabapentin, and topiramate. She calls your pharmacy to ask if any of her meds can cause nosebleeds b/c she has had one or two in the past week. You refer her to her pcp, where her platelet count is reported to be 95,132/mm2. Which of the following is true? A. none of her anticonvulsants cause thrombocytopenia B. valproate can cause a dose-related thrombocytopenia C. gabapentin has inhibited the metabolism of topiramate, and the elevated concentration of topiramate is responsible for the thrombocytopenia D. gabapentin can cause idiosyncratic thrombocytopenia E. topiramate can cause thrombocytopenia

B. Valproic acid can cause clinically significant thrombocytopenia. Gabapentin is not associated with any drug interactions that affects metabolism, and it does not cause a decrease in platelets. Topiramate does not cause thrombocytopenia

Which of the following conditions indicate the possibility of renal cl of a weakly acidic drug being sensitive to changes in urine pH? A. the drug is secreted and not reabsorbed B. the drug has a pka value of 5 C. the drug has a small volume of distribution D. all of the drug is excreted unchanged by the kidneys (ex: fe=1)

B. Weakly acidic drugs with pka values 3 and 7.5 can be highly unionized in the range of urine pH (5-8), and can thus undergo significant reabsorption if the unionized form is nonpolar. A: No pH sensitivity in renal clearance is expected unless the drug is reabsorbed Eratio <<1). C: Clearance and volume have nothing to do with a drug's likelihood of being affected by changes in urine pH. D. The fraction excreted unchanged says nothing about the mechanisms of renal elimination. However, it is important to note that for drugs with high fe values that are susceptible to changes in urine pH, large changes in the PK of the drug may be observed.

When searching for di on the internet, pharmacists should do which of the following? A. verify the info in only one source on the internet B. verify the info in multiple sources C. trust all authors of information on the internet, as along as they have a PharmD, MD, or PhD D. disregard the potential for errors or bias in the info

B. When searching for di on the internet, pharmacists should verify the info w/ multiple sources

If the pka of a drug is 5, at which of the following pH values do the ionic and nonionic forms of the drug exist at equal ratio? A. pH 1 B. pH 5 C. pH 7 D. pH 9

B. When the pH = pka, the ratio of ionized to un-ionized becomes the same

What alcohol is used to compound a non-sterile preparation when its type and percentage are not specified? A. ethyl alcohol 100% B. ethyl alcohol 95% C. ethyl alcohol 70% D. isopropyl alcohol 70%

B. When the type or percentage of alcohol is not specified in a compounded preparation, alcohol, USP, is used. It contains 95% ethyl alchol

Which of the following drugs is an oral prodrug of 5FU A. fluorouracil B. xeloda C. fludara D. cytoxan

B. Xeloda (capecitabine) is an oral prodrug of 5FU. Fludara is fludarabine and used in CLL and NHL IV. Cytoxan is cyclophosphamide and is available po and IV.

What is the generic name of Imuran? A. mycophenolate mofetil B. azathioprine C. cyclosporine D. tacrolimus E. prednisone

B. azathioprine

LB is a 45yo wf presenting to the EF with a fever of 103, flank pain, dysuria, urinary urgency, and frequency. Her lab tests are significant for an increased WBC counts of 18,000 cells/mm3 and 3% immature forms (bands). Her urinalysis received >105 cells/ mL of GNR. What therapy would be useful for LB? A. oral quinolone B. IV quinolone C. oral penicillin D. IV carbapenem E. IV vancomycin

B. given the severity of the dx, parenteral therapy would be reasonable for initial therapy. B/c the gram stain of the urine revealed GNR, either a quinolone or an extended spectrum beta-lactam would be reasonable empiric therapy until the organism is identified and sensitivities obtained. A carbapenem would be adequate but should be reserved for infx when MDR pathogens are suspected

Which of the following oral contraceptives is a biphasic product? A. ortho tri-cyclen B. ortho novum 10/11 C. ortho novum 1/35 D. yasmin E. amethyst

B. ortho-novum 10/11 is a biphasic oral contraceptive. Ortho tri-cyclen is triphasic. Yasmin and ortho novum 1/35 is monophasic low dose estrogen.

Inhibition of platelet aggregation by clopidogrel is due to the active metabolite formed by CYP2c19. Studies have shown that the PPI omeprazole, a moderate 2c19 inhibitor, reduces clopidogrel's antiplatelet activity when given concomitantly or if given 12 hours apart. Consequently, the package insert for Plavix warns against concomitant use of Plavix and strong or moderate 2c19 inhibitors Which of the following dose regimens would you recommend? A. the use of PPIs should be avoided in the pts taking plavix B. the ppi protonix, a weak 2c19 inhibitor, should be considered as an acid reducing agent in pts taking plavix C. concomitant use of 2c19 inducers would not be expected to affect the antiplatelet activity of plavix D. individuals who lack the ability to metabolize plavix because of a genetic polymorphism in 2c19 would experience increased antiplatelet activity when taking clopidogrel

B. the PPI protonix, a weak CYP2c19 inhibitor, should be considered as an acid-reducing agent in pts taking plavix. The package insert warns against use of strong or weak CYP2c19 inhibitors, but since protonix is a weak inhibitor, its ok

The drug transporter P-gp is involved in numerous processes in drug disposition. P-gp activity is directed responsible for the following processes: I. Glomerular filtration II. Transport of drug from hepatocytes into the bile III. Transport of drug from the small intestine the systemic circulation (ie blood stream) IV. Degradation of drug in the lumen of the duodenum V. Maintenance of the integrity of the blood-brain barrier by transport of drug out of the brain A. V only B. II and V only C. II and III only D. I, II, and V only E. all of the above

B. the drug transporter Pgp is directly responsible for the transport of drug from hepatocytes into the bile and maintenance of the integrity of the blood brain barrier by transport of drug out of the brain

The plenum in a laminar flow workbench is the area A. where the air is prefiltered B. where the air is pressurized for distribution over the HEPA filter C. where compounding tastes place D. that serves no purpose E. directly above the HEPA filter in a horizontal laminar flow hood

B. the plenium is the area behind the HEPA filter that allows air to be pressurized for even distribution over the filter

Which of the following would be the best choice for ventricular rate control in afib secondary to hyperthyroidism? A. adenosine B. digoxin C. apixaban D. propranolol E. atropine

D. BB are preferred rate-controlling agent for hyperthyroidism b/c they inhibit the adrenergic response and decrease thyroid hormone conversion

53yo African American female (65kg) w/ htn and hld is seen in the outpt nephrology clinic for eval of kidney dx progression. Current BP is 156/82, scr 2.6 (stable x past 4mo), BUN 44, urinary albumin excretion rate is 600mg/d. Her medications are enalapril 20mg/d x 1yr and simvastatin 20mg daily x 2years The recommended target BP for this pt is A. <11/70 B. <125/90 C. <130/80 D. <140/90

C. 130/80 b/c albumin excretion >300mg/d and CKD stage 4

Following major GI resection, a pt w/ severe short bowel syndrome is started on PN. It is anticipated that this pt may need this therapy for 6mo to 1year. The PN prescription for this pt includes D20W aa 5% at 105mL/h (2500mL/d) IV fat emulsion 20% at 10mL/h x 24h (240mL/d) 0.45% saline at 50ml/h x 24hr (1200mL/d) How many calories from IV lipid will this pt receive each day? A. 240 B. 360 C. 480 D. 600 E. 720

C. 20% = 20g/100mL 240mL x 20g/100mL = 48g 1g lipids = 10kcal 48g x 10kcal = 480

Which of the following is correct concerning the USP risk levels of compounded sterile preparations? A. preparations intended for administration over 3 days would be classified as low risk B. A high-risk sterile preparation that has met the requirements of the sterility test can be stored for not more than 24hours at a controlled room temperature C. the storage time for a medium-risk preparation under refrigeration is no longer than 9 days D. a CSP that will be administered to multiple patients or to a single patient multiple times is classified as a high-risk CSP E. after meeting the requirements of the sterility test, a sterile preparation can be stored indefinitely

C. A medium-risk CSP may not be stored longer than 9days at cold temperature. USP 797 does not address administration at all. ONe a CSP has met the requirements of the sterility test, the storage periods specified under the risk levels no longer apply. However, the BUD based on chemical stability always applies. A CSP that will be administered to multiple patients or to a single patient multiple times is a medium-risk CSP

Which of the following diuretics would be most appropriate for the initial treatment of a pt with aki and significant volume overload? A. metolazone B. spironolactone C. bumetanide D. mannitol

C. A pt w/ AKI generally requires aggressive diuresis (while avoiding dehydration). Bumex is a loop diuretic that is more potent than a thiazide-like diuretic (spironolactone) and would be a rational choice for initial therapy of aki. Spironolactone may cause hyperkalemia in a pt w/ AKI. Mannitol is an osmotic diuretic that may cause volume depletion and requires more aggressive monitoring and is not indicated

Which of the following is NOT considered a potential CV benefit of ACEI in IHD? A. ACEI reduce the incidence of MI B. ACEI reduce the incidence of cv death and stroke in pt at high risk for vascular dx C. ACEI have been proven to be superior and provide more antianginal effects than do BB D. ACEI should be used in all stable angina pts with known CAD who also have DM E. ACEI have shown greater benefit post-MI in higher risk pts

C. ACEI have been proven to be superior and provide more antianginal effects than do BB. BEWARE OF THE NOT!!!!! D;<

First degree AV heart block can be categorized as a disorder of A. automaticity B. reentry C. conduction D. increased ventricular excitation E. slowed sinus node firing

C. AV heartblock is caused by slowed conduction through the AV nodes

KY is a 36yo female admitted for the 5th time in the past 3yrs for MDD. She presents with lack of appetite, avolition, anhedonia, and suicidal ideation with a plan. Vitals include BP 127/76, P82, RR 18, T 98.6. Labs TSH 4, FBG 135, -UDS, and BAL 0. Her current meds are fluoxetine 40mg qd, metformin, and hydroxyzine. KY states compliance and denies ade. Past medications include citalopram and venlafaxine. The psychiatrist asks for your recommendation for this pt with refractory depression. What is the best option? A. d/c fluoxetine, initiate pristiq B. d/c fluoxetine, wait 2 weeks, start selegiline C. add ability to current regimen D. d/c fluoxetine, start lexapro E. add serax to current regimen

C. Abilify is approved for adjunct treatment for refractory depression. The pt has already failed a trial w/ effexor; therefore, initiating therapy w/ its isomer Pristiq (desvenlafaxine) may not be the best option. MAOIs (selegiline) are a viable fourth-line treatment option; however you would wait 5 weeks before initiating an MAOI after d/cing prozac. Lexapro is an isomer of citalopram which the pt already failed. BZDs (seraz = oxazepam) are not recommended for the treatment of depression

Analysis of the cerebrospinal fluid may give valuable clues to the identity of the pathogen in meningitis. Given the following results, what would be indicative of a bacterial infx? I. increased WBCs II. increased glucose III. increased protein A. I only B. II only C. I and III only D. II and III only E. all of the above

C. Bacterial meningitis infx show an increase in WBC and proteins in the CSF. Glucose is decreased

Which of the following is not a potential source of physical or chemical incompatibility? A. dilution of a drug in a cosolvent system into an aqueous solution B. addition of a drug solution with a high pH into a solution with a low pH C. adsorption of a lipid-soluble drug into the matrix of a polypropylene container D. a photosensitive drug such as sodium nitroprusside is D5W exposed to light E. leaching of phthalate plasticizer into the solution from a PVC container

C. Absoprtion of a lipid soluble drug into the matrix of PVC containers does occur. Polypropylene and polyethylene contain little or not plasticizer

Which cytokine released by activated CD-4 lymphocytes plays a major role in the subsequent activation of numerous lymphocyte lineages? A. IL-1 B. TNF-a C. IL-2 D. IFN-y E. complement

C. Active CD4 Tcells produce and release various lymphokines, particularly IL2 which is important for activation and proliferation of numerous lymphocyte lineages

Which of the following pt conditions would affect the decision to initiate therapy with nicotinic acid? A. concomitant therapy with a diuretic B. concomitant therapy with renally toxic medications C. active liver dx D. baseline elevation of scr E. uncontrolled htn

C. Active liver dx would be a ci to initiating therapy with nicotinic acid b/c hepatically metabolized. Uncontrolled dm not htn may cause concern b/c niacin can affect blood glucose

Treatment of torsades may include all of the following except A. d/c any drugs associated with prolonged QT B. isoproterenol infusion C. adenosine D. magnesium sulfate E. AV pacing

C. Adenosine is typically used only to terminate PSVT. Treatment should consist of IV mag sulfate and electric pacing. An isoproterenol infusion can be used while waiting for electrical pacing.

Which of the following medications has this warning: "for pts switching from ir niacin, therapy with this drug should be initiated with a low dose and then titrated to the desired therapeutic response" A. pravigard B. vytorin C. advicor D. atorvastatin E. ezetimibe

C. Advicor (niaspan + lovastatin) contains niaspan which is not dose equivalent to ir or modified-release niacin preparations

Which of following is NOT a characteristic of afib A. no discernable P waves B. ventricular rate of 100-130bpm C. regular qrs patten D. narrow qrs complex E. chaotic contractions

C. Afib represents chaotic atrial activity resulting in no identifiable P wave.

Which of the following medications is not associated with drug-induced hypertension A. prednisone B. indomethacin C. rosiglitazone D. cocaine E. cyclosporine

C. All agents listed are possible causes of drug induced HTN through multiple moa except rosi

Drug information is often retrieved from the internet by both parents and health care providers. Which of the following is the best advice to provide to a pt seeking drug information on the internet? A. it is not usually necessary to search more than one website when searching for di on the internet B. drug news provided by the media is an evidence based source of di C. regardless of the number or types of sources consulted for di, it is wise to verify the information with a health care provider D. all info provided by pharmaceutical manufacturers is unbiased

C. Although pts may be encouraged to seek info on appropriate di sources freely available on the internet, pts should verify the info identified on the internet with a healthcare provider to ensure appropriate use of info identified

Which of the following is the drug of choice for absence seizures in a child <2yo? A. phenytoin B. phenobarb C. ethosuximide D. valproic acid E. primidone

C. Although valproic acid is extremely effective and is frequently used as monotherapy for absence seizures, it should not be given if <2yo

Which of the following is an antibiotic that exhibits concentration dependent bacterial killing? A. pen G B. ceftriaxone C. gentamicin D. aztreonam

C. Aminoglycoside antibiotics exhibit conc-dependent bacterial killing while B-lactam antibiotics display time-dependent bacterial killing.

Which of the following electrolyte abnormalities are most commonly associated with amphotericin? I. Hypokalemia II. Hypomagnesemia III. Hypocalcemia A. I only B. III only C. I and II only D. II and III only E. all of the above

C. Ampho B causes renal wasting of k and mag

Agents that may be used in the enteric coating of tablets include A. hydroxypropyl methycellulose B. carboxymethylcellulose C. cellulose acetate phtalate D. none of the above

C. An enteric-coated tablet has a coating that remains intact in the stomach but dissolves in the intestine when the pH exceeds 6. Enteric-coating materials include cellulose acetate phthalate, polyvinyl acetate phthalate, and hydroxypropyl methylcellulose phthalate

Which of the following is correct concerning certification of a laminar flow workbench? A. the particles introduced into the plenum of the hood must be approximately 0.5 micron B. airflow from the HEPA filter must be 120ft/min C. a leak greater than 0.01% of the upstream smoke concentration through the filter is considered a serious leak D. a total particulate counter can be used to check the integrity of the HEPA filter E. if a HEPA filter leaks, it cannot be patched, it must be replaced

C. Any leak greater than 0.01% of the upstream smoke concentration is a serious leak. The smoke particles are 0.3 micron. The airflow from the HEPA filter should be 90ft/min, plus or minus 20%. A total particle counter is used to classify the environment, not to certify the integrity of the filter. The HEPA filter can be patched

Which of the following AEDs are NOT associated w/ di? A. carbamazepine and gabapentin B. carbamazepine and levetiracetam C. gabapentin and levetiracetam D. carbamazepine only E. phenytoin and carbamazepine

C. At this time, neither gabapentin nor keppra is associated with significant drug-drug interactions. The absorption of gabapentin may be reduced by concurrent admin of al or mag containing antacids; hence, antacids should be given 2hr before or after a dose of gabapentin. Carbamazepine is an inducer that is associated with numerous drug-drug interactions

All of the following ADE are manifestations of u opioid agonists except: A. constipation B. respiratory depression C. atrial flutter D. nausea E. miosis

C. Atrial flutter is not a documented ade of opioids; however, therapeutic doses of many opioids produce peripheral vasodilation, reducing peripheral resistance, and inhibition of the baroreceptor reflexes. Recently, methadone has been associated w/ torsades, an atypical rvt.

Ms. Tiny's mother (age 59) is worried that she will develop breast cancer like her daughter. Which of the following is appropriate advice for her? A. perform a monthly breast self-eval B. receive a clinical breast examination by a qualified healthcare provider q 5yr C. undergo an annual mammography and an annual clinical breast exam by a qualified healthcare provider D. undergo annual bilateral breast biopsies

C. Biopsies should never be performed as initial screening tests; however, if results from the mammography and clinical breast exam point ot dx, a biopsy is needed to make a diagnosis. The american cancer society still recommends an annual mammography and a clinical breast exam for a woman at normal risk for breast cancer <20 monthly self eval 21-39 eval by clinical professional >40 mammography

Blinding of subjects is an important process within clinical trials to minimize which of the following? A. confounding variables B. systematic error C. bias D. variability

C. Blinding of subjects is an important process within clinical trials to minimize bias. It does not minimize confound variables, systemic error, or variability

Boolean operators are often useful when conducting a search of secondary literature include all of the following except A. and B. or C. MeSH D. not

C. Boolean operators that are often useful when conducting a search of secondary lit include and, or, and not. MeSH terms help focus a search of secondary lit

The rationale of adding caffeine to a simple analgesic for migraine treatment is to I. decrease required dose of apap/asa II. cause cerebral arterial vasoconstriction III. increase gastric acidity and perfusion A. I only B. III only C. I and III only D. II and III only E. all of the above

C. Caffeine has analgesic and possibly anti-inflammatory properties. Therefore, reduced doses of apap and asa may be required. Caffeine may also increase gastric acidity and perfusion, enhancing the absorption of asa

Edgar W. (58kg, 20yo) is receiving 80mg of gent as an IV infusion over 30min q8h. Two plasma samples are obtained to monitor serum gent conc as follows: 30min after end of infusion + 30min before next dose. The serum gent conc at these times are 4.9 and 1.7mg/L, respectively. Assume ss. Develop a reasonable dosing regimen that will produce peak and trough conc of approx 8 and 1mg/L, respectively. A. 120mg q8h B. 160mg q8h C. 140mg q12h D. 180mg q12h E. 280mg q24h

C. Calculate K k=ln(1.7/4.9)/(6.5) = 0.163/h Calculate the Vd assuming steady state Css,max = Cmeasured/e^(-kt) Css,max= 4.9/ e^(-0.163x0.5h) =5.32mg/h Css,min=Cmeasured/e^(-kt) Css,min=1.7/ e^(-0.163x0.5h) = 1.57mg/h V=Ro/k x (1-e^(-kTinf))/(Css,max-Css,min x e^(-kTinf) V=80/0.5h/0.163 x (1-e^-0.163x0.5)/(5.3-1.57x1-e^-0.163x0.5) V=19.8L Calculate tau tau=ln(Css,max(desired)/Css,min(desired))/k + Tinf tau=ln(8/1)/0.163 + 0.5hr = 13.3h Recommended dosing interval = 12hr D=Css,max(desired)x K x V x Tinf x (1-e^-k(tau)/1-e^-kTinf) D=8x 0.163 x 19.8L x 0.5 x (1-e^-0.163(12)/(1-e^0.163(0.5) D=142mg Recommended dose: 140mg q12h

Which of the following is associated w/ autoinduction? A. phenobarb B. phenytoin C. carbamazepine D. gabapentin E. levetiracetam

C. Carbamazepine induces its own metabolism with peak effects seen about 21d after beginning the medication or following an increase in dosage. Phenobarb and phenytoin are inducers. Gabapentin and keppra are not cleared hepatically

Valtrex is a prodrug for the antiviral acyclovir. It is enzymatically converted to acyclovir in the intestine and the liver. Probenecid and cimetidine increase the mean acyclovir AUC by 48 and 27%. Neither cimetidine nor probenecid affects the absorption of valacyclovir. Assume a glomerular filtration rate of 125ml/min. The pk parameters for acyclovir after valacyclovir admin are Cl=255ml/min; fe=0.9; plasma pr binding=15%; according to the above information, which of the following statements is accurate regarding acyclovir pk after oral admin of valacyclovir A. tagamet and probencid increase the acyclovir fraction absorbed (fa) B. acyclovir clearance would be increased by increasing urine pH C. changes in acyclovir AUC after coadmin of probenecid and cimetidine likely result from inhibition of acyclovir renal secretion D. cimetidine and probenecid alter acyclovir pr binding

C. Changes in acyclovir AUC after coadministration of probenecid and cimetidine likely result from inhibition of acyclovir renal secretion. The predominant renal clearance mechanism can be estimated by determine the Eratio: CLr=CLt x fe= 255mL/min x 0.9 = 229.5mL/min Eratio=CLr/(fup x GFR) = 229.5/((1-0.15)x125mL/min) =2.2 = filtration and net secretion Eratio > 1 indicates that glomerular filtration and net secretion are probable renal clearance and mechanisms. Thus, alterations in pr binding or urine pH will have little effect on the clr and AUC of acyclovir

Which sulfonylurea has been associated with the greatest incidence of prolonged hypoglycemia in the elderly? A. tolazamide B. tolbutamide C. chlorpropamide D. glimepiride E. glipizide

C. Chlorpropamide has a t1/2 life of 35hrs and duration of action of 60hrs; therefore, it has been associated w/ prolonged hypoglycemia in the elderly.

The estimated creatinine clearance for a 47yo male pt with an ideal body weight of 176lb and a serum cr of 2.2mg/dl is A. 32ml/min B. 40ml/min C. 47ml/min D. 93ml/min

C. Clcr = (140-age)(IBW in kg)/ (72 x scr) x (0.85 if woman) IBW = 45+2.3(inches over 6) for women 50+ for men 1 kg = 2.2lb

An increased risk of peptic ulcer dx occurs when NSAIDs are combined with which of the following? A. ranitidine B. ferrous sulfate C. dexamethasone D. carbamazepine E. acetaminophen

C. Corticosteroids such as dexamethasone are known to increase the risk of peptic ulcers in combination with NSAIDS

Which of the following is true regarding sample size in RCTs? A. increased sample size reduces statistical power B. increased sample size is needed for a crossover design versus a parallel design C. decreased sample size is needed where the expected difference in outcome btw groups is large D. decreased sample size is needed where large variability exists in the study outcome being measured

C. Decreased sample size is needed where the expected difference in outcome between groups is large. Increased sample size increases statistical power and is needed where there is large variability in the study outcome being measured. Decreased sample size is needed for a crossover study design because pts are serving as their own control

The dose of which of the following medications should be reduced in pts with renal insufficiency? A. metoprolol B. carvedilol C. digoxin D. nitro E. dobutamine

C. Digoxin eliminated by kidneys

Drug-induced lupus usually manifests as A. severe nephrotic syndrome B. blood disorders including low red blood cell, white blood cell, and platelet counts C. arthritis D. severe neurologic dx

C. Drug-induced lupus often manifests as arthritis, not the other, more severe manifestations

Which of the following is correct regarding electroconvulsive therapy? A. for maximum effectiveness, seizures should last 5-10min B. ECT is CI in pts w/ COPD C. ECT is effective treatment for pregnant females with major depression D. memory loss lasting 1-2w is common E. ECT may cause amenorrhea in female pts

C. ECT is safe and effective therapy for pregnant females and pts with COPD. Relative ci include increased icp, recent mI, recent intracranial hemorrhage, and cerebral lesions. Memory loss is a common side effect, but it usually occurs only on the day of treatment and perhaps the following day. The induced seizure lasts 30-90seconds and is monitored by EEG. Additionally, ECT does not cause amenorrhea in female pts

Which of the following are goals of therapy for PUD? I. reduced episodes of diarrhea II. eliminate sx III. reduce risk of gastric cancer IV. heal ulceration V. avoid spreading H. pylori A. I, II, and III only B. I and III only C. II and IV only D. V only E. all of the above

C. Eliminate sx and heal ulcers

Which of the following is an appropriate regimen for a pt w/ early-onset HAP w/o risk factors for MDR pathogens? A. doxycycline B. azithromycin C. unasyn D. cipro and vanco E. cefepime, cipro, and vanco

C. Empiric therapy for early-onset HAP w/o risk factors for MDR resistant pathogens is as follows: ceftriaxone, a fluoroquinolone, unasyn, or ertapenem. Doxycycline or azithromycin is appropriate for outpatient treatment of CAP. Cefepime, cipro and vanco in combination are appropriate for late-onset HAP or pts w/ risk factors for MDR.

Compared to a manufactured product, a compounded preparation has which of the following characteristics? A. made according to current GMP B. subject to an approved NDA C. patient specific in conjunction with the prescriber D. has an NDC number

C. Extemporaneous pharmacy compounding requires the three-party team of physician, patient, and pharmacist in which the physician writes a prescription requiring the pharmacist to compound a preparation for a specific patient. All other characteristics are for pharmaceutical manufactured products

Which of the following can be an adjunct medication to statins and diet in a child with homozygous familial hypercholesterolemia? A. niacin B. fibrate C. ezetimibe D. mipomersen E. no adjunct available

C. Ezetimibe could be added to statins and diet in a pediatric pt. Niacin and fibrates are not recommended. Safety and efficacy of mipomersen in peds has NOT been established

Which of the following radiopharmaceuticals accumulates in viable myocardial tissue via the Na/K ATPase pump? A. F-18 FDG B. Tc-99m tertrofsmin C. Rb-82 chloride D. Tc-99m sestamibi

C. F-18 FDG is transported via glucose transporters, Tc-99m tetrofosmin and Tc-99m sestamibi enter the cell via passive diffusion

The pk parameters for codeine in healthy adults are as follows: Oral F: 50%, fe<0.01, Vss: 2.6L/kg, plasma pr binding 7% Codeine is well absorbed (fa=1, fg=0.8). You may assume that hepatic blood flow in a 70kg adult is 1350 ml/min. The hepatic cl of codein is A. 851ml/min B. 1350ml/min C. 500ml/min D. 675ml/min

C. F= fa x fg x F* F*= F/Fa x fg = 0.5/ 1 x 0.8 = 0.63 ER = 1-F* = 1-0.63 = 0.37 CLhepatic = Q x ER = 1350mL/min x 0.37 CLhepatic = 500mL/min

The anticoagulant effect of apixaban is mediated by A. inhibiting vit-k dependent clotting factors B. block platelet p2y12 receptor C. inhibiting clotting factor xa D. direct thrombin inhibition E. inhibiting VKORC1 enzyme

C. Factor Xa inhibitor

Which of the following is an acceptable reason for excluding a pt from an intent to treat analysis of study data? A. adverse events B. lack of response C. failure to meet inclusion criteria D. noncompliance with study medications

C. Failure to meet inclusion criteria

Which of the following is the most easily titratable opiate for use in the ICU? A. morphine B. hydromorphone C. fentanyl D. apap E. dexmedetomidine

C. Fentanyl is used as a continuous infusion and is more titratable than morphine or hydromorphone. APAP and precedex are not opiates

Which of the following biological agents is indicated for treatment of ovulatory failure? A. Ganirelix (Antagon) B. Glucagon (GlucaGen) C. Follitropin alfa (Gonal-F) D. Eptifibatide (Integrilin) E. Thyrotropin (Thyrogen)

C. Follitropin alfa. Ganirelix is a recombinant hormone indicated for the treatment of luteinizing hormone surge during fertility therapy. Glucagon is a recombinant hormone indicated for treatment of hypoglycemia. Eptifibatide is a recombinant enzyme indicated for the treatment of acute coronary syndromes. Thyrotropin is a recombinant hormone indicated for the treatment of thyroid cancer

Which type of water should be used when compounding a non-sterile preparation? A. tap water B. potable water C. purified water D. water for injection

C. For non-sterile compounding, the USP specifies that purified water be used. Purified water is prepared by the processes of distillation, reverse osmosis, or ion exchange. These methods remove heavy metals and other contaminants from the water. Water for injection is used in sterile compounding , and tap and potable water meet the requirements for drinking water but not for compounding

The use of liposomal technology has favorable affected the therapeutic index of which of the following drugs? A. Cyclosporine B. Itraconazole C. Amphotericin B D. Cisplatin E. Propofol

C. Formulation of this antifungal agent as a liposomal preparation (AmBisome) has significantly reduced the nephrotoxicity and other adverse effects associated with this drug

Which of the following most accurately describes a medication and its corresponding effect? A. dm is an absolute ci to the use of nicotinic acid B. asa is dosed tid to prevent flushing in niacin C. gemfibrozil may reduced TG by as much as 50% D. colesevelam has similar pt tolerability problems as cholestyramine E. ezetimibe frequently causes muscle toxicity

C. Gemfibrozil can reduce TG by 20-50%. DM is a relative contraindication to the use of niacin. ASA is dosed daily before the first nicotinic dose of the day. Colesevelam is a tablet and avoids most of the palatability problems of other resins. Ezetimibe does not cause muscle toxicity

Which of the following is NOT an effect of glucocorticoids? A. immunosuppression B. decreased prostaglandin synthesis C. inhibition of glycogenolysis D. decreased neutrophils at sites of infection E. inhibition of macrophages

C. Glucocorticoids have potent effects on glucose and carbohydrate metabolism. They promote glycogen breakdown rather than inhibiting it.

A health care provider inquires about the recommended monitoring parameters for pts started on Ridaura. Which of the following represents the most appropriate response? A. baseline ophthalmologic exam, CBC, and serum cr, followed, by a yearly CBC, scr, and ophthalmologic exam B. baseline LFTs, CBC, and albumin, followed by monthly LFTs C. baseline CBC, serum cr, and urinalysis for pr, followed by a CBC and urinalysis for protein every 1-2mo D. no recommended monitoring parameters at this time

C. Gold therapy is associated with glomerulonephritis, thrombocytopenia, and leukopenia; therefore, a baseline renal eval and periodic testing should occur during the entire course of therapy

All of the following groups can serve as a control in a RCT except: A. standard treatment B. placebo C. historical D. usual care

C. Historical controls cannot be used in an rct

Which of the following is an advantage of central vein PN over peripheral vein PN? A. allows easier catheter placement B. does not require a pump for administration C. allows for fluid restriction D. does not have to be filtered E. uses dilute formulations in most cases

C. Hyperosmolar nutrients (dextrose, amino acids) can be used to concentrate the PN formulation, but the PN would have to be administered via a central vein

50yo man. PMH: htn, recent-onset dm2, and hld. FH nonsignificant. SH nonsignificant. NKDA. Occasionally takes APAP for h/a and no other OTC meds or herbal products. Current meds: hctz 25qd, lipitor 10qd. BP 144/90, pulse 70, 185lb, 5'9''. a1c: 7.3%. TC 250, HDL 40, TG 145 Which of the following baseline labs would be optimal to assess prior to initiation of this pt's medication for dyslipidemia? A. CBC, SCr, LFTs B. SCr, LFTs C. SCr, LFTs, CPK, TSH D. CBC, LFTs E. SCr, LFTs, CPK

C. In addition to SCr (for renally adjusted statins), CPK, and LFTs, measuring TSH is optimal b/c hypothyroidism is a factor in statin-induced myalgias

Which of the following radiopharmaceuticals is indicated for radiolabeling leukocytes? A. Tc-99m DTPA B. F-18 FDG C. In-111 oxine D. I-123 iobenguane

C. In-111 oxine; Tc-99m DTPA is used in brain and renal studies, F-18 FDG is used as a metabolic indicator, and I-123 iobenguane is used in the detection of pheochromocytomas and neuroblastomas

A pt presents to your ambulatory clinic w/ a BP 210/125, PMH: DM2, CHF, CKD What are the treatment goals for the pt with HTN emergency? A. systolic pressure should be reduced to 120 w/in the first hour of treatment to reduce risk of further organ damage B. Diastolic pressure should be reduced to 80 w/in the first hour of treatment to reduce the risk of further end organ damage C. BP should be reduced to 160/100 w/in the first 2-6hr of therapy D. MAP should be reduced by 50% w/in the first min to hours of therapy E. BP should be reduced to no lower than 180/110 in the first hour, b/c excessive falls in BP may precipitate coronary ischemia

C. Initial goal of BP would be a drop of MAP of no more than 25% w/in min to hours and to 160/100 w/in 2-6hrs

25yo wm has been increasingly disruptive at home. He has been claiming that he is president of the US and has been staying awake all night planning bills for Congress. He has been argumentative and threatening with his friends and family. Though he appears extremely tired physically, he is constantly active. The psychiatrist has decided to initiate Eskalith CR 450mg bid in this pt. Which of the following lab tests will need to be performed for this pt before starting this drug therapy regimen? A. pregnancy test B. LFT C. SCr D. fasting glucose E. UDS

C. Initiation of lithium requires several baseline lab tests; pregnancy test, ECG, and BP to assess cardiovascular status, thyroid function tests to rule out euthyroid goiter or hypothyroidism, Scr/BUN (lithium is 100% renally eliminated), and CBC with differential to evaluate for leukocytosis. Electrolytes should also be evaluated (decreased sodium can increase lithium lvls). The pregnancy test should not be performed b/c the pt is male. LFTs must be performed prior to initiation of divalproex. Baseline fasting glucose must be performed when prescribing atypical antipsychotics

Margaret Q, (100kg, 26yo) presents to the ED with acute sx of asthma. She recently started smoking again and has been taking oral theophylline for several years. The immediate determination of her theophylline plasma conc results in a lvl of 4mg/L. Theophylline population pk parameters: CL 0.04L/h/kg; V 0.5 L/kg; Therapeutic range: 10-20mg/L What is the appropriate IV LD of aminophylline for Margaret to achieve a target concentration of 12mg/L A. 300mg B. 400mg C. 500mg D. 600mg E. 750mg

C. LD = Css x Vss = (Ctarget - Cpredose) x Vd LD = 12-4mg/L x (0.5L/kg x 100kg)= 400mg theophylline = 500mg aminophylline The LD can be determined on the basis of the target concentration to be achieved and the Vd. The predose lvl of 4mg/L needs to be subtracted from the target conc, b/c the LD has to account for only the conc difference.

Identify a baseline laboratory test that is required before initiating a statin A. serum k B. cbc C. lfts D. hs-CRP E. plt function tests

C. LFTs and CPK

A 7yo boy on VA for partial complex seizures w/ secondary generalization that are refractory to phenobarb, phenytoin, carbamazepine, and gabapentin continued to have seizures and was started on lamictal 2weeks ago. Today he presents w/ a diffuse maculopapular erythematous rash with lesions on his lips. Which of the following is correct? A. a rash associated with lamotrigine generally occurs w/in the first few days; hence, the rash is not associated with an anticonvulsant B. the pt should be given diphenhydramine, and lamotrigine should be continue C. lamotrigine should be d/c D. the rash is secondary to drug interaction btw gabapentin and carbamazepine E. all of the anticonvulsants are associated with a life-threatening rash. To prevent SE associated w/ abrupt d/c of the anticonvulsants, the medications should be slowly d/c'd

C. Lamotrigine has FDA boxed warning for severe rash. Because this pt has a diffuse rash and lesions on his lips, lamotrigine should be discontinued. Because the incidence of severe rash may be higher in children than in adults, current practice would be to d/c lamictal and not "treat through" the rash w/ benadryl. Gabapentin does not interact w/ lamictal; however, the combo of valproic acid and lamotrigine is associated with a higher incidence of rash. Although abrupt d/c of an anticonvulsant may induce SE< an anticonvulsant may be abruptly d/c'd in the face of a life threatening event

During nutritional assessment, the measurement of body cell mass includes A. bone B. interstitial fluid C. skeletal muscle D. intravascular fluid E. extracellular fluid solids

C. Lean body mass includes bone, skeletal muscle, visceral organs, and extracellular solids. Body cell mass includes only the lean, metabolically active tissue such as skeletal muscle and visceral organs (ex: liver)

Which of the following is the preferred narcotic to relieve chest pain after the use of SL NTG? A. meperidine B. oxycodone C. morphine D. hydromorphine E. fentanyl

C. Morphine has vasodilator properties, thereby decreasing both preload and afterload, which decreases o2 demand. In addition, morphine lowers hr by relieving pain and anxiety. If a true morphine allergy exists, meperidine may be used as an alternate agents but is not preferred over morphine.

All of the following opioids are metabolized through cyp enzymes except: A. hydrocodone B. oxycodone C. morphine D. methadone E. fentanyl

C. Morphine is metabolized via hepatic glucuronidation

Which of the following opioids has a toxic metabolite that can accumulate in renal failure? A. oxycodone B. fentanyl C. meperidine D. hydromorphone E. methadone

C. Normeridine, a metabolite of meperidine, can accumulate with chronic use, with renal impairment, and when the dose exceeds 600mg q24h

Which of the following references would be the best source to identify di regarding whether a medication is lactose free? A. AHFS Drug information B. Orange book C. Red book D. Martindale: The complete drug ference

C. Of the choices provided, the Red Book would be the best source to identify info regarding whether a medicine is lactose free or not

Which of the following are correct concerning ampuls? A. ampuls can be left in the hood and used for several days once opened B. ampuls are single-dose containers and can be left in the hood and used for several days once opened C. ampuls are single-dose containers and must have the neck wiped with a sterile alcohol pad before being broken D. none of the above

C. Once an ampul is opened, it must be used immediately

All of the following are known ADE of cyclosporine EXCEPT A. hirsutism B. nephrotoxicity C. oral ulceration D. gingival hyperplasia E. hyperlipidemia

C. Oral ulceration not common.

When counseling pts, pharmacists have the opportunity to do which of the following? A. pharmacists communicate verbal drug info only B. pharmacists provide written di only C. pharmacists provide pts with written and verbal di D. pharmacists do not use di skills when interacting with pts

C. Pharmacists have the opportunity to provide pts with written and verbal di during the counseling session

A pt who is receiving chronic phenytoin therapy is hospitalized for an elective surgical procedure. Admission labs note that the pt has a phenytoin conc of 8mcg/mL (therapeutic range=10-20mcg/mL) and an albumin conc of 3g/dL. Phenytoin: F=0.2-0.9, CL-variable, <1% excreted unchanged in the urine 88-93% bound to plasma proteins (primarily albumin). Given this information and the therapeutic range of phenytoin, you would recommend that the physician A. decrease dose phenytoin, b/c high-extraction drugs exhibit increased unbound conc with increases in fraction unbound in the plasma B. increase dose rate of phenytoin, b/c low-extraction drugs exhibit increased CL w/ increases in fraction unbound in the plasma C. not change the dose rate of phenytoin b/c low-extraction drugs do not exhibit changes in unbound conc w/ increases in fraction unbound in the plasma D. not change the dose rate of phenytoin b/c low-extraction drugs exhibit equal and offsetting changes in CL and F with increases in fraction unbound in plasma

C. Phenytoin has to be a low-extraction drug b/c its bioavailability is as high as 90%. The large range in F is due to variability in the absorption of the drug. You know it is not high extraction b/c if it were, you could never get an F=90%. Phenytoin is a low-extraction drug and restrictively cleared. Normal albumin ranges: 3.5-5mg/dL. Thus, the pt probably has increased fraction unbound in plasma d/t low albumin levels. Because phenytoin is a low-extraction drug, CL is dependent on Fup and CLint. Increased fup would lead to increased CL and decreased total plasma concentrations.

Which of the following antihypertensive agents can cause first-dose syncope, palpitations, peripheral edema, and priapism? A. hydralazine B. nitroprusside C. prazosin D. verapamil E. moexipril

C. Possible ADE of a1 blocks (prazosin) include first-dose syncope, palpitation, peripheral edema, and priapism

Which of the following sedative agents is least likely to cause respiratory depression? A. ativan B. propofol C. dexmedetomidine D. midazolam E. fentanyl

C. Precedex does not cause respiratory depression

Which of the following is NOT used to calculate a sample size? A. desired statistical power B. estimated effect size C. probability of the results affecting clinical practice D. variability and experimental error

C. Probability of the results affecting clinical practice

A progestin-only oral contraceptive would be preferable over a combo oral contraceptive in which of the following cases? A. a 24yo college student who is sexually active B. a 48yo perimenopausal female with irregular menstrual cycle C. a woman who is breastfeeding her infant D. a 30yo obese pt with dm2 E. a 52yo postmenopausal woman

C. Progestin-only contraceptives are preferred in breastfeeding women b/c they do not negatively affect milk supply. Perimenopausal females may take combined oral contraceptives to regulate their menstrual cycle

45yo male PMH: migraines w/ aura, htn (controlled) Meds: sumatriptan 100, lisinopril 10 Sumatriptan fills #9: 1/1, 2/1, 3/1, 3/9, 3/14 Which of the following is true regarding initiation of a ppx migraine therapy? A. High risk for rebound h/a caused by excessive use of sumatriptan B. Pt is limiting use of sumatriptan to 3d/w and not candidate for ppx treatment C. Pt is candidate for ppx d/t increasing rate of attacks D. Requires ppx b/c HTN is a CI to using abortive therapies E. Ppx treatment is CI in migraines w/ aura

C. Prophylactic therapy should be considered b/c his migraines occur more than twice monthly and the frequency of attacks is increasing

Which of the following is preferred as a first line sedative agent for ICP control in pts w/ TBI? A. pentobarbital B. lorazepam C. propofol D. vecuronium E. sedation is not recommended in pts w/ TBI

C. Propofol is the sedative of choice for pts who who require neuro assessment often. Pts with TBI may require multiple neuro checks daily, the short duration of action of propofol allows rapid awakening

Which of the following is the most appropriate treatment for bulimia? A. insight-oriented therapy B. fluoxetine 20mg qd C. fluoxetine 60mg qam + CBT D. olanzapine 10mg qhs E. olanzapine 20mg qhs + family therapy

C. Prozac is approved for treating bulimia; 60mg is the most common dose. A combination of psychotherapy and pharmacotherapy is preferred

Which of the following is true regarding anorexia? I. AN is characterized by recurrent and inappropriate compensatory behavior to prevent weight gain II. pts with AN generally appear to be of normal weight for age and height III. AN pts have an intense fear of gaining weight or becoming fat, although they are underweight A. I only B. II only C. I and III only D. II and III only E. all of the above

C. Pts w/ AN refuse to maintain body weight at or above a minimal, normal weight for age and height. Pts w/ AN may respond to antidepressants, but CBT is usually more effective. AN is most commonly seen in white, middle to upper class females

Which of the following is NOT a measure of central tendency? A. median B. mean C. range D. mode

C. Range is a measure of dispersion

Which of the following reversible ade is associated with rapid infusion time of vanco? A. nephrotoxicity B. ototoxicity C. red-man syndrome D. neurologic toxicity

C. Red-man syndrome is a histamine-related rxn that occurs when vancomycin is administered rapidly. It is important to note that this is not a true drug allergy. Red-man syndrome can be minimized by slowing the infusion time and administering antihistamines such as diphenhydramine prior to infusion

TH is a 33yo woman currently taking Nordette ocp. She presents to your pharmacy with a prescription for oral rifampin for 6 weeks to treat prosthetic valve endocarditis. Which of the following choices describes appropriate action taken by the pharmacist? A. call the pcp, request a change to amoxicillin to avoid a drug interaction between nordette and rifampin B. dispense the rifampin, and counsel TH on the appropriate administration and duration of therapy for the antibiotic C. dispense the rifampin, and counsel TH regarding the potential for rifampin to interfere with the efficacy of nordette. Instruct TH to use a backup method of contraception throughout her course of rifampin therapy D. refuse to refill the rifampin prescription, and counseldTH that should should never take antibiotics while on oral contraception E. dispense the rifampin, and stop the pts nordette prescription

C. Rifampin has a pharmacokinetic interaction with combined hormonal contraceptives. Women taking rifampin should be educated about the possibility of oral contraceptive failure and should be encouraged to use nonhormonal contraception throughout the course of therapy

All of the following are secondary causes of htn except A. renovascular dx B. pheochromocytoma C. SLE D. primary aldosteronism E. aortic coarctation

C. SLE

Insomnia, GI upset, and h/a are common ade of which of the following antidepressants? A. phenelzine B. amitriptyline C. fluoxetine D. trazodone E. both B and C

C. SSRIs commonly cause insomnia, GI upset, anxiety, h/a, and sexual dysfunction. Phenelzine, amitriptyline, and trazodone cause sedative effects

A drug is administered via continuous infusion at a rate of 60mg/h, resulting in a ss plasma conc of 5mcg/mL. If the plasma conc is intended to be doubled to 10mcg/mL, the infusion rate must be A. left the same B. increased by 30mg/h C. increased by 60mg/h D. increased by 120mg/h E. decreased by 30mg/h

C. Steady-state plasma concentration of a constant-rate infusion is directly proportional to the infusion rate Ro, through Css=Ro/CL Thus, Ro has to be doubled from 60mg/h to 120mg/h to increase Css from 5 to 10 mcg/mL. IE DOUBLE 60 means to ADD another 60mg/h

Ms. Tiny presents to your pharmacy with complaints of lower leg calf pain that is tender to the touch and red. You suspect a deep-vein thrombosis. Which of the following agents is mostly likely to be associated with this condition? A. desloratidine B. pseudoephedrine C. tamoxifen D. percocet 5/325

C. Tamoxifen is well-known to increase the incidence of thromboembolic events. All pts on this medication should be counseled on the signs and symptoms of thromboembolic events

Which of the following radiopharmaceuticals is used as an adjunct in determining the site of stroke in patients previously diagnosed with stroke? A. In-111 DTPA B. Tc-99m sestamibi C. Tc-99m bicisate D. I-123 ioflupane

C. Tc-99m bicisate; In-111 DTPA is used for cisternography studies, Tc-99m sestamibi does not have indication for the CNS. I-123 ioflupane is used in the assessment of pts suspected of having parkinsonian syndromes

Which of the following drug information resources contains the orange book? A. AHFS drug information B. ASHP website C. FDA website D. CDC website

C. The FDA website contains the electronic version of the Orange Book

Which of the following is an important determinant of the anticoagulant response to warfarin A. CYP2C19 B. CYP2D6 C. VKORC1 D. Pgp E. creatinine clearance

C. The VKORC1 gene determines the activity of the vit k epoxide-reductase enzyme, which is the target for warfarin. Warfarin is not affected by mutations in CYP2D6 or CYP2C19 genes. Warfarin is not a pgp substrate and is not eliminated renally

Which of the following tests does not have to be completed on a high-risk compounded sterile preparation that will be administered by IV injection and is prepared in a lot size of 30 single-dose vials before release to a patient? A. bacterial endotoxin test B. visual disturbance C. sterility test D. verification of the sterilizing filter integrity E. LAL test

C. The bacterial endotoxin test (LAL test), visual inspection test, and bubble test should all be completed before the CSP is dispensed. Because the sterility test takes 14d, the preparation may be dispensed before the results are known; however, a system to recall the CSP must be in place if it does not meet the test's requirements

Which of the following is correct? A. filter integrity testing of the filter membrane is done to determine at what pressure the filter will break B. the manufacturer of the filter membrane determines the bubble point of the membrane, this value is always the same, no matter what solution has been filtered C. as the pore size of the filter membrane decreases, the pressure at which the air can be pushed from the largest pore increases D. the bubble point test is a destructive test E. it is not necessary to perform the bubble point test if a certificate of quality from the filter manufacturer is provided

C. The bubble test is not a destructive test, and the value depends on the solution being filtered. When a CSP is filter sterilized, the bubble point test must be done before the preparation may be dispensed

Sulfur Benzoyl peroxide aa 10% PEG base qs ad 90g Base: PEG 3350 1 part, PEG 400 1.5 parts What quantity of base is required for the preparation? A. 81g B. 76.2g C. 68.8g D. 72g

C. The combined weight of the benzoyl peroxide and sulfur is 21.2g (12.2g benzoyl + 9g sulfur). Because 90g must be compounded, 90-21.2g = 68.8g of base is needed

What is the characteristic of a preparation in the moderate compounding category according to USP chapter 795? A. It requires manipulation of a commercial product by adding a component B. It has a USP compounding monograph C. It requires special calculations to determine component quantities per preparation of individualized units D. It has appeared in a peer-reviewed journal

C. The compounding of troches is an example of a preparation in the moderate category. The cavities of the mold used to make them require calibration. The USP recognizes three compounding categories of simple, moderate, and complex

When administered iv, darbepoetin alfa has a terminal t 1/2 approximately ___ that of epoetin alfa? A. 2fold shorter than B. 2fold longer than C. 3fold longer than D. 3fold shorter than

C. The half-life of darbe is 3x longer than that of epo, giving this agent the added benefit of reduced frequency of admin

What condition usually increases the rate of drug dissolution for a tablet? A. Increase the particle size of the drug B. Decrease the surface area of the drug C. Use of ionized, or salt, form of the drug D. Use of the free acid or free base form of the drug

C. The ionized, or salt, form of a drug is generally more water soluble and therefore dissolves more rapidly than the nonionized (free acid or free base) form of the drug. According to the Noyes-Whitney equation, the dissolution rate is directly proportion to the surface area and inversely proportional to the particle size. Therefore, an increase in the particle size or a decrease in the surface area slows the dissolution rate

The lowest concentration of anti-infective that prevents microbial growth is called the A. minimum bactericidal concentration B. minimum bacteriostatic concentration C. minimum inhibitory concentration D. minimum inhibiting concentration E. minimum Schillings concentration

C. The minimum inhibitory concentration determines the level of anti infective to which dosing regimens may be set

Which of the following statements about the GPIs is LEAST accurate? A. abciximab (Reopro), eptifibatide (integrilin), and tirofiban (Aggrastat) are all administered as a bolus followed by a CI B. it is possible to experience an allergic rxn after repeated exposure to reopro C. eptifibatide, tirofiban, and abciximab can all be reversed by a platelet infusion D. tirofiban and integrilin are renally eliminated, therefore, dosage adjustment is required for pts w/ renal dysfn E. all are indicated as adjuncts to pci

C. The only GPI that is reversed by a platelet infusion is reopro, making C the least accurate statement.

Positive: 30 (present), 40 (absent), 70 (total) Negative: 20 (present), 10 (absent), 30 (total) What is the predictive value positive for the test? A. 30/50 B. 10/50 C. 30/70 D. 10/30

C. The predictive value positive of a test is the probability of an individual having the disease when the test result is positive (positive present)/ (total positive)

Initial treatment of active TB infections in which no resistant strains of mycobacterium tuberculosis are suspected should include A. rifabutin and pyrazinamide B. rifampin and pyrazinamide C. ethambutol, rifampin, isoniazid, and pyrazinamide D. isoniazid, rifabutin, and pyrazinamide E. ethambutol and rifampin

C. The preferred treatment for active TB infx is a four-drug regimen consisting of ethambutol, rifampin, isoniazid, and pyrazinamide for the initial 2 month, followed by rifampin with isoniazid for 4 additional months.

A new pt presents to your clinic. He reports that it took him 3hrs to get ready for this appointment and that he returned home several times to check to see if the door was locked. During the interview, he straightens up your desk, making sure all square items are at right angles to each other. Which medication would be most appropriate for this pt? A. trazodone B. olanzapine C. fluvoxamine D. alprazolam E. lithium

C. The pt is displaying s/sx of OCD (long time spent getting ready, there is repeated checking behavior, and preoccupation with rearranging items to 90 degree angles) These activities are time consuming and limit his social and occupational functioning. Fluvoxamine is primarily used for OCD. SSRIs are first-line agents in the treatment of OCD. Clomipramine, a tricyclic, is also FDA approved for OCD. Trazodone is an antidepressant most commonly used for insomnia

Which of the following is correct? A. the rabbit test is the most sensitive because it can detect pyrogens from all sources B. the rabbit test is an in vitro test C. some drugs may inhibit the formation of a gel in the BET D. no drug will enhance the formation of the gel in the BET E. the pyrogen test is a quantitative test

C. The pyrogen (rabbit) test is an in vivo test and is not as sensitive as the BET. It is not a quantitative test.

Which of the following is correct? A. the rabbit test and the LAL test are the same test B. LAL reagent will determine the fever-producing potential of the pyrogens C. there are two types of techniques for the BET: the gel-clot technique and the photometric technique D. the CSP being tested has no effect on the test E. all CSPs may be tested using the rabbit test

C. The pyrogen test, also known as the rabbit test, determines the fever-producing potential of the pyrogens. The BET is also known as the LAL test. The drug product can inhibit or enhance the gel formation in the BET

Which of the following statements is true? A. regression analysis can determine whether 2 variables have a causal relationship B. correlation analysis can determine whether 2 variables have a causal relationship C. if there is a linear relationship between 2 variables, the relationship outside the range of the values of the independent variable is not necessarily linear D. a linear regression model can be used to predict the value of the dependent variable for any values of the independent variable

C. The relationship between the variables may not be the same outside the range of values of the independent variable

Which parts of the syringe are considered critical sites? A. ribs only D. collar only C. ribs and tip only D. collar and tip only E. ribs, collar, and tip

C. The ribs of the plunger and the tip of the syringe are critical sites of the syringe.

All of the following are contraindications or precautions associated with sirolimus EXCEPT A. de novo lung transplant recipient B. hyperlipidemia C. dm D. de novo liver transplant recipient E. allergy to sirolimus

C. The use of sirolimus in de novo lung and liver transplant recipients is ci b/c of increased incidence of fatal ade. Additionally, use of sirolimus in pts w/ uncontrolled hyperlipidemia is strongly discouraged b/c of its profound effects on lipid biosynthesis and catabolism

Which of the following is correct concerning a vertical laminar flow hood? A. it is always a biological safety cabinet (BSC) B. in vertical laminar flow, the hands of the operator must not be behind an object C. a vertical laminar flow hood has turbulent airflow within 1 inch of the work surface D. a vertical laminar flow hood has the laminar airflow blowing at the operator E. the operator works in a vertical flow hood and a horizontal flow hood in the same manner

C. There are several types of vertical laminar flow hoods, of which the BSC is one. The operator must never work over the top of items in hte hood, and all work should be done at least 1 inch from the work surface

Which of the following diuretics may retain its effectiveness at a gfr <30ml/min A. hctz B. chlorothiazide C. metolazone D. spironolactone

C. There is some evidence that metolazone is beneficial in pts with kidney dx and a GFR >30. This benefit is NOT the case for other thiazide or thiazide like diuretics or w/ K sparing diuretics. Metolazone is frequently used in comb w/ loop diuretics for this reason

A 42yo male (70kg) on hemodialysis 3x weekly receives epoetin alfa for treatment of anemia. He has been stable on an epoetin dose of 4000u IV 3x weekly with an average hemoglobin of 11g/dL (hct 33). Over the past 3mo, his hgb has dropped to 9g/dL. Iron indices reveal the following: serum ferritin 78 and transferrin saturation 12%. The best initial treatment for this pt is to A. increase dose epoetin alfa to maintain a hgb of 11-12 B. admin iv iron (sodium ferric gluconate) at a maintenance dose of 125mg/w C. administer 1g total dose of IV iron in divided doses D. begin oral ferrous sulfate 325po

C. This pt is iron deficient d/t low serum ferritin (<100ng/mL) and transferrin sat <20%. NO change in epo should be made until the iron deficiency is corrected. This pt will require a full course of iron (1g administered IV in divided doses w/ each dialysis session) as opposed to maintenance dose, which should be administered once the pt is iron replete. Sodium ferric gluconate may be administered in doses of 125mg /dialysis session for eight sessions to get the total 1g dose. Ferumoxytol would be administered as 2 510mg doses given 3-8d apart. Absorption of oral iron is poor, making IV iron preferred in this HD pt.

Which of the following factors is a contraindication to the use of HT in postmenopausal women? A. diabetes B. basal cell skin cancer C. thromboembolic dx D. depression E. obesity

C. Thromboembolic disease is a definite CI to the use of HT therapy in postmenopausal women

78yo hispanic female PMH: hypercholesterolemia, anemia, cad, htn Which of the following of the pt's conditions could the synthroid exacerbate? A. hypercholesterolemia B. anemia C. cad D. htn E. constipation

C. Thyroid hormones enhance oxygen consumption and increase oxygen demand. THe pt has a pmh of cad and is elderly. Thyroid supplements would actually lower her cholesterol in the long run but may precipitate angina acutely.

32yo wm presents with blood diarrhea (fewer than 4 stools per day) for 2 days. PE, CBC, and ESR are normal. Colonoscopy reveals distal colitis. Which of the following is the best choice for initial therapy for him? A. prednisone 40mg po qd B. sulfasalazine 4-6g po qd C. mesalamine 1-4g rectally qhs D. mesalamine 4-6g po qd E. methylprednisolone 16mg IV q8h

C. Topical aminosalicylates are more effective than oral aminosalicylates and topical steroids for mild distal UC. Oral and IV steroids are reserved for more severe cases of UC or cases that do NOT respond to oral and topical aminosalicylates. The oral dose of mesalamine in answer D is incorrect

Toprol XL is an agent that A. is CI in HF B. blocks B1,B2,a1 receptors C. blocks only B1 receptors D. should not be used in combo with Zestril E. increases the serum digoxin conc

C. Toprol XL is a cardioselective BB

Which of the following medication combination is CI? A. tylenol and ultram B. glucosamine sulfate and chondroitin C. ultram and parnate D. tylenol and glucosamine sulfate

C. Tylenol and Ultram are marketed therapeutically as Ultracet. Glucosamine sulfate with or without chondroitin is recommended as an alt therapy in the treatment of OA. The combination of Ultram and Parnate, a MAOI, is CI b/c of risk of serotonin syndrome

Certain factors may increase the risk of microbial contamination of a CSP. Which of the following would not be a risk factor? A. very complex compounding steps B. lengthy exposure of a critical site during compounding C. use of appropriate aseptic technique D. batch compounding without preservatives for multiple patients E. preparation of a CSP from nonsterile powders

C. Use of good aseptic technique is one way to ensure a good preparation

Which of the following trough concentrations should be targeted for a pt receiving vanco for the treatment of MRSA pna? A. 5-10 B. 10-15 C. 15-20 D. trough monitoring not necessary

C. Vanco trough conc targets of 15-20 are now recommended for the majority of vancomycin treatment indications with the exception of ssti, which require lower targets (10-15mcg/mL)

What would be an appropriate water or fluid requirement for a 60kg pt w/ no extrarenal fluid losses? A. 800mL B. 1200mL C. 2400mL D. 3600mL E. 4800mL

C. Water requirements are 30-40mL/kg/d for pts w/o extrarenal fluid losses; 60kg x 40mL/kg/d = 2400mL/d

Which of the following statements is NOT true about the chi-squared test? A. the chi-squared test is a nonparametric test B. the test statistic of a chi-square test follows a chi-square distribution when the null hypothesis is true C. the chi-square test always applies when the data are counts or frequencies D. the rejection and nonrejection regions of the chi-square test are determined on the basis of the chi-square distribution

C. When the expected frequencies are too small, a chi-square test would not be appropriate

Which of the following macrolide antibiotics is used primarily to accelerate gastric emptying? A. azithromycin B. clarithromycin C. erythromycin D. telithromycin

C. With no emergence of newer macrolide antibiotics with enhanced activity, erythromycin is now used more commonly for its ade of enhancing motility than for its antibacterial action

Which of the following statements is true regarding Zyloprim? A. it works to decrease the formation of uric acid by inhibiting xanthine oxidase B. it does not require dosage adjustment in pts with renal insufficiency C. skin rxns, including SJS, have been reported with its use D. it should be used for the treatment of an acute gouty arthritis attack

C. Zyloprim's use has been associated with serious skin rxns that occur at any point during therapy. Inhibition of formation of xanthine oxidase and facilitates clearance of more water-soluble precursors of uric acid, oxypurines. XO is the RLS in uric acid synthesis

Calcium and phosphate can interact to form a precipitate in parenteral nutrition solutions. Of the follow situations, which would not enhance calcium and phosphate precipitate formation? A. high concentration of calcium and phosphate B. increase in solution pH C. decrease in temperature D. use of the chloride salt of calcium E. a slow infusion rate

C. an increase in temperature could enhance precipitate formation

The most common electrolyte abnormality associated with ACEI is A. hypomagnesemia B. hypokalemia C. hyperkalemia D. hyperphosphatemia E. hypernatremia

C. hyperkalemia b/c of aldosterone inhibition

Metformin should be withheld for 48hr prior to any procedure requiring the use of parenteral iodinated contrast medium because of the potential for which of the following ade? A. optic neuritis B. metabolic acidosis C. lactic acidosis D. purple toe syndrome E. tinnitus

C. lactic acidosis; monitor renal function

Persons on insulin therapy should be advised to rotate their injection sites for which of the following reasons? A. it reduces the risk of infection B. it reduces the risk of lipoatrophy C. it reduces the risk of lipohypertrophy D. it reduces the risk of generalized myalgia E. this advice is outdated b/c of the use of human insulin

C. lipohypertrophy is a bulging of the injection site is caused by nonrotation of injection sites. lipoatrophy (pitting) may occur d/t antigenic response to insulin

DM is the leading cause of which of the following microvascular complications? A. pancreatitis B. fatty liver C. blindness D. stroke E. deafness

C. retinopathy

If the powder is a chart should weigh 350mg, what is the acceptable weight range for dispensing? A. 280-420mg B. 300-400mg C. 315-385mg D. 333-368mg

D. A 5% error is allowed 350 x 1.05 =368 350 x 0.95 = 333

Which of the following is the most appropriate recommendation for a pt with stage 3 CKD, dm, and albumin-to-cr ratio of 200mg/g to delay progression of CKD? A. enalapril and losartan B. enalapril and aliskiren C. losartan and aliskiren D. enalapril only

D. A single agent is recommended (either ACEI or ARB). Combo therapy lacking evidence so not recommended. Avoid aliskiren and ACEI/ARB in dm d/t risk renal impairment, hypotension, and hyperkalemia and a warning to avoid if mod to severe renal impairment (<60)

Lisinopril may cause hemodynamically mediated aki by preventing which of the following compensatory mechanisms of the kidney? A. vasodilation of the afferent arteriole B. vasoconstriction of the afferent arteriole C. vasodilation of the efferent arteriole D. vasoconstriction of the efferent arteriole

D. ACEI may contribute to development of AKI in pts w/ condition resulting in prerenal kidney dx by preventing the compensatory vasoconstriction of the efferent arteriole mediated by at ii that occurs in an attempt to increase gfr

Appropriate monitoring parameters for enalapril therapy in the treatment of HF include A. serum calcium B. serum albumin C. thyroid stimulating hormone D. serum potassium E. hbA1c

D. ACEIs can increase serum potassium as well as creatinine

Which of the following are examples of tertiary lit? A. Medline and AHFS drug information B. Embase and Facts and Comparisons C. Medline and Embase D. AHFS DI and Facts and Comparisons

D. AHFS DI and Facts & Comparisons are examples of tertiary lit. Other types of tertiary lit include Lexi-Comp DI Handbook and Micromedex

A pt is receiving phenytoin capsules 300mg qd for seizure control. She requires tube feeding w/ a 1kcal/mL formulation at 85mL/h (2000mL/d). What would be the most appropriate intervention to maintain a therapeutic drug concentration and maintain the required nutrition support? I. increase the dose of phenytoin to 600mg/d II. hold the EN 2hr before and after the dose III. increase the EN to 100mL/h x 20hr A. I only B. III only C. I and II only D. II and III only E. all of the above

D. Absorption of phenytoin is markedly impaired when it is given concurrently with enteral tube feeding. The enteral tube feeding should be held 2hr before and after the daily dose of phenytoin capsules To maintain the current dose of EN, the rate of feeding should be increased to 100mL/h x 20h (2000mL/d)

In a pt with metabolic acidosis, what anion salt would you use to add the majority of sodium and potassium to a PN formulation? A. chloride B. gluconate C. phosphate D. acetate E. sulfate

D. Acetate is converted to bicarb in the liver and would thus help or at least not exacerbate the metabolic acidosis

Which of the following mechanisms most likely contributes to the benefits of B-blockers in the treatment of heart failure? A. stimulation of B2 receptors B. increased HR and decreased BP C. stimulation of B1 receptors D. blockade of increased sympathetic NS activity E. blockade of ATII receptors

D. Activation of sympathetic NS plays an important role in the initiation and progression of HF. THe benefits of BB are thought to be due to the blockade of the sympathetic NS's increased activity

A 64yo male with SIHD c/o angina that occurs after walking 2-3blocks. No lesions detected on this angio are amenable to intervention. HR 58-62, BP 130/68. Current meds: asa, crestor, metoprolol, ramipril, and tadalafil. Which of the following interventions will be of most benefit to treat this pts angina? A. add imdur 60 po qd B. add dilt 180 po qam B. increase metoprolol to 100mg bid D. add amlodipine 5mg qd E. add ranolazine 1000mg bid

D. Adding a CCB will give additional antianginal effects. Using amlodipine rather than a stronger av nodal blocker such as dilt is important given the pts low hr. Ranolazine is another option but dose should be started at 500mg bid

Which of the following statements is correct regarding the treatment of bipolar disorder? A. lithium and divalproex sodium are considered first-line therapy options for mood stabilization B. when treating a pt w/ lithium, one must monitor the WBC and ANC b/c lithium's propensity to cause agranulocytosis C. pts diagnosed w/ bipolar I disorder exhibit intermittent cycles of mania and major depression D. A and C E. none of the above

D. Agranulocytosis is a side effect that is monitored weekly with clozapine therapy for 6mo. After 6mo, monitoring can be done q2w; after 6more months, monitoring can be done q4w for the duration of therapy

Which of the following therapies has been used for the treatment of bipolar disorder? A. olanzapine B. topiramate C. ccb D. all of the above E. none of the above

D. All of the therapies listed have been used for mood stabilization in bipolar disorder

Many career options are available to pharmacists in a variety of practice settings. Which of the following is true regarding the practice of providing drug information? A. pharmacists practicing in a community pharmacy do NOT need to have drug information skills B. only specialty pharmacists practicing in drug info centers need to have skills on di C. with expanding technologies, pharmacists, do not need to know about drug information references D. all pharmacists need to have skills in providing di

D. All pharmacists need to have skills in providing DI, regardless of practice setting, including specialty pharmacists practicing in di centers as well as those practicing in a community pharmacy. With expanding technology, it is crucial that pharmacists know about available di references

When Ms. Tiny presents the tamoxifen prescription, you notice that the directions are missing. You call the health care provider to clarify the instructions for this pt. Which of the following is a correct choice. A. 10ml po qd B. 20ml po qd C. 40mg po bid D. 20mg po qd

D. Breast cancer dose is 20mg po qd. Drug is NOT available in liquid form

66yo wm presents with chest pain that is sharp, aching, and nonradiating x past few weeks. Occur mainly during daily walk and relieved at rest. EKG normal. Cath 2y ago. Troponins negative PMH: HTN, PUD, asthma, CAD FH: significant for CVA, DM2, and MI SH: smokes 1ppd x40years, EtOH socially Meds: Proventil, Flovent, Prilosec, asa, hctz Vitals: 148/92, HR 82, RR 18, 72 inches, 200lbs Considering comorbidities, which of the following would be the most appropriate therapeutic intervention at this time? A. SL NTG PRN B. Inderal C. Zestril and SL NTG PRN D. Cardiezem and SL NTG PRN E. Coreg and SL NTG

D. All pts with chronic chest pain need SL NTG for acute use but SL NTG is not ok as monotherapy. CCB would help control his angina as well as lower his BP. Although BB should be considered first line in pts with stable angina for anti-ischemic effects, CCB can be additive or used in settings where a CI exists. In this case, the pt has asthma so a cardioselective BB is needed. Both inderal (propranolol) and coreg (carvedilol) are not cardioselective. ACEI are not effective as antianginal agents

A 45yo m presents to his local health care provider w/ a complaint of extreme stiffness for the past 2 months that begins in the morning and lasts until noon on most days. he also states that he feels "drained" all the time and that both of his knees are swollen and painful. Also notes rheumatoid nodules. Labs significant for elevated CRP and ESR and +Rh factor. States that he has been taking OTC ibuprofen at a dose of 200mg 2-3x/d w/o relief. Which of the following represents the best drug therapy for this pt? A. increase dose of ibu to 800 tid B. increase dose of ibu, add mtx 25 bid C. increase dose ibu, add celecoxib 100 bid D. increase dose ibu, add leflunomide at a dose of 100 qd x3, then decrease to 20 qd

D. Although the pt currently has room to increase his dose of NSAID, he would benefit from the addition of a DMARD. This pt has a dx duration of less than 6mo w/ mod dx and poor prognostic factors. MTX represents a viable option but the dose is wrong (should be 7.5-25 qweekly). The addition of leflunomide is the best option

A pt w/ nephrotoxicity caused by tobramycin would likely present with an increase in serum creatinine A. immediately after starting therapy and with nonoliguria B. immediately after starting therapy with oliguria C. 5-7d after starting therapy and with oliguria D. 5-7d after starting therapy and with nonoliguria

D. Aminoglycoside induced nephrotoxicity is characterized by a delay in changes in serum creatinine (approx 5-7d) and relatively normal urine output

Which of the following most accurately characterizes aminoglycoside toxicity? A. thrombocytopenia and neutropenia B. CPK elevations and myalgias C. QT prolongation and risk of arrhythmia D. nephrotoxicity and ototoxicity

D. Aminoglycosides cause both nephrotoxicity and ototoxicity. Nephrotoxicity is exhibited as an acute tubular necrosis that is usually reversible and seldom requires dialysis. Ototoxicity is due to eighth cranial nerve damage and may be irreversible

Which of the following is NOT an acceptable indication for testosterone? A. anemia B. hypogonadism C. delayed puberty D. body building E. metastatic breast cancer

D. Anabolic steroids may be abused by those who are seeking enhanced muscular development and endurance, such as athletes. For this reason, all of these agents are subject to the controlled substance act

Because combination DMARD therapy may be more efficacious in the refractory RA population, which of the following represents the BEST choice for combination therapy? A. arava 20 qd + rheumatrex 5 qd B. remicade 3mg/kg IV + enbrel 50 qweekly C. myochrysine IM weekly, + plaquenil 200 bid D. remicade 3mg/kg IV q2mo + rheumatrex 25mg qweekly

D. Arava plus MTX (Rheumatrex) may be a very efficacious combination, but it increases the risk of liver toxicity significantly, and MTX should not be given daily. Gold therapy (myochrysine) in combo with plaquenil increases the risk of rash (although it may rarely be used together), and gold therapy should be given monthly. Remicade and Enbrel, two biologics, should not be used together. Remicade is approved for use in combo with MTX. This combo is the best choice

54yo male presents to ED with crushing substernal chest pain and radiation to left arm. PMH: htn, copd, gout. H/o smoking x 30y and occasional EtOH. BP 170/85, hr 72, rr 18, and temp 97. Before admission, took EC ASA 81, combivent, tiazac, and zyloprim Allergies: sulfa EKG: ST depression, Twave changes Troponins positive LVEF <35 Dx: NSTEMI, HF What is the preferred BB for this pt? A. propranolol B. carvedilol C. labetalol D. metoprolol E. nadolol

D. B/c h/o copd, need cardioselective bb (metoprolol). Remember after M is NOT cardioselective and carvedIlol (NOT olol) and propanAlol (NOT olol) is not cardioselective and has a activity.

Each of the following can be symptoms of afib except A. dizziness B. palpitations C. angina D. htn E. sudden-onset slurred speech

D. B/c loss of functional atrial contractions and rvr (producing palpitations), cardiac output may decrease, resulting in decreased perfusion of major organs, particularly the brain (dizziness, confusion, etc) and heart (angina, and HF exacerbation). Depending on vascular tone, BP may remain stable or fall as a direct result of decreased co; however, htn would not be expected. Pts w/ afib are at increased risk of thrombus, particularly stroke, secondary to pooling of blood in the left atrium and subsequent thrombus formation

The use of glucocorticoids is associated with numerous adverse effects and long-term consequences. Which of the following are initiatives to treat, prevent, or minimize these ade? A. instructing the pt to take the glucocorticoid in divided daily doses B. instructing pts on long-term therapy to add elemental calcium and 400-800IU of daily ergocalciferol to their regimen C. adding daily chondroitin for arthritis to help offset bone effects of steroids D. informing pts that stopping glucocorticoid abruptly is CI

D. B/c of adrenal suppression that occurs with long-term glucocorticoid therapy, pts should taper off the agent

What antihypertensive agent should not be used in a pt with essential htn and a h/o depression w/ suicidal ideation A. captopril B. prazosin C. metolazone D. reserpine E. amlopidine

D. B/c of its possible increased risk of depression, reserpine should not be used in pts for whom the risk of depression or suicide already exists

The healthcare provider recommends switching to a less constipating opioid. Which of the following medications is least likely to cause constipation? A. ER morphine B. methadone C. oxycodone ER D. transdermal fentanyl patch E. hydromorphone

D. B/c transdermal delivery bypasses absorption from the GI tract, constipation has been reported to be less frequent w/ transdermal fentanyl than with other opioids

A 29yo depressed black female has shown little improvement w/ fluoxetine treatment so the psychiatrist decides to change her medication to tranylcypromine. What is your recommendation for the switch? A. gradually decreased fluoxetine dosage over a 4w period, and then start tranylcypromine B. wait 2w after stopping fluoxetine and then being tranylcypromine C. over 6w, gradually decreased fluoxetine dosage as you gradually increase the tranylcypromine dosage D. wait 5w after stopping fluoxetine before initiation tranylcypromine E. maintain fluoxetine dosage and start tranylcypromine, stop the fluoxetine when the tranylcypromine has achieved a therapeutic level

D. Because of fluoxetine long half-life, 5 weeks should pass before initiating MAOI therapy. If fluoxetine is not cleared from the body by the time the MAOI is started, there is a risk of developing serotonin syndrome

A new anticonvulsant has just been approved by the FDA. It's bioavail>95% and its highly protein bound to a1-acid glycoprotein. It undergoes extensive hepatic metabolism by 2c9. Less than 5% excreted unchanged in urine. It is known to inhibit 3a4. A pt on this anticonvulsant has developed significant depression and is being started on an antidepressant that is 93% bound to albumin and is potent inhibitor of 2c19. The antidepressant is a pro-drug that is metabolized by 3a4 to an active metabolite that is hepatically cleared by 2c9. The neurologist wants to know if any di may occur that would necessitate a change in drug dosage. Which of the following is the appropriate response? A. no drug interactions should occur in this pt B. the dose of the anticonvulsant should be reduced b/c of a potential pr bound interaction that would increase the serum concentration of the anticonvulsant C. the dose of the anticonvulsant should be increased D. the pt may not benefit from the antidepressant, and another antidepressant that is not metabolized by cyp3a4 should be used E. b/c of interaction in the gut that decreases bioavailability, the dose should be increased

D. Because the new anticonvulsant inhibits cyp3a4, D is the correct answer.B/c the antidepressant is a prodrug, which must be metabolized to become active, it may not be effective

Mary D (68kg, 47yo) has recently received her first 0.25 dose of digoxin. Plasma digoxin conc 12 and 24hr following oral admin of this dose are 0.72 and 0.33mcg/L, respectively. The therapeutic plasma conc range is 0.8-2mcg/L. Predict Mary's digoxin trough conc at ss, assuming that oral digoxin therapy is continued at a dose rate of 0.25mg qd A. 0.62mcg/L B. 0.93mcg/L C. 1.32mcg/L D. 1.57mcg/L E. 1.95mcg/L

D. Because trough conc after the first dose are known (0.33mcg/L=Css,1), trough conc during multiple dose at ss can be predicted by multiplying the trough after the first dose w/ Css,min= Css,1 x 1/e^(-k(tau)) where tau=12hr, and k=ln(0.72/0.33)/(12hr)=0.065/h Css,min= 0.33mcg/L x 1/e^(-0.065(12)) = 1.57mcg/L

Which of the following surfactants is incompatible with bile salts? A. Polysorbate 80 B. Potassium stearate C. SLS D. Benzalkonium chloride

D. Benzalkonium chloride is a cationic surfactant and can interact with bile salts

Tertiary syphilis in adults should be treated with A. benzathine penicillin G 2.4million units x 1d B. azithromycin 1g orally once C. aqueous crystalline pen G 4million units q4h for 10-14d D. benzathine pen G 2.4million units once a week for 3 w E. doxycycline 100mg po bid for 7d

D. Benzathine pen G 2.4mil units once a week for 3 weeks is standard therapy for either late latent syphilis or tertiary syphilis. Single dose benzathine pen G is appropriate for primary, secondary, or early latent syphilis. Response C is the treatment for neurosyphilis

A nurse would like to know if she can administer the benadryl and zantac in the same iv line simultaneously. Which of the following resources will provide you with this information? A. wolter's kluwer health's facts and comparisons B. trissel's handbook on injectable drugs and drug prescription in renal failure C. micromedex and the sanford guide to antimicrobial therapy D. trissel's handbook on injectable drugs and micromedex

D. Both Trissel's and Micromedex IV compatibility tool can be used to assess whether benadryl and zantac simultaneously

Which of the following conditions alters the pk profile of cyclosporine? I. biliary obstruction II. malnutrition III. hyperglycemia A. I only B. II only C. III only D. I and II only E. all of the above

D. Both biliary obstruction and severe malnutrition would change the pk of cyclosporine. Cyclosporine requires bile for emulsification and absorption. If bile flow is obstructed, then the bioavailability would significantly decrease. Additionally, gengraf is a highly lipoprotein-bound drug. In severe malnutrition, total protein stores are depleted, therapy increasing the total free drug

Which of the following drugs carry an FDA boxed warning? A. carbamazepine and levetiracetam B. felbamate and levetiracetam C. lamotrigine and clobazam D. carbamazepine and felbamate E. phenytoin and carbamazepine

D. Both carbamazepine and felbamate are associated with aplastic anemia and hepatic failure. Valproic acid has a risk of hepatotoxicity/ pancreatitis and lamictal with a serious rash. Keppra has not FDA boxed warning.

RY is a 67 yo male w/ a CC of swollen big left toe and extreme pain. The area is red and tender. Laboratory analysis reveals a uric acid level of 10mg/dL. Review of RY's past medical history reveals HTN and CHF. A diagnosis of gout is made. Which of the following is the best choice for the treatment of RY's acute gouty arthritis attack? A. probenecid 500mg now, followed by 500mg bid B. indomethacin 50mg now, followed by 50mg tid-1id C. allopurinol 100mg qd D. colchicine 1.2mg followed by 0.6mg in 1hr if sx persist

D. Both probenecid and allopurinol may exacerbate an acute gouty arthritis attack and should be reserved for the prevention of further attacks only. Indomethacin is an option for the treatment of an acute gouty arthritis attack; however, because of NSAIDs' tendency to cause fluid retention in the renal tubules, it would not be the ideal agent in a pt with CHF. Colchicine represents the best option from this list

A 42yo woman has been successfully treated w/ valproic acid for years, but she has experienced some undesirable ade. She is slowly titrated onto a new anticonvulsant, and the va is gradually d/c'd. She presents to the ED with severe flank pain and is diagnosed w/ kidney stone. Which of the following may have precipitated her current situation? A. gabapentin B. lamotrigine c. levetiracetam D. topiramate E. phenytoin

D. Both topiramate and zonisamide may cause kidney stones. Although neither agent is ci in an individual with a h/o kidney stones, these drugs should be used cautiously in such pts. Pts should be counseled to remain adequately hydrated b/c doing so may decrease risk of stone formation

Which of the following products is a four phasic oral contraceptive? A. nordette B. ortho tri-cyclen C. nor QD D. natazia E. necon 1/35

D. Natazia is a four phasic oral contraceptive

Which of the following is true for CD? I. infliximab is the drug of choice for mild to mod dx II. oral corticosteroids are the drugs of choice for maintenance therapy III. topical aminosalicylates are the drugs of choice for mild to moderate dx IV. oral cyclosporine is the drug of choice for maintenance therapy V. budesonide should be used as initial therapy for CD of the ileum and right colon A. I, II, and III B. I and III C. II and IV D. V only E. all of the above

D. Budesonide is the treatment of choice for mild to moderate CD of the ileum and right colon. Infliximab is reserved for moderate to severe dx in pts who do NOT respond to oral corticosteroids or immunosuppressive agents. It may also be used in pts in whom ade from oral corticosteroids must be avoided. Oral corticosteroids should not be used for long-term maintenance therapy. Oral budesonide is the drug of choice for mild to moderate CD, not topical corticosteroids. Oral cyclosporine has no role in CD. IV cyclosporine may be used in severe or fulminant CD that does not respond to 5-7d of IV corticosteroids

Which of the following characteristics is not correct regarding benzos? A. they potentiate the effect of g-aminobutyric acid B. they are highly lipophilic C. they are cross-tolerant w/ alcohol D. their pharmacologic action is similar to that of buspirone E. there is risk of seizure if they are abruptly d/c'd

D. Buspirone moa is agonism of 5-HT1a receptors Buspirone also possess moderate affinity for D2 receptors. It does not interact with the BZD-GABA receptor complex like the BZDs do. The BZDs cross the BBB and are cross-tolerant w/ alcohol. If BZDs are abruptly d/cdd seizures can result

Which of the following may exacerbate GERD sx by lowering the LES pressure? A. quinidine B. iron C. potassium chloride D. diltiazem E. tetracycline

D. CCBs decrease LES pressure. All others have direct irritant effects on esophageal mucosa.

The pk parameters for captopril in healthy adults are Cl=800mg/min, fe=0.5, Vss=0.81L/kg, plasma pr binding 75% Captopril is a weakly basic drug that is used in the treatment of HTN. Assume a GFR of 125ml/min. What is the mechanism for renal cl of captopril? A. Filtration only B. Reabsorption only C. Secretion only D. Filtration and net secretion E. Filtration and net reabsorption

D. CLrenal = Cl(tau) x fe CLrenal = 800mL/min x 0.5 = 400mL/min Eratio = CLrenal/ fup x GFR Eratio = 400/ 0.25 x 125ml/min = 12.8 Eratio > 1 filtration and net secretion

What is the most appropriate calcium intake for adults over 65yo? A. 600-800mg B. 800-1000mg C. 1000-1200mg D. 1200-1500mg E. 1500-1800mg

D. Calcium requirements: Males <65: 1000mg/d, >65: 1200-1500mg/d Females >50yo is 1200-1500, <50yo is 1000-1200mg

Beth R (58kg, 63yo) is suffering from symptomatic ventricular arrhythmia. She will be started on an oral multiple-dose regimen with the antiarrhythmic mexiletine. The population average values of mexiletine for clearance and volume of distribution are CL=0.5L/h/kg and V=6L/kg, respectively. Although a therapeutic range of 0.5-2mg/L has been described, avoiding large peak-to-trough fluctuations is recommended. The available oral dosage forms are 150,200,and250mg capsules with an oral bioavailability of F=0.9. Design an appropriate and reasonable oral dosing regimen that keeps the plasma conc at an average conc of approx 1mg/L, with a peak-to-trough fluctuations </= 100% (ex: with conc w/in the limits of 0.75 and 1.5mg/L) A. 150mg q6h B. 200mg q6h C. 200mg q8h D. 250mg q8h E. 375mg q12h

D. Calculate necessary DR to maintain Css,av=1mg/L DR= D/tau = Css,av x CL/F DR= 1mg/L x (0.5L/h/kgx 58kg)/0.9 DR= 32.2mg/h maximum dosing interval tau= ln(Css,max/Css,min)/K tau= ln(Css,max/Css,min) x V/ CL tau= ln(1.5/0.75) x (6L/kg) / (0.5L/h/kg) = 8.3h 32.2mg/h x 8hr = 257.8mg Dosing regimen: 250mg q8h

Which process best describes how the following preparation should be compounded? Camphor 1% Menthol 1% Thymol 0.5% White petrolatum qs ad 30g M.ft. ointment A. Dissolve the camphor, methanol, and thymol in alcohol, and incorporate them into the white petrolatum by geometric dilution B. Dissolve the camphor, methanol, and thymol in glycerin, and incorporate them into the white petrolatum by geometric dilution C. Dissolve the camphor, methanol, and thymol in propylene glycol, and incorporate them into the white petrolatum by geometric dilution D. Form a eutectic mixture with the camphor, methanol, and thymol, and incorporate it into the white petrolatum by geometric dilution

D. Camphor, methanol, and thymol are three components that liquefy when mixed together, forming a eutectic mixture. This liquid mixture is then gradually incorporated into the white petrolatum by geometric dilution

NSAID side effects you should counsel a pt about include A. cat x teratogenicity B. neurologic and immunologic effects C. gi and dermatologic effects D. cardiovascular, renal, and gi effects

D. Cardiovascular, renal, and GI side effects are the primary counseling points to cover

Which of the following cephalosporins is classified as a first generation agent with enhanced gram positive activity? A. cefuroxime B. ceftazidime C. cefepime D. cefazolin

D. Cefazolin is an IV, first gen cephalosporin with extensive gram positive antibacterial activity. it is considered an agent of choice for MSSA along with the penicillinase-resistant penicillins

A pt is taking prednisone 40mg qd x 6mo. On abrupt cessation, which of the following may occur? A. myopathy B. diabetes C. infx D. adrenal crisis E. psychosis

D. Chronic administration of glucocorticoids such as prednisone will lead to feedback inhibition of pituitary ACTH release and atrophy of the adrenal cortex. When prednisone is abruptly stopped, the adrenals will not be able to meet the body's demand for cortisol during severe stress, and adrenal crisis may occur

Which group of genes encodes for antigens that are responsible for self- or nonself- recognition? A. Class I HLA B. Class II HLA C. Class III HLA D. MHC E. minor histocompatibility complex

D. Class I and Class II HLA are the actual antigens important for self- and nonself- recognition. The group of genes that encode for these antigens is the major histocompatibility complex

Clinical research generally seeks to answer questions in all of the following health care areas except A. disease prevention B. diagnosis C. treatment D. reimbursement

D. Clinical research generally seeks to answer questions related to disease prevention, diagnosis, and treatment. Reimbursement is a payment issue not normally addressed in clinical trials

Which of the following is a drug discovery strategy that uses nucleic acids and amino acids in various combinations to synthesize vast libraries of oligonucleotide or peptide compounds for high-throughput lead compound screening? A. whole cell screening B. natural product screening C. gene therapy D. combinatorial chemistry E. rDNA technology

D. Combinatorial chemistry

Which trace element should be reduced or removed in pts with cholestasis who are receiving PN? A. zinc B. chromium C. selenium D. copper E. iodide

D. Copper is excreted via the biliary tract. Pts with severe cholestasis should have copper removed during short-term PN. In long-term PN, copper may be required in reduced doses to prevent anemia. Serum copper concentrations should be monitored regularly in long-term pts who have cholestasis

Which of the following drugs may be used to diagnose adrenal insufficiency? A. desmopressin B. clemastine C. captopril D. cosyntropin E. aminoglutethimide

D. Cosyntropin (Cortrosyn) is a synthetic analogue of ACTH that is used to diagnose adrenal insufficiency. It works by stimulating the adrenal cortex to secrete cortisol. If cosyntropin administration does not result in an appropriate increase in cortisol release, adrenal insufficiency is present.

Which of the following disease states is not suitable for a crossover study design? A. glaucoma B. migraine headaches C. epilepsy D. pna

D. Crossover study designs can be used only for stable, chronic, or episodic conditions. Glaucoma, migraine headaches, and epilepsy meet this criterion. PNA does not meet this criterion, and therefore, is generally not suitable for a crossover design

Following major GI resection, a pt w/ severe short bowel syndrome is started on PN. It is anticipated that this pt may need this therapy for 6mo to 1year. The PN prescription for this pt includes D20W aa 5% at 105mL/h (2500mL/d) IV fat emulsion 20% at 10mL/h x 24h (240mL/d) 0.45% saline at 50ml/h x 24hr (1200mL/d) How many calories from dextrose will this pt recieve daily? A. 1100 B. 1300 C. 1500 D. 1700 E. 1900

D. D20W = 20g/100mL 2500mL/d D20W = 2500mL x 20g/100mL = 500g D 1g dextrose = 3.4kcal 500g x 3.4kcal = 1700kcal

Which of the following is NOT true of drug information provided by pharmacists? A. it may be tailored to a specific patient B. it may be developed for the benefit of a large group of pts with a common medical need C. it may be written or verbal information provided to patients or health care providers D. it may be only written information provided to patients or health care providers

D. DI includes written or verbal info provided to pts or health care providers. DI provided by pharmacists may be pt tailored or developed to benefit a large group of pts with a common medical need

The anticoagulant effect of dabigatran is mediated by A. inhibiting synthesis of vitamin K dependent clotting factors B. blocking the platelet P2Y12 receptors inhibiting clotting factor C. inhibiting clotting factor xa D. direct thrombin inhibitor E. inhibiting the VKORC1 enzyme

D. DTI

Decreased urine production is an effect of A. carmustine B. propylthiouracil C. ACTH D. desmopressin E. SSKI

D. Desmopressin (DDAVP) is a synthetic analogue of vasopressin, or antidiuretic hormone (ADH). Thus, it decreases urine production by increasing water reabsorption. ACTH is used to increase cortisol. SSKI and PTU are used in hyperthyroidism. Carmustine is used in cancer treatment.

Which of the following is true regarding digoxin therapy in pts with HFrEF? A. digoxin reduces mortality B. concomitant amio therapy decreases digoxin plasma concentrations C. digoxin is CI in pts with HFrEF and afib D. the target digoxin plasma conc is 0.5-1ng/mL E. concomitant glyburide therapy increases digoxin plasma concentrations

D. Digoxin only improves sx not mortality. Amio increases digoxin levels (decrease dig dose by 50%) and glyburide does not affect it

Doxorubicin is an antineoplastic agent that A. is not related to epirubicin and daunorubicin B. interacts with the microtubules of cells during mitosis C. has an oral dosage form commercially available D. causes cumulative cardiac toxicity

D. Doxorubicin is an antitumor antibiotic related to epirubicin and daunorubicin. These agents act by binding tightly to DNA through intercalation and by inhibit the topoisomerase II enzyme. Doxorubicin does have a liposomal IV product, but it is not available orally. All anthracyclines are associated w/ cardiac toxicity and hae cumulative dosing limits to prevent this

A drug with which of the following characteristics is most likely to be removed by HD? A. fu 0.05, Vd 0.2L/kg B. fu 0.05, Vd 0.6L/kg C. fu 0.30, Vd 0.6L/kg D. fu 0.95, Vd 0.2L/kg

D. Drug characteristics that make that make an agent more likely to be removed by dialysis include low protein binding, small volume of distributions, and low mw. Among the choices given, the agent that best meets these criteria is choice D, which has a high fraction unbound in the plasma and a low volume of distribution

Drug information is A. electronic information pertaining to medications only B. written information pertaining to medications only C. verbal information pertaining to medications only D. written, electronic, and verbal information pertaining to medications

D. Drug information is written, electronic, and verbal information pertaining to medications

Epoetin alfa and darbepoetin alfa stimulate erythropoiesis by which of the following? A. prevention of excessive rbc destruction B. prevention of degradation of bone marrow C. differentiation of peritubular interstitial cells of the kidney D. differentiation of erythroid progenitor stem cells in the bone marrow

D. ESAs including epo and darbe work in the bone marrow to stimulate differentiation of erythroid progenitor stem cells and results in an increase in rbc production

Laboratory markers of infx, such as C-reactive protein, WBC count, and erythrocyte sedimentation rate, may not be accurate in which of the following pt populations? A. elderly pts B. pts w/ COPD C. malnourished pts D. A and C E. all of the above

D. Elderly and malnourished pts may not be able to respond to appropriate lab markers of infections b/c of limited reserves or depletion of inflammatory factors

Which carbapenem antibiotic displays broad spec activity yet does not cover pseudomonas? A. imipenem-cilastatin B. doripenem C. meropenem D. ertapenem

D. Ertapenem is the only carbapenem that does not exhibit coverage against P. aeruginosa. It is also the only carbapenem that is administered once daily

All of the following are potential confounding variables except A. a subject's concurrent medications B. medical complications that occur during the study period C. study dropouts D. exclusion of pts with severe forms of the dx being studied

D. Exclusion of pts with several forms of dx being studies not a confounding variable b/c it can be controlled

Which of the following radiopharmaceuticals is indicated to image beta-amyloid neuritic plaques in patients exhibiting cognitive impairment? A. Tc-99m DTPA B. Tc-99m biscisate C. In-111 DTPA D. Florbetapir F-18

D. Florbetapir F-18 is the only one with the indication. Tc-99m DPTA is used to evaluate patients for brain death and Tc-99m bicisate is used for evaluation of stroke patients in the evaluation of brain death. In-111 DTPA is indicated for cisternography studies

The major troublesome side effects in nicotinic acid therapy is A. diarrhea B. vomiting C. hair growth D. flushing E. dizziness

D. Flushing occurs in many pts. To decrease flushing intensity, take asa 325 30 min prior to first dose of nicotinic acid. Itching may also occur with flushing

The absorption rate of a drug is most rapid when the drug is formulated as A. a controlled-release product B. a hard gelatin capsule C. a compressed tablet D. a solution

D. For a drug in solution, no dissolution is required before absorption. Consequently, compared with other drug formulations, a drug in aqueous solution has the highest bioavailability rate and is often used as the reference preparation for other formulations

A 48yo pt presents with a new dx of ckd. Which agent would be appropriate choice as initial therapy in this pt based on JNC8? A. clonidine B. guanethidine C. diltiazem D. perindopril E. nisoldipine

D. For pts w/ HTN and CKD, JNC8 recommends the use of ACEIs, ARBs, diuretics, and CCBs

Which of the following radiopharmaceuticals is used in assisting with the diagnosis of opportunistic infections? A. Thallous chloride T1-201 B. I-123 iobenguane C. In-111 pentetreotide D. Ga-67 citrate

D. Ga-27 citrate is useful for diagnosis of opportunistic infx, and it has application in oncology as well. T1-201 chloride is indicated for cardiology, not infectious disease; I-123 iobenguane and In-111 pentetreotide are both oncology indications.

Following an IV injection of Ga-67 citrate, Ga-67 disassociates from citrate and becomes immediately bound to A. erythrocytes B. leukocytes C. platelets D. transferrin

D. Ga-67 immediately binds to plasma transferrin, and it is not dependent on WBC, erythrocytes, or platelets for localization

Which of the follow statements is true? A. the response "don't know" should be left blank in the data set B. missing data should always be left blank in the data set C. all coding decisions have to be made after data collection and before data entry D. coding instructions may be printed on the data collection form

D. Generally, all coding decisions should be made before data collection. Coding instructions are then often printed on the data collection form

LB is a 45yo wf presenting to the EF with a fever of 103, flank pain, dysuria, urinary urgency, and frequency. Her lab tests are significant for an increased WBC counts of 18,000 cells/mm3 and 3% immature forms (bands). Her urinalysis received >105 cells/ mL of GNR. What infection does LB have A. herpes simplex B. gonorrhea C. syphilis D. urinary tract infx E. food poisoning

D. Given the clinical presentation and lab results, LD has a severe UTI. The presence of systemic sx suggest upper UTI or pyelonephritis

Chronic administration of glucocorticoids predisposes pts to which of the following? A. arthritis B. obesity C. alzheimer's dx D. osteoporosis E. hepatitis

D. Glucocorticoids have catabolic effects on a number of tissues, including muscle, fat, skin, and bone. Chronic administration leads to osteopenia and osteoporosis

Ms. Tiny calls you 2 months after starting antineoplastic therapy. She has been experiencing frequent hot flashes and wants to know if one of her medications might be causing this ade. You tell her that the hot flashes are likely caused by A. tamoxifen and desloratidine B. percocet and zoladex C. celebrex and tamoxifen D. zoladex and tamoxifen

D. Hot flashes are a common adverse effect experienced by pts taking selective estrogen receptor modulators (SERMs) such as tamoxifen, and gonadotropin-releasing hormone agonists such as Zoladex (goserelin)

Which of the following opioids is not appropriate for use as an around-the-clock medication in chronic pain? A. morphine B. oxycodone C. fentanyl D. hydromorphone E. methadone

D. Hydromorphone is an opioid with a short half-life with no available long-acting formulation. Thus, it is not useful as around-the-clock dosing

As an o/w type of ointment base, hydrophilic ointment, USP, possess which property? A. emollient B. greasy C. occlusive D. water washable

D. Hydrophilic ointment is water washable and does not posses the other properties listed of being an emollient that softens the skin, of being greasy because the external phase is water, and of being occlusive because it prevents water from evaporating from the skin

Mr. Doe was diagnosed with drug-induced hyperkalemia. Which medication on his profile could be responsible for this? A. mycophenolate mofetil B. furosemide C. prednisone D. tacrolimus E. asa

D. Hyperkalemia (incidence 20-40%) is a well-documented ade w/ tacrolimus

Hyperkalemia is a possible ADE of which of the following? A. trandolapril, captopril, felodipine B. felodipine, trandolapril, doxazosin C. doxazosin, captopril, amiloride D. amiloride, trandolapril, captopril E. captopril, felodipine, doxazosin

D. Hyperkalemia is a possible ADE w/ ACEIs, ARBs, and K+ sparing diuretics. Doxazosin is a peripherally acting a1 blocker. Felodipine is a CCB that is not linked to causing hyperkalemia.

Which of the following radiopharmaceuticals has indications for both cardiology and oncology? A. I-123 iobenguane B. I-123 ioflupane C. In-111 capromab pendetide D. F-18 FDG

D. I-123 iobenguane and In-111 capromab pendetide have oncology indications but not cardiology indications. I-123 ioflupane does not have either

Which component of a total nutrient admixture should be added last before storing it in a fridge? A. phosphorus B. magnesium C. trace elements D. calcium E. intravenous fat emulsion

D. If calcium is added last, the PN formulation will contain the final volume, including all other nutrients. The chance of calcium causing a precipitate will be decreased b/c all other components (ex: phosphorous) are diluted in the entire volume of the PN

Insulin therapy is indicated in all of the following except A. newly diagnosed dm1, a1c 7.5% B. GDM not controlled by diet C. hyperglycemic hyperosmolar nonketotic syndrome D. newly diagnosed dm2, a1c 7.9% E. dka

D. Newly diagnosed w/ a1c<10% should have a trial w/ MNT and exercise, if this nondrug therapy fails, oral antidiabetic monotherapy should be added

All of the following questions are essential vehicles for practitioners in keeping up to date with new literature except A. review articles B. clinical practice guidelines C. attendance at continuing education courses D. case reports

D. Important vehicles for practitioners relative to keeping up to date with new literature include review articles, clinical practice guidelines, and attendance at continuing education. Case reports are generally not an important method for keeping up to date with new literature

Dihydroergotamine differs from ergotamine in which of the following ways? A. DHE has higher incidence of n/v B. DHE has higher incidence of physical dependence C. DHE has no CI for ischemic CVD D. DHE has a higher incidence of diarrhea E. DHE can be administered concomitantly with a triptan

D. In contrast to ergotamine, DHE causes minimal peripheral vasoconstriction, little nauseas and vomiting, and no physical dependence. Diarrhea, however, is NOT prominent

Which of the following is the process of using potassium iodine when compounding Strong Iodine Solution, USP? A. coagulation B. coalescence C. comminution D. complexation

D. In solution, potassium iodide ionizes into potassium and the iodide ion. The iodide ion complexes with elemental iodine to form the soluble I3 complex. This process is called common ion complexation

All of the following are risk factors for the development of stress ulcers except A. sepsis B. coagulopathy C. mechanical ventilation D. age > 40yo E. burns

D. Increased age is not a risk factor for srmd

Which set of the following lab values is most consistent with a diagnosis of intrinsic AKI? A. urinary granular casts absent, FEna <1, urinary osmolality 600mOsm/kg B. urinary granular casts absent, FEna >1, urinary osmolality 600mOsm/kg C. urinary granular casts present, FEna <1, urinary osmolality 300mOsm/kg D. urinary granular casts present, FEna >1, urinary osmolality 300mOsm/kg

D. Intrinsic AKI is generally characterized by the presence of granular casts (indicating structural damage), a fractional excretion of sodium >1, and a urine osmolality similar to that of plasma osmolality (indicating changes in concentrating ability of the kidney)

Which of the following statements is true concerning In-111 pentetreotide? A. it is an analog of guanethidine B. it is primarily a therapeutic radiopharmaceutical C. one day pretreatment with octreotide is recommended D. its planar imaging dosage is different from its SPECT dosage

D. It's a diagnostic radiopharmaceutical and an analog of somatostatin. If possible, octreotide admin should be suspended before admin. Dosage for SPECT imaging is twice as great as planar imaging

A pt requires a cardioselective BB in his or her outpt medication regimen after recent discharge from the hospital with a new MI. You suggest he or she take A. labetalol B. esmolol C. propranolol D. atenolol E. carvedilol

D. Labetalol, propranolol, and carvedilol are nonselective BB. Esmolol is cardioselective but not po. Atenolol is cardioselective and po

Which of the following drugs may produce hypothyroidism? A. amitriptyline B. sertraline C. cholestyramine and ACTH D. lithium and amiodarone E. levothyroxine

D. Lithium and amiodarone have both been associated with hypothyroidism. Amiodarone contains iodine and may cause hypo- or hyper- thryoidism

Which of the following statins must be dosed with the evening meal? A. lipitor B. simvastatin C. pitavastatin D. lovastatin E. rosuvastatin

D. Lovastatin is the only statin that must be dosed with the evening meal to increase its absorption

A 19yo female recently started treatment for bulimia. She is experiencing severe anxiety when eating, fearing that she will lose control. Which medication is most appropriate for her anxiety? A. bupropion SR 150mg B. olanzapine 10mg C. lithium carbonate 300mg D. lorazepam 0.5mg E. zonisamide 100mg

D. Low-dose short acting bzds have shown benefit if severe anxiety associated with eating exists. Olanzapine may improve weight gain and psychological symptoms. Bupropion is ci b/c of the risk of seizures.

A physician calls the pharmacy and provides the name of a medication from another country. Which of the following references would be the best place to search for internationally available medications? A. AHFS drug information B. PDR C. red book D. Martindale

D. Martindale: The Complete Drug Reference contains DI regarding internationally available medications

Sulfur Benzoyl peroxide aa 10% PEG base qs ad 90g Base: PEG 3350 1 part, PEG 400 1.5 parts What would be the maximum BUD for the preparation according to USP Chapter 795 guidelines? A. 14d B. 30d C. 3mo D. 6mo

D. Maximum BUD would be 6mo. Water is contained in the hydrous benzoyl peroxide used to compound the preparation, but that component's container should be labeled with an expiration date. If the expiration date is at least 6mo and no additional water is necessary to make the preparation, then a 6mo BUD is used

35yo is a 5'8'', 52kg. PMH: crohn's dx, intermittent diarrhea Meds: prednisone, mesalamine 1g tid, loperamide albumin 2.5, prealbumin 13 triceps skinfold=3mm (10-14) calf skinfold = 4mm (10-15) What type of malnutrition does this pt have? A. starvation related B. acute disease related C. obesity D. chronic disease related E. injury related

D. Measurements of nutritional assessment are depressed, and the pt has a chronic dx that is contributing to inflammation

Which of the following electrolyte abnormalities typically occurs in pts w/ severe kidney dysfunction (crcl <15)? A. metabolic acidosis B. metabolic alkalosis C. hypophosphatemia D. hypernatremia

D. Metabolic acidosis is a common secondary complication of AKI. Other electrolyte abnormalities include hyperkalemia, hypophosphatemia. Sodium disorders are usually caused by other concomitant disorders, but not by AKI alone.

Which of the following agents has the best activity against the gram-negative anaerobe Bacteroides fragilis? A. clindamycin B. colistin C. fidaxomicin D. metronidazole

D. Metronidazole displays a purely anaerobic spectrum of activity and is the drug of choice for one of the main gram negative anaerobes B. fragilis typically located in the lower GI

Which of the following statements is true? A. Most substances acquire a surface charge by ionization, ion adsorption, and ion dissolution B. The term "surface tension" is used for liquid-vapor and solid-vapor tensions C. At the isoelectric point, the total number of positive charges is equal to the total number of negative charges D. All of the above

D. Most substances acquire a surface charge by ionization, ion adsorption, and ion dissolution. At the isoelectric point, the total number of positive charges is equal to the total number of negative charges

56yo female presents to ED c/o crushing, substernal cp x 3hr, unrelieved by SL NTG. PMH: htn, dm2, tia x2, hld, metabolic syndrome. HR and rhythm regular, no S3/4 sounds present. Vitals: BP184/119, hr100, rr32. EKG ST elevation and VF. Transferred to chest pain center and started on O2. To get PCI. Which of the following agents would be beneficial to this pt? A. tPA 100mg IV over 90min B. IV magnesium C. ppx lidocaine D. metoprolol 12.5mg po E. diltiazem 240mg po

D. One of the relative contraindications for fibrinolytic therapy is severe uncontrolled htn (BP > 180/110mmHg). Routine use of mag post MI is NOT recommended and should be reserved only for pts w/ hypomagnesemia or torsades. BB reduce the incidence of ventricular arrhythmias, recurrent ischemia, reinfarction, infarct size, and mortality in pts w/ STEMI

Which of the following is correct concerning the necessity that operators in the buffer area be properly gowned? A. operators don gowns because they shed particles, and the nonshedding gowns keep them sterile B. sterile gloves are used to avoid contamination of the CSP in case the operator accidentally touches a critical site during compounding of the preparation C. frequent sanitization with sterile 70% isopropyl alcohol is essential to keep the operators hand's sterile during the compounding process D. nonshedding garb and sterile gloves help contain the particles shed from the operators E. operators must don gowns before working at the laminar flow workbench but not before entering the buffer area

D. Operators in the buffer area should wear clean, nonshedding gowns and gloves to help contain the particles that they shed. The sterile gloves are no longer sterile once they are out of the package. Proper aseptic technique must always be used.

Sulfur Benzoyl peroxide aa 10% PEG base qs ad 90g Base: PEG 3350 1 part, PEG 400 1.5 parts PEG 400 is a liquid with a specific gravity of 1.13. What volume should be measured for the preparation? A. 43mL B. 40.3mL C. 38.3mL D. 36.5mL

D. PEG 400 is 1.5 parts of the base. If 1 part is 27.5g, then 1.5 parts = 41.3g. To determine the volume, divide by the specific gravity of 1.13. Thus, 36.5mL is needed

In most critically ill pts, which of the following is the NMB agent of choice? A. propofol B. vecuronium C. cisatracurium D. pancuronium E. any agent may be used first line

D. Pancuronium is the NMB agent of choice for most critically ill pts b/c less expensive

Which of the following should be reserved for the treatment of intracranial htn that is refractory to initial treatment? A. propofol B. mannitol C. hypertonic saline D. pentobarb E. phenytoin

D. Pentobarb should be reserved for treating refractory intracranial hypertension. Propofol, mannitol, and hypertonic saline are first line agents. PHenytoin does not affect ICP

Pharmacists need to be proficient in which of the following skills? A. verbal communication skills only B. written communication skills only C. lit eval skills only D. all of the above

D. Pharmacists need to be proficient in written and verbal communication skills as well as in literature eval skills

Which of the following is associated w/ Michealis-Menton PK? A. carbamazepine B. valproic acid C. topiramate D. phenytoin E. phenobarb

D. Phenytoin has capacity-limited or saturable pk

Addiction is currently understood to be A. characterized by compulsive use of drugs B. synonymous with physical dependence on a medication C. the use of a substance for psychical effects D. A and C E. A and B

D. Physical dependence is the occurrence of a withdrawal syndrome after an opioid is stopped or quickly decreased without titration. Addiction is the psychological dependence on the use of substances for physical effects and is characterized by compulsive use

45yo wf, h/o seizure dx has experienced heartburn after meals intermittently for the past 2 weeks. Worsens when reclining for bedtime. Meds: phenytoin 300 qhs. Sx not troublesome and wants to self-treat w/ OTC meds The pt's sx are NOT relieved after 2 weeks of OTC treatments with famotidine and lifestyle modifications. She states her sx are becoming "troublesome." Which of the following is the best choice? A. add prokinetic agent metoclopramide to famotidine B. endoscopy should be performed because she has sx suggestive of complications of GERD C. add alginic acid 2tabs qhs to famotidine D. d/c current therapy, and initiate therapy w/ omeprazole 20mg qd E. continue famotidine x 1 more week to achieve maximum effectiveness

D. Prokinetic agents are useful mainly in pts w/ concurrent gastric motility disorders and are not routinely recommended. She does not currently exhibit sx of GERD complications. Alginic acid is ineffective when the pt is lying in the supine position and should not be given at bedtime. Non-prescription medications For GERD should be d/c if sx not relieved w/in 2w

The best empiric regimen to treat a prostate infection is A. ciprofloxacin for 10d B. bactrim for 10d C. cipro and bactrim for 10d D. cipro for 4-6w E. bactrim for 4-6w

D. Prostate infx are difficult to treat, requiring 4-6w therapy. Although bactrim is a reasonable choice for treating most prostate infx, cipro if preferred b/c of its ability to concentrate in the prostate fluid

The most common organisms associated with CAP in adults treated as outpts are A. pseudomonas aeruginosa, mycoplasma pneumo, and h. flu B. strept pneumo, h. flu. and klebsiella pneumo C. mycoplasma pneumo, strept pneumo, h. flu, and kleb pneumo D. mycoplasma pneumo, strept pneumo, h. flu, and chlamydophila pneumo E. mycoplasma pneumo, strept pneumo, h. flu, and pseudomonas aeruginosa

D. Pseudomonas aeruginosa is more likely in pts with risk factors for multidrug resistant bacteria such as late-onset HAP or VAP. Kleb pneumoniae is also not commonly associated with CAP.

In a RCT that follows a parallel design, which of the following is true? A. pts serve as their own control B. all pts end up receiving all of the interventions in random different orders, depending on group assignment C. only pts who complete the entire protocol are included in the final analysis D. pts are assigned to groups that receive a particular treatment over time; the only planned difference in the groups is the intervention

D. Pts are assigned to groups that receive a particular treatment over time; the only planned difference in the groups is the intervention

A pt with afib receiving rivaroxaban should avoid which of the following? A. calcium supplements B. orange juice C. digoxin D. clarithromycin E. pravastatin

D. Pts receiving xarelto should avoid clarithromycin because it is a combined pgp inhibitor and strong CYP3A4 inhibitor which leads to increased xarelto plasma concentrations and increased bleeding risk

The antiviral agents with the best spectrum of activity against recent influenza A strains is A. acyclovir B. rimantadine C. amantadine D. oseltamivir and zanamivir E. lamivudine and adefovir

D. Recent influenza A strains have demonstrated marked resistance to amantadine and rimantadine; therefore, these agents are not currently recommended. Acyclovir, lamivudine, and adefovir are not active against the flu

Which life-threatening complication caused the restriction of alosetron? A. SJS B. TEN C. aplastic anemia D. ischemic colitis E. chronic diarrhea

D. Severe constipation, ischemic colitis, and death have been reported w/ alosetron

A 70yo female who has mild congestive heart failure, gastroesophageal reflux disease (GERD), dm2, and RA had a recent cerebrovascular accident. She will not regain her pre-morbid degree of mental status, so a decision is made to give her long-term nutritional support. Which method would be most appropriate for this pt? A. central parenteral nutrition B. nasogastric tube feeding C. peripheral parenteral nutrition D. jejunostomy tube feeding E. nasoduodenal tube feeding

D. She is not a candidate for long-term PN b/c her GI tract would be accessible and functional. Nasogastric and nasoduodenal methods are used only for short-term EN. A jejunostomy would be ideal b/c she also has GERD and perhaps gastroparesis from her dm

Which of the following medications is CI w/in 24hr of a nitrate A. metoprolol B. quinapril C. verapamil D. sildenafil E. felodipine

D. Sildenafil use is CI w/in 24hr of nitrate

Which of the following radiopharmaceuticals exerts it palliative effect through its beta particle emission? A. Tc-99m MDP B. In-111 oxyquinoline C. I-123 iobenguane D. Sm-153 EDTMP

D. Sm-153 EDTMP produces beta particles

Which component is used as an antioxidant to increase the stability of potassium iodide oral solution (SSKI) by preventing the release of free iodine? A. sodium alginate B. sodium glycinate C. sodium succinate D. sodium thiosulfate

D. Sodium thiosulfate is the antioxidant that prevents the iodide ion from oxidizing to form free iodine. The other choices are not antioxidants

Which of the following radiopharmaceuticals is a calcium analog? A. Thallous chloride T1-201 B. Rb-82 chloride C. Florbetapir F-18 D. Strontium-89 chloride

D. Sr-89 chloride; Tl-201 chloride and Rb-82 chloride are K analogs, florbetapir F-18 is not a calcium analog

Which of the following statements about statistics is true? A. data used in statistical analysis have to include all entities in a population B. if an investigator has a research question, she or he has to collect original data C. statistical analysis draws conclusions on the basis of data from the whole population D. statistics can help distinguish random variances from real differences when drawing conclusions

D. Statistical analysis can help researchers to distinguish random variation from real differences when drawing conclusions

The duration of follow-up for a RCT is determined by all of the following except A. outcome to be measured B. long-term safety concerns C. duration of time required to see an effect D. study budget

D. Study budget is not a scientific factor in determining the duration of f/u

Which of the following oral antifungal medications is commonly used to treat onychomycosis? A. ampho B B. caspofungin C. posaconazole D. terbinafine

D. Terbinafine is an oral antifungal agent that is commonly used to treat dermatophytes that cause onychomycosis

Which equation describes the rate of drug dissolution from a tablet? A. Fick's Law B. Henderson-Hasselbalch equation C. Michaelis-Menten equation D. Noyes-Whitney equation

D. The Noyes-Whitney equation describes the rate of drug dissolution from a tablet. Fick's first law of diffusion is similar to the Noyes-Whitney equation in that both equations describe drug movement attributable to a concentration gradient. The Michealis-Menten equation involves enzyme kinetics, whereas Henderson-Hasselbalch equations are used for determination of pH of the buffer and the extent of ionization of a drug molecule

The women's health initiative study found an association between HT and all of the following conditions except A. breast cancer B. stroke C. cvd D. colon cancer E. dvt

D. The WHI study demonstrated a decreased risk of colon cancer with HT

Sulfur Benzoyl peroxide aa 10% PEG base qs ad 90g Base: PEG 3350 1 part, PEG 400 1.5 parts What quantity of sulfur should be weighed for the preparation? A. 7.5g B. 8.5g C. 4.5g D. 9g

D. The abbreviation "aa" means "of each" This indicates that a 10% concentration of both benzoyl peroxide and sulfur are required. Thus, 90g x 0.1 = 9g sulfur

A pt presents to your ambulatory clinic w/ a BP 210/125, PMH: DM2, CHF, CKD Which of the following would cause the pt to be classified as a hypertensive emergency? A. BG > 300, which increase the pt's risk for AKI B. Scr> 3 C. N/B/D x 3d D. S4 gallop and a CXR consistent pulmonary edema E. polyuria combined w/ polydipsia

D. The classification of hypertensive urgencies and emergencies is determined by the presence or absence of acute target organ damage and not by the actual blood pressure measurement. Presence of an S4 gallop and a CXR consistent w/ pulmonary edema suggest acute LVF w/ pulmonary edema, which represents defined target organ damage and in turn means emergency

Which of the following measures of dispersion is independent of the measurement unit? A. range B. variance C. standard deviation D. coefficient of variance

D. The coefficient of variance can be expressed as a percentage of the mean. Because the mean and standard deviation have the same units of measurement, the units cancel out when the coefficient of variation is computed

53yo African American female (65kg) w/ htn and hld is seen in the outpt nephrology clinic for eval of kidney dx progression. Current BP is 156/82, scr 2.6 (stable x past 4mo), BUN 44, urinary albumin excretion rate is 600mg/d. Her medications are enalapril 20mg/d x 1yr and simvastatin 20mg daily x 2years On the basis of this pt's estimated creatinine clearance, she would be classified as which of the following KDIGO categories of ckd? A. G1 B. G2 C. G3a D. G4

D. The pt's estimated clcr is 26 which is classified as stage 4 (15-29)

What is the recommended dose of raloxifene in the prevention and treatment of postmenopausal osteoporosis? A. 10mg qd B. 15mg qd C. 40mg qd D. 60mg qd E. 120mg qd

D. The recommended dose is 60mg po once daily for raloxifene

Which process best describes how to compound the following preparation using available 2% salicylic acid ointment? Salicylic acid 5% White petrolatum qs ad 30g A. weigh 0.5g salicylic acid powder, and qs to 30g with 2% salicylic acid ointment, incorporating the salicylic acid powder by geometric dilution B. weigh 0.5g of salicylic acid powder, levigate it with 5mL of ethyl alcohol, and qs to 30g with 2% salicylic acid ointment, incorporating the salicylic acid powder by geometric dilution C. weigh 1.5g of salicylic acid powder, and qs to 30g with 2% salicylic acid ointment, incorporating the salicylic acid powder by geometric dilution D. weigh 1g of salicylic acid powder, levigate it with 4g of mineral oil, and incorporate it into 25g of 2% salicylic acid ointment by geometric dilution

D. The compounded preparations must contain 1.5g of salicylic acid. 25g of 2% ointment contain 500mg, requiring an addition 1g of salicylic acid powder. The remaining components must not contain any salicylic acid and must consist of a combination of the levigating agent (mineral oil) and white petrolatum. With a specific gravity of 0.89, the 4g of mineral oil would measure 4.5mL A and C are wrong b/c no levigating agent

After termination of an IV constant rate infusion, the plasma conc of a drug declines monoexponentially (C=Co x e^(-k*t)). Conc measure at 2hr and 12hr after the end of the infusion are 12.9mcg/mL and 6mcg/mL, respectively. Calculate the initial conc at the end of the infusion, and predict the conc 24hr after termination of the infusion A. 13.5 and 2.9mcg/mL B. 16.5 and 3.8mcg/mL C. 16.5 and 1.3mcg/mL D. 15 and 2.4mcg/mL E. 15 and 1.3mcg/mL

D. The first step is to calculate the elimination rate constant k from the measured plasma conc k=ln((6/12.9mcg/mL)/(12-2hr) = 0.0765/hr Find Co by plugging in either (2,12.9) or (12,6) C=Co x e^(-0.0765t) 12.9mcg/mL=Co x e^(-0.0765(2)) Co=15mcg/mL Plug in 0 and 24 into the equation C=15 x e^(-0.0765t) C=15 x e^(-0.0765(0)) = 15 C=15 x e^(-0.0765(24)) = 2.39mcg/mL

Which of the following statements is most accurate regarding the use of CCB in IHD? A. amlodipine and felodipine reduce MVO2 by decreasing conduction through the AV node B. CCB should be used as first line therapy in pts with stable angina C. newer-generation dhp like nifedipine ir are safe in the treatment of IHD D. CCB can be used in combo with BB to attenuate the effect of increased sympathetic tone E. the combo of verapamil and metoprolol in a pt with reduced LV systolic fn is safe and well tolerated by most pts

D. The increase sympathetic tone caused by some dhp ccb can lead to a reflex tachy, counteract with using a BB

Which of the following statements is true? A. it is appropriate to estimate a simple linear regression equation for any 2 variables B. a negative intercept of a linear regression means the higher the independent variable C. a positive slope suggests that the regression line crosses the vertical axis above the origin D. the investigator typically has control over the values of the independent variable

D. The independent variable is usually controlled by the investigator, and the investigator may preselect its values

Which of the following statements is true? A. when 2 variables are correlated, only one regression model can be estimated from them B. investigators can preselect values for both variables for a simple linear correlation analysis C. investigators can preselect values of the dependent variable for a simple linear regression analysis D. investigators have some control over the independent variable for a simple linear regression analysis

D. The independent variable is usually controlled by the investigator, and the investigator may preselect its values

What is the emulsifier when lime water and olive oil are triturated together to form a primary emulsion? A. oleic acid B. calcium hydroxide C. calcium oxide D. calcium oleate

D. The lime water is calcium hydroxide solution, and the olive oil contains oleic acid. The two react to form calcium oleate, which is the emulsifying agent that forms the w/o emulsion. Calcium oxide is not an emulsifier

The pk parameters for captopril in healthy adults are Cl=800mg/min, fe=0.5, Vss=0.81L/kg, plasma pr binding 75% When cimetidine (a highly lipid soluble weak base that is highly secreted in the renal proximal tubule) and captopril are coadmin, the renal cl of captopril is reduced to approximately 125ml/min. What is the most likely mechanism to account for this reduction in renal cl? A. cimetidine reduces the filtration cl of captopril B. cimetidine enhances the reabsorption of captopril C. cimetidine increases the unbound fraction of captopril D. cimetidine blocks the renal secretion of captopril

D. The most likely mechanism to account for this reduction in renal clearance is that cimetidine blocks the renal secretion of captopril

Which of the following recombinant blood factors is available in recombinant form for clinical therapeutic use? A. Factor III B. Factor V C. Factor VI D. Factor VII E. Factor X

D. The other factors listed are not clinically available in recombinant form

Which of the following may be used to assess the relative bioavailability of two chemically equivalent drug products in a crossover study? A. Dissolution test B. Peak concentration C. Time-to-peak concentration D. Area under the plasma-level time curve

D. The plasma drug concentration versus time curve measures the bioavailability of a drug from a product. The peak plasma drug concentration (Cmax) relates to the intensity of the pharmacologic response, while the time for peak plasma drug concentration (Tmax) relates to the rate of systemic absorption

Which of the following sx or findings is consistent w/ the diagnosis of gout? A. the presence of positively birefringent crystals in the affected synovial joint fluid B. the presence of calcium pyrophosphate in the affected synovial joint fluid C. the presence of symmetrically swollen joints D. the presence of hyperuricemia

D. The presence of calcium pyrophosphate is indicative of pseudogout. Presence of NEGATIVE birefringent crystals at affected site is definite gout. Hyperuricemia is a suggestive sx of gout

SJ is a 30yo white female with a 10y history of schizophrenia. Her current therapy includes haloperidol 10mg po bid. The psychiatrist wants to convert her to haloperidol decanoate. What would be the appropriate equivalent monthly dose of haloperidol decanoate? A. 100mg B. 10mg C. 500mg D. 200mg E. 12.5mg

D. The pt is on haloperidol 10mg bid. Therefore, the total oral daily dose is multiple by 10. The dose would be Haldol-D 200mg IM q 4weeks. 10 mg bid = 20mg/day x 10 = 200mg IM q4w

A pediatric pt receives an immunosuppressive therapy with oral cyclosporine solution. His concentration-adjusted dosing regimen is 85mg q12hr. Because of a recent change in his insurance coverage, he needs to be switched from the drug product he is currently using to a generic solution dosage form of cyclosporine that is covered by his insurance. The bioavailability of the dosage form he previously used in 43%; the bioavailability of the generic dosage form is 28%. What is the appropriate dosage form to maintain the same systemic exposure as obtained from the previously used dosage form? A. 25mg q12h B. 55mg q12h C. 184mg q12h D. 130mg q12h E. 305mg q12h

D. The systemic exposure or avg ss conc for an oral dosing regimen is given by Css,av= (FxDR)/CL where DR is the dose rate and F the oral bioavailability of the respective dosing regimens. If Css,av should be maintained constant, if follows that Css,av=(F1xDR1)/CL=(F2xDR2)/CL or F1xDR1=F2xDR2 where the subscript denotes the different dosing regimens. Thus DR2, the dose rate for the generic dosage form, can be calculated as DR2= (F1xDR1)/F2 = (0.43x85mg/12hr)/0.28 = 130.5mg/12hr

Which of the following statements is true? A. if the test statistics falls into the non-rejection region, the test is said to be significant B. the rejection region can be on only one tail of the distribution C. researchers typically prefer higher levels of significance D a test statistic can assume different values when different samples are drawn from the population

D. The test statistic is a numerical value calculated from the data in the sample. The test statistic can assume many different values, and the particular sample determines the specific value that the test statistic assumes

Which of the following drugs could adequately treat a skin and ssti caused by CA-MRSA? A. metronidazole B. fidaxomicin C. colistimethate D. bactrim

D. The three antibiotics commonly used to treat CA-MRSA and SSTI are doxycycline, clindamycin, and bactrim

How many nucleotides (or codons) exist for the coding of the 20 possible amino acids specified by the genetic code? A. 4 B. 12 C. 20 D. 61 E. 64

D. There is a degeneracy in the genetic code. Some amino acids may encode as many as 6 codons, whereas others may be encoded by only one. Of the 64 possible codons, 3 are stop codons.

A pt who is suffering from heat intolerance, weight loss, tachycardia, tremor, and anxiety may be treated w/ A. apap B. mitotane C. cyproheptadine D. propylthiouracil E. diazepam

D. These are cardinal features of hyperthyroidism. PTU and methimazole are effective in reducing the excessive thyroxine level

39yo f presents w/ mild abd pain and diarrhea x 12w. No significant PMH and occasionally misses days from her full-time job. Her sx are worse after meals. Which of the following is the best choice for initial therapy? I. paroxetine 10mg qd II. amitriptyline 10mg qhs III. tegaserod 6mg bid IV. alosetron 1mg qd V. dicyclomine 10mg qid after meals A. I, II, and III B. I and III C. II and IV D. V only E. all of the above

D. This pt has mild, IBS-D. Symptomatic treatment with dicyclomine is appropriate initial therapy, especially b/c her symptoms are meal related. A TCA may be added to dicyclomine if needed. Tegaserod is indicated for IBS-C. Alosetron is reserved for pts w/ severe, IBS-D who have failed other therapies. Paroxetine may be added if initial therapies are ineffective or if she develops severe abdominal pain or psychiatric comorbidities

A 26yo female who is recently initiated on a combination hormonal oral contraceptive complains of late-cycle breakthrough bleeding. Which of the following is she most likely experiencing? A. too much estrogen B. too little estrogen C. too much progestin D. too little progestin E. too much androgen

D. Too little progestin may result in breakthrough bleeding late in the menstrual cycle. She should be changed to a product with a higher progestin content

Which of the following opioids has the longest duration of analgesic effect? A. methadone B. CR morphine C. hydromorphone D. transdermal fentanyl E. CR oxycodone

D. Transdermal fentanyl provides analgesia for up to 72hr. The analgesic effects of methadone do not correlate w/ its long half life

Which of the following effects on myocardial o2 demand is NOT affected by BB? A. decreased HR B. decreased BP C. decreased contractility D. peripheral vasodilation E. decreased conduction through the AV node

D. Unlike nitrates or CCB, BB do not cause peripheral vasodilation

A solution containing 55% ethyl alcohol is required. Solution of 15% and 80% ethyl alcohol are available. What volume of the 15% solution is required to make 100mL? A. 64.8mL B. 41.7mL C. 61.5mL D. 38.5mL

D. Using alligation: 25parts/65parts=x/100mL 80____________ = 55-15 = 40 parts of 80% 55 15_____________ = 80-55 = 25 parts of 15% TOTAL = 65 parts (note: larger number on top!)

Which of the following carries an FDA boxed warning for pancreatitis? A. carbamazepine B. felbamate C. zonisamide D. valproic acid E. phenytoin

D. Valproic acid may cause fatal hemorrhage pancreatitis

For which of the following antibiotics should weekly CPK levels be drawn? A. vanco B. linezolid C. quinupristin-dalfopristin D. daptomycin

D. Weekly CPK levels should be drawn when pts are initiated on daptomycin therapy. Daptomycin therapy should be discontinued when CPK levels reach 5 times the ULN when pts are symptomatic (commonly myalgias), or 10 times the ULN when asymptomatic. Monitoring should be performed closely when pts are on concomitant statin therapy

Which of the following statements is true about statistical hypotheses? A. an alternative hypothesis is a statement presumed to be true in the study population B. the null hypothesis is typically the same as the research hypothesis C. a statistical test can either prove or reject a null hypothesis D. a statistical test cannot prove a null hypothesis

D. When hypothesis testing does not reject the null hypothesis, it does not mean proof the null hypothesis. Hypothesis testing indicates only whether the available data support or do not support the null hypothesis

Which of the following statements is NOT true for type I and type II errors? A. a type I error occurs when a true null hypothesis is rejected B. a type I error cannot occur at the same time as a type II error C. a type II error occurs when a false null hypothesis is not rejected D. a type I error occurs when a true hypothesis is not rejected

D. When the null hypothesis is true, but the statistical decision is to reject the null hypothesis, the investigator has committed a type I error

Which of the following statements is true? A. for a population parameter, an investigator cannot produce both a point estimate and an interval estimate B. the lower limit of a confidence interval is also called the precision of the test C. the reliability coefficient is always determined from a t distribution D. the reliability coefficient for the confidence interval can be determined on the basis of the standard normal distribution only when certain conditions are met

D. When the sample size is large, reliability coefficients can be obtained from the standard normal distribution

Which of the following products is available in injectable and nasal spray dosage form A. raloxifene B. alendronate C. teriparatide D. calcitonin E. prempro

D. calcitonin; teriparatide is inj qd only, all others are oral

ALL of the following are true of acarbose except A. it's contraidicated in ibs B. it should be take w/ the first bite of each meal C. it does NOT cause hypoglycemia or weight gain D. hypoglycemia attributable to combination therapy when treated w/ sucrose E. LFTs are given q3mo during first year and periodically thereafter (if dose > 50mg tid)

D. give oral GLUCOSE not sucrose

Which of the following is the most appropriate treatment for severe hypoglycemic episode when a person is unconscious A. 1/2 cup diet soda B. 3 hard candies containing sugar C. 1 glucose tablet D. glucagon injection E. 2-3 small sugar cubes

D. glucagon, a pancreatic hormone that is given parenterally, is the most appropriate treatment for a severe hypoglycemic episode b/c the pt may be unconscious and not able to take po

Which of the following is an indication that a pt is developing a long-term complication of DM? A. tremor B. glycosuria C. leukocytosis D. proteinuria E. tinnitus

D. indication of nephropathy

Ideal properties for a BB in the treatment of ACS include which of the following? A. available as an IV product, cardioselectivity B. low lipophilicity, has ISA C. has ISA, cardioselectivity D. cardioselectivity, low lipophilicity, does not have ISA E. noncardioselective, high lipophilicity

D. po preferred to avoid ade. cardioselect to reduce chance for broncospasm. low lipophlicity would reduce neurological ade. bb w/ isa reduce hr to a lesser degree than non-isa bb so not as good at lowering o2 demand

Which of the following statements is true? A. parametric tests are always preferred over nonparametric tests B. when a sample size is large, a nonparametric test has the advantage of being more easily and quickly applied than a parametric test C. the investigator needs to known the form of the population distribution to use a nonparametric test D. the sign test takes into consideration only the positive and negative signs of the data

D. the sign test takes into consideration only the positive and negative signs of the data. The raw data used in the calculation of the test statistic for the sign test are plus and minus signs

Following major GI resection, a pt w/ severe short bowel syndrome is started on PN. It is anticipated that this pt may need this therapy for 6mo to 1year. The PN prescription for this pt includes D20W aa 5% at 105mL/h (2500mL/d) IV fat emulsion 20% at 10mL/h x 24h (240mL/d) 0.45% saline at 50ml/h x 24hr (1200mL/d) How many grams of protein will this pt receive each day? A. 25 B. 50 C. 75 D. 100 E. 125

E. 5% aa = 5g/100mL 2500mL x 5g/100mL = 125g aa 1g aa = 4kcal

Which of the following are advantages of using a 0.22micron filter when administering a two-in-one PN formulation? I. traps particulate matter II. prevents precipitates from entering the pt III. filters most bacteria A. I only B. III only C. I and II only D. II and III only E. all of the above

E. A 0.22 micron filter will do all three; in contrast, a 1.2 micron filter will NOT filter bacteria

Which of the following agents would NOT be administered at the same time as ufh? A. tPA B. reteplase C. eptifibatide D. TNKase E. streptokinase

E. A combo of UFH and streptokinase is not desirable because streptokinase is a nonspecific fibrinolytic, and UFH may increase the risk of bleeding b/c of streptokinase's long half life. Heparin should be given for at least 48hr with the other lytic choices to reduce risk of reocculsion

Which of the following statements is true regarding the ade of ergotamine? A. ergotamine inhibits the chemoreceptor trigger zone to minimize nausea and vomiting B. ergotamine has minimal risk of dependence C. muscle weakness is an uncommon side effect of ergotamine D. angina-like pain reported with the triptans is not seen with ergotamine use E. overuse of ergotamine can result in ischemia

E. ADE of ergotamine include N/V, physical dependence, muscle weakness, and angina-like pain. Overuse may result in ischemia and may progress to gangrene

What is the recommended dosage of daily calcium intake for a 65yo women? A. 250mg B. 500mg C. 750mg D. 1000mg E. 1200mg

E. According to the nationals osteoporosis foundation, daily calcium intake should be 1000mg/d for men age 50-70 and 1200mg per day for women age 51 and older and men aged 71 and older.

Which of the following agents can be used to treat genital herpes simplex infx? I. acyclovir II. famciclovir III. valacyclovir A. I only B. III only C. I and II only D. I and III only E. all of the above

E. Acyclovir, famciclovir, and valacyclovir are all approved for the treatment of genital herpes

Which of the following is correct concerning placement of items and work performed in the laminar flow workbench? A. items should be placed in a horizontal flow hood to the right or left of the work area B. items in a vertical laminar flow hood should be placed so that an operator's hand never goes over the top of a critical site while the operator is working in the hood C. An object placed in a horizontal laminar flow hood disturbs the airflow downstream of the object equal to 3 times the diameter of the object D. when working in a horizontal laminar flow workstation, an operator must perform all work at least 6 inches inside the hood E. all of the above

E. All are correct

Which of the following is true regarding bulimia? I. the individual may consume 5,000-20,000 calories in a single binge episode that may last as long as 2-8hrs. II. the two specific types of bulimic pts are the purging type and the nonpurging type III. there is an average of 2 binges per week for 3 consecutive months A. I only B. II only C. I and III only D. II and III only E. all of the above

E. All of the answers

Which of the following factors should be considered when choosing a sterilizing filter? A. the volume of solution to be filtered B. the compatibility of the membrane with the solution to be filtered C. whether the solution to be filtered is hydrophobic or hydrophilic D. the compatibility of the filter housing with the product to be filtered E. all of the above

E. All should be considered

56yo female presents to ED c/o crushing, substernal cp x 3hr, unrelieved by SL NTG. PMH: htn, dm2, tia x2, hld, metabolic syndrome. HR and rhythm regular, no S3/4 sounds present. Vitals: BP184/119, hr100, rr32. EKG ST elevation and VF. Transferred to chest pain center and started on O2. To get PCI. Which of the following regimens is appropriate for this pt at this time? A. asa 81, plavix 600, and lovenox 1mg/kg sq bid B. asa 81, plavix 600, ufh 60u/kg bolus followed by ufh 12u/kg/h C. ticagrelor 180mg and eptifibatide 180mg/kg/iv bolus followed by 2mcg/kg/h infusion D. asa 325, prasugrel 60mg, bivalirudin 0.75mg/kg iv bolus followed by 1.75mg/kg/h E. asa 325, ticagrelor 180, bivalirudin 0.75mg/kg iv followed by 1.75mg/kg/hr

E. Antiplatelet therapy during pci includes asa, a p2y12, and appropriate antithrombotic agent. Dosing of asa during acute even is at least 162. If h/o tia/ stroke, do NOT give prasugrel (ABSOLUTE CI)

JB is an 18yo wf diagnosed w/ gonorrhea B/c JB has gonorrhea, she should also receive treatment for A. syphilis B. HIV C. hepatitis D. genital herpes E. chlamydia

E. B/c of the high rate of comorbid chlamydia infx, pts diagnosed with gonorrhea should also receive therapy against chlamydia: azithromycin 1g orally once or doxy 100mg orally bid x 7d

The pharmacist receives a prescription for Fosamax 70mg daily for prevention of osteoporosis with instructions to the pt to take w/ food and remain upright for at least 30 min following ingestion. From the following choices, identify the errors in this prescription. A. the dose of Fosamax should be 35 weekly for prevention B. fosamax should be taken at least 30 min prior to a meal C. fosamax should not be taken with food D. pts should lie down for 1 hr following administration of fosamax E. choices A, B, and C are correct

E. Correct dose is 35mg once weekly or 5mg daily. The 70mg dose is for treatment of osteoporosis. The medication should be taken with a full glass of water at least 30 min prior to ingesting food or other beverages. Pts should remain upright for at least 30 min following ingestion of fosamax

Which of the following describes appropriate selection of initial resuscitation fluid in severe sepsis or septic shock? A. either crystalloids or colloids are acceptable choices B. colloids are recommended as a first-line treatment b/c they have a lower risk of renal failure C. albumin is recommended first, crystalloids may be added if refractory D. crystalloids and albumin should be started together E. crystalloids are recommended first; albumin may be added if refractory

E. Crystalloids first then albumin is acceptable if pt's hypotension is refractory. Nonalbumin colloids cause more renal dysfn and are not recommended

Approximately how many people in the US experience severe chronic pain? A. 10 mil B. 23 mil C. 40 mil D. 50 mil E. 76 mil

E. Currently 76mil ppl suffer from some sort of chronic benign pain

In which situations should hydromorphone or fentanyl be used for analgesia in critically ill pts? I. morphine allergy II. renal dysfn III. hemodynamic instability A. I only B. III only C. I and II only D. II and III only E. all of the above

E. Morphine may cause more hemodynamic instability than does hydromorphone or fentanyl b/c of more histamine release. In addition, morphine has a renally excreted, partially active metabolite that may accumulate in renal failure. Hydromorphone and fentanyl do not have such a metabolite.

Following major GI resection, a pt w/ severe short bowel syndrome is started on PN. It is anticipated that this pt may need this therapy for 6mo to 1year. The PN prescription for this pt includes D20W aa 5% at 105mL/h (2500mL/d) IV fat emulsion 20% at 10mL/h x 24h (240mL/d) 0.45% saline at 50ml/h x 24hr (1200mL/d) Calculate the daily nitrogen balance (g/d) in this pt if she excretes 12g of urea nitrogen during the urine collection and 4g are used as insensible and stool losses each day A. -4 B. -2 C. 0 D. 2 E. 4

E. NB= nitrogen in - nitrogen out Nitrogen intake = protein intake (125g from aa) / 6.25 = 20g Nitrogen out = 12g + 4g = 16g NB = 20-16 = 4 A NB of 4 would be suggestive of nutritional adequacy with this PN formulation

Which of the following is associated w/ the development of progressive multifocal leukoencephalopathy? A. prednisone B. sulfasalazine C. mesalamine D. mtx E. natalizumab

E. Natalizumab has been associated w/ the development of progressive multifocal leukoencephalopathy

All of the following are useful in the rapid treatment of severe hyperkalemia except A. potassium restriction B. IV calcium C. IV regular insulin and dextrose D. IV sodium bicarb E. oral kayexalate

E. Oral kayexalate does not act very quickly. It requires transit time through the intestines to bind k and create a gradient that pulls more k into the lumen of the GI tract

The possible benefits of LMWH over UFH include all of the following except A. predictable response B. ease of administration C. no recommended routine monitoring D. lower incidence of heparin-induced thrombocytopenia E. no renal adjustment necessary

E. Renally adjust LMWH at CrCl <30mg/min

Pts with HFrEF who experience fluid retention after BB initiation should have A. the BB dose increased B. the digoxin dose increased C. the BB d/c D. the ACEI d/c E. the diuretic dose adjusted

E. Some pts with HF may experience increases in fluid retention after initiation of BB therapy. Fluid retention typically is managed best by adjusting the diuretic dose and closely monitoring the pt's weight

Which of the following is an important consideration when using BB for treating HFrEF? A. they are effective only in post MI pts B. all BB are equally effective for the treatment of HFrEF C. therapy should be initiated at the target dose D. pts with fluid overload are the optimal candidates for initiating therapy E. therapy should be initiated at low doses and titrated upward slowly

E. Start at low doses and gradually titrate to target dose. Start only when pt stable and not too fluid overloaded. Only carvedilol, toprol xl, and bisoprolol approved for HF

Which of the following has the most potent LDL lowering effect? A. nicotinic acid B. fibric acids C. omega-3 fatty acids D. cholesterol inhibitors E. HMG-CoA reductase inhibitors

E. Statins reduce LDL by 18-55%

Which of the following dosage forms may use surface-active agents in their formulations? A. Emulsions B. Suspensions C. Colloidal dosage forms D. Creams E. All of the Above

E. Surface active agents facilitate emulsion formation by lowering the interfacial tension between the oil and water phases. Adsorption of surfactants on insoluble particles enables these particles to be dispersed in the form of a suspension

Which of the following medications should be avoided in female liver transplant recipients? A. tacrolimus B. cyclosporine C. mycophenolate sodium D. azathioprine E. tacrolimus ER

E. Tacrolimus XL is associated w/ increased mortality in female transplant pts

Which of the immunosuppressive medications listed may cause new onset diabetes after transplant. A. horse antithymocyte globulin B. azathioprine C. mycophenolate mofetil B. basiliximab E. tacrolimus

E. Tacrolimus may cause new onset dm after transplant

Which of these medications is most likely to worsen opioid induced constipation? A. paxil B. zocor C. lotensin D. premarin E. elavil

E. The anticholinergic effects of tricyclic antidepressants such as Elavil can exacerbate opioid induced constipation and urinary retention

What is the maximum dose in kcal/kg/d of dextrose in PN for adult pts? A. 5 B. 10 C. 15 D. 20 E. 25

E. The dose of dextrose in PN should never exceed 5mg/kg/min in adult pts. This dose can be converted to 25kcal/kg/d

What is the highest dose of estrogen (ethinyl estradiol) offered in an oral contraceptive? A. 25mcg B. 30mcg C. 35mcg D. 40mcg E. 50mcg

E. The highest dose of estrogen is 50

Which parts of the needles are considered critical sites? A. the hub only B. the needle shaft only C. the hub, bevel, and bevel tip only D. the needle shaft, bevel, and bevel tip only E. the hub, needle shaft, bevel, and bevel tip

E. The hub, the needle shaft, bevel, and bevel tip are all critical sites

All of the following may decrease the effect of thyroid hormone supplementation except A. antacids B. bile acid sequestrants C. estrogens D. sucralfate E. theophylline

E. Theophylline is affected by Synthroid NOT the other way around. Numerous drugs are known to decrease thyroid hormone absorption including antacids, calcium salts, magnesium, sucralfate, and bile acid sequestrants. Estrogens, soybean formula, and enzyme-inducing epileptic drugs may decrease circulating thyroid hormone levels and necessitate a dose increase of thyroxine. Synthroid also causes an increase in warfarin effectiviness but a decrease in antidiabetic drugs, digoxin, and theophylline.

Which of the following is an emulsifying agent? A. Span 80 (Sorbitan monooleate) B. Tween 80 (Polyoxethylene sorbitan monooleate) C. SLS D. Gum acacia E. All of the above

E. They are all emulsifiers

Which of the following is affected by medications that inhibit cyp systems? A. tegaserod B. alosetron C. fibercon D. PEG E. amitriptyline

E. Tricyclic antidepressant serum conc are affected by cyp interactions

Which of the following describes the appropriate use of hydrocortisone in severe sepsis or septic septic shock? A. use in all pts with severe sepsis or septic shock B. use in all pts on vasopressors (ie in septic shock) C. use only in pts w/ documented adrenal insufficiency D. if started, treat until the pt is discharged from the ICU E. use only if BP is refractory to fluids and vasopressors

E. Use only in pts who are refractory to fluids and vasopressors. The typical duration is 7d

50yo man. PMH: htn, recent-onset dm2, and hld. FH nonsignificant. SH nonsignificant. NKDA. Occasionally takes APAP for h/a and no other OTC meds or herbal products. Current meds: hctz 25qd, lipitor 10qd. BP 144/90, pulse 70, 185lb, 5'9''. a1c: 7.3%. TC 250, HDL 40, TG 145 What is the pt's LDL cholesterol A. 130mg/dl B. 153mg/dl C. 162mg/dl D. 178mg/dl E. 181mg/dl

E. Use the Friedewald equation to calculate LDL LDL = TC - (HDL + TG/5) LDL = 250 - 40 - (145/5) LDL = 181mg/dl

Dosage adjustment should be considered when warfarin is administered with the following drugs except A. amio B. sotalol C. quinidine D. propafenone E. diltiazem

E. Warfarin is metabolized by CYP2C9, 1A2, 3A4 Amio inhibits 2C9, 1A2, and 3A4 Quinidine interaction is unknown mechanism Propafenone inhibits 1A3, 3A4 Sotalol is primarily renally eliminated and does not result in CYP interactions

Pramlintide (symlin) A. is a basal insulin B. is an insulin analog C. is an oral insulin D. is an inhaled insulin E. is an amylin analog

E. amylin analog


Ensembles d'études connexes

Principles of Accounting 1 - Chapter 1

View Set

FA - Midterm 3 Smartbook Questions

View Set

Forensics Ch 9 Review (Firearms, Tool Marks & Other Impressions)

View Set

Ch 5 Entrepreneurship and Starting a Small Business SmartBook...

View Set